Gastroenterology and Hepatology

अब Quizwiz के साथ अपने होमवर्क और परीक्षाओं को एस करें!

Gastro 59 A 57-year-old woman is evaluated as a new patient. She is asymptomatic and has no significant medical history. Her father died of a myocardial infarction at age 71 years, and her 81-year-old mother is alive and healthy. There is no family history of cancer, but the patient has a friend who was recently diagnosed with colon cancer and is concerned about her risk for the disease. On physical examination, vital signs are normal; BMI is 25.5; the rest of the physical examination is normal. Laboratory results are all within normal limits. Which of the following is an appropriate screening strategy for colon cancer in this patient? A Annual digital rectal examination every year B Colonoscopy every 10 years C Double-contrast barium enema every 3 years D Flexible sigmoidoscopy every 10 years

B Screening for colorectal cancer in the average-risk population should be started at age 50 years. This patient is at average risk for colorectal cancer; this population includes persons with no personal or family history of colon adenoma or cancer and who do not have a condition that predisposes them to cancer, such as inflammatory bowel disease. Screening for colorectal cancer is cost effective and well tolerated; it also saves lives. Screening is feasible because the 10 to 15 years needed for a polyp to develop into cancer are sufficient time to detect and remove an adenoma. Screening in the average-risk population should be started at age 50 years; there are various effective screening strategies in average-risk patients, with surveillance intervals depending upon chosen strategy. Endoscopic screening of the distal colon with flexible sigmoidoscopy decreases colon cancer diagnoses in the screened colonic segment and decreases colorectal cancer-related mortality by 45%. Colonoscopy reduces the incidence of colorectal cancer by 76% to 90% when compared with control populations. Advanced adenomas (≥1 cm in diameter, villous histology, high-grade dysplasia or cancer) are detected three times more often by sigmoidoscopy than by fecal occult blood testing. Annual fecal occult blood testing is an option but requires that two samples be collected from each of three spontaneously passed stools. Digital examination to retrieve a sample is not an acceptable substitute. Flexible sigmoidoscopy in combination with fecal occult blood testing in 5-year intervals would be an option as well. Double-contrast barium enema is no longer considered to be a primary method of colorectal cancer screening by the U.S. Preventive Services Task Force. If double-contrast barium enema must be used for technical or anatomical reasons, the correct screening interval is 5 to 10 years.

Gastro 46 A 78-year-old woman is evaluated in the hospital after being admitted 5 days ago for a 2-week history of abdominal pain and nausea, along with black, tarry stools for the past 36 hours. On day 1 esophagogastroduodenoscopy showed a clean-based bleeding gastric ulcer that was positive for Helicobacter pylori infection; the ulcer was treated with injection therapy and coagulation therapy with probe cautery, and proton pump inhibitor therapy was initiated. The bleeding did not stop, and esophagogastroduodenoscopy was repeated on day 3 with endoclip therapy. The bleeding continued, and the patient has received eight units of packed erythrocytes. On physical examination on day 5, the temperature is 37.2 °C (99.0 °F), the blood pressure is 95/50 mm Hg, the pulse rate is 103/min, and the respiration rate is 16/min. Rectal examination reveals melanotic stool. Laboratory studies reveal hemoglobin of 10.8 g/dL (108 g/L); all other tests, including coagulation parameters, are normal. Which of the following is the most appropriate next step in the management of this patient? A Bleeding scan B Helicobacter pylori eradication therapy C Intravenous octreotide D Surgery

D Surgical treatment of upper gastrointestinal bleeding should be considered when endoscopic therapy fails. Most cases of upper gastrointestinal bleeding can be effectively treated with medical and endoscopic therapy. The most common need for surgical treatment in such bleeding is failure of the bleeding to respond to endoscopic treatment. Surgical consultation should also be considered in patients who rebleed, in patients who require a large number of transfusions, and for large ulcers of the lesser curve of the stomach or posterior wall of the duodenum. The diagnosis of bleeding from a gastric ulcer has already been made in this patient, and therefore no further diagnostic modalities such as a bleeding scan are required. Some studies have suggested the use of intravenous octreotide in the use of peptic ulcer bleeding, but no data supports its effectiveness after endoscopic therapy fails. In patients with Helicobacter pylori, eradication prevents ulcer recurrence but has no effect on the outcome of patients with acute upper gastrointestinal bleeding. However, all patients with bleeding peptic ulcers should receive high-dose proton pump inhibitor therapy. A meta-analysis showed that adjuvant high-dose proton pump inhibitor therapy following endoscopic hemostasis for ulcers at high risk of rebleeding reduces rebleeding, surgery, and mortality.

Gastro 82 A 47-year-old man is evaluated in the emergency department for 7 days of odynophagia with epigastric pain, nausea and vomiting, diarrhea, and low-grade fever; he has lost 4.5 kg (10 lb) during the episode. The patient received a kidney transplant 6 months ago for hypertensive kidney disease; his medications are atenolol, prednisone, tacrolimus, and mycophenolate mofetil. He has no other medical problems and denies any HIV risk factors. On physical examination, the temperature is 38.0 °C (100.8 °F), the blood pressure is 148/94 mm Hg, the pulse rate is 90/min, and the respiration rate is 22/min. Esophagogastroduodenoscopy shows a 2.5-cm esophageal ulcer with raised borders. The rest of the esophagus appears normal. Biopsy specimen from the base of the ulcer shows intense inflammatory infiltrates with granulation tissue associated with occlusion body cells. Which of the following is the most likely diagnosis? A Candida albicans esophagitis B Cytomegalovirus esophagitis C HIV esophagitis D Pill-induced esophageal ulcer

B Sudden-onset odynophagia in an immunosuppressed patient suggests infectious esophagitis. The sudden onset of odynophagia should raise suspicion for inflammatory conditions of the esophagus. In the absence of risk factors for caustic, radiation-induced, or pill-induced esophageal injury, infectious esophagitis is frequently the diagnosis, especially in immunosuppressed patients. In the absence of oral thrush and endoscopically visible esophageal plaques, candidal infection is less likely than cytomegalovirus-induced ulcerations. The viral cytopathic effect seen on microscopy confirms the diagnosis. Furthermore, candidal esophagitis is more likely to present with dysphagia rather than odynophagia. Primary HIV ulcers can be visibly indistinguishable from cytomegalovirus ulcers, but the lack of HIV infection risk factors and the characteristic histologic findings make such a diagnosis unlikely in this case.

Gastro 41 A 48-year-old man is evaluated 8 weeks after having been hospitalized for an episode of sudden nausea, hematemesis, and profound hypotension. He was treated in the emergency department with fluids and an erythrocyte transfusion. An esophagogastroduodenoscopy showed a 1.5-cm clean-based ulcer in the duodenal bulb; biopsy specimens of the body of the stomach and antrum showed chronic active gastritis and the presence of Helicobacter pylori. He was discharged from the hospital and prescribed a 10-day course of omeprazole, clarithromycin, and amoxicillin. He had taken omeprazole for the past 8 weeks and has just completed therapy before his visit. The patient's father had gastric cancer. On examination, the patient is asymptomatic and afebrile; the blood pressure is 118/72 mm Hg and the pulse rate is 88/min. Physical examination and complete blood count are normal. Which of the following is the most appropriate next step in the management of this patient? A Helicobacter pylori serum antibody test B Helicobacter pylori urea breath test in 8 weeks C Helicobacter pylori urea breath test now D Repeat esophagogastroduodenoscopy with biopsies E No further testing

B The two noninvasive tests for determining Helicobacter pylori eradication are the stool antigen test and the urea breath test; the sensitivity of breath testing is adversely (nachteilig) affected by recent therapy with a proton pump inhibitor or antibiotic. Although testing for eradication of Helicobacter pylori is not cost-effective or practical in most cases, in this patient with a history of significant gastrointestinal bleeding from H. pylori infection requiring hospitalization and transfusion, eradication testing would be important to prevent future complications. In addition to patients with H. pylori-associated ulcers, eradication testing should be considered in patients with previous early-stage gastric cancer, H. pylori-related MALT lymphoma, or persistent dyspepsia after a test-and-treat approach. The father's history of gastric cancer would be another compelling reason to evaluate for eradication of the organism in this patient. Urea breath testing is a noninvasive test for evaluation of H. pylori eradication, but the sensitivity of the test can be markedly reduced in patients taking a proton pump inhibitor or recent antibiotics, and therefore, waiting to perform this test in 4 to 8 weeks after recent discontinuation of omeprazole would be recommended rather than testing now, which may lead to false negative results. Although serum antibody testing is widely available and has good negative predictive value, it is not recommended after therapy because sustained antibody titers cannot easily distinguish between active and treated infection. Because this patient had a duodenal rather than gastric ulcer, a follow-up esophagogastroduodenoscopy to assess for healing and rule out malignancy is not required; in addition, repeating the procedure to evaluate for H. pylori eradication would not be a cost-effective testing method and would only be recommended if additional histopathologic evaluation were needed, for example, if dysplasia had been found initially. To do no further testing would not be recommended in this patient with a significant H. pylori-associated ulcer bleed.

Gastro 39 A 26-year-old man is evaluated in the emergency department for shortness of breath and fatigue. He has had maroon stools for the past 2 weeks associated with occasional mild, crampy periumbilical pain but no severe pain. He has had no stools for the past 48 hours. On physical examination, the temperature is 37.1 °C (98.8 °F), the blood pressure is 94/62 mm Hg supine, the pulse rate is 116/min, and the respiration rate is 14/min; the BMI is 19.5. He has pale conjunctivae and regular tachycardia. The chest is clear and the abdomen is normal with no organomegaly. Examination of the stool is positive for occult blood. Laboratory studies reveal hemoglobin 6.8 g/dL (68 g/L) with a mean corpuscular volume of 74 fL and a platelet count of 446,000/µL (446 × 109/L); serum biochemistry tests, prothrombin time, activated partial thromboplastin time, and INR are normal. Following volume resuscitation, esophagogastroduodenoscopy is normal and colonoscopy shows some old blood but no active bleeding. Wireless capsule endoscopy fails to identify a source of bleeding. Which of the following is the most appropriate next step in the evaluation of this patient? A Small-bowel barium contrast study B CT scan of the chest and abdomen C Mesenteric angiography D Technetium-99m (99mTc) pertechnetate (Meckel) scan

D Bleeding from a Meckel diverticulum may be a source of obscure bleeding in young patients. In a young patient, the next best test of the options provided to identify a source of bleeding would be a technetium-99m (99mTc) pertechnetate scan (Meckel scan) to identify the presence of Meckel diverticulum. Meckel diverticulum is the most common congenital anomaly of the gastrointestinal track and is located near the ileocecal valve. It often contains heterotopic gastric mucosa that ulcerates and bleeds. Technetium-99m (99mTc) pertechnetate has an affinity for the gastric mucose, and the Meckel scan identifies the heterotopic mucosa. Bleeding from a Meckel diverticulum is common in children but is an uncommon cause of blood loss in adults. CT scan does not provide a luminal view of the intestine and is not likely to identify a bleeding source in this patient. Mesenteric angiography is not the appropriate next step because the patient does not have active, ongoing, brisk bleeding that would be detected and treated with angiography. A small-bowel barium study would not be a good choice; it obscures endoscopic visualization, has poor sensitivity for identifying sources of blood loss, and does not offer therapeutic options.

Gastro 75 A 28-year-old woman who is 30 weeks pregnant is evaluated for a 2-week history of pruritus and scleral icterus. It is her first pregnancy, and she has no significant medical history; she does not drink alcohol and takes only a prenatal vitamin. On physical examination, the patient is afebrile; the blood pressure is 100/70 mm Hg, the pulse rate is 72/min, and the respiration rate is 15/min. Examination reveals a gravid uterus, mild scleral icterus, and linear excoriations on the skin; there is no ascites or lower extremity edema. Laboratory studies: Hemoglobin 13.4 g/dL (134 g/L) Platelet count 275,000/µL (275 × 109/L) Bilirubin (total) 4.2 mg/dL (71.8 µmol/L) Bilirubin (direct) 2.3 mg/dL (39.3 µmol/L) Aspartate aminotransferase 44 U/L Alanine aminotransferase 38 U/L Alkaline phosphatase 180 U/L Lactate dehydrogenase 82 U/L INR 1.0 Hepatitis B surface antigen Negative Hepatitis B surface antibody Positive Hepatitis C virus antibody Negative Hepatitis A virus antibody (IgG) Positive Antinuclear antibody Negative Anti-smooth muscle antibody Negative Ultrasonography of the liver is normal. Which of the following is the most likely diagnosis? A Acute fatty liver of pregnancy B Acute hepatitis A infection C Cholestasis of pregnancy D HELLP syndrome

C This patient has cholestasis of pregnancy, the most common and most benign pregnancy-related liver disorder. It often presents in the second or third trimester of pregnancy and is associated with pruritus and mild elevation of the bilirubin level with or without a mild elevation of the aminotransferase levels. The total bilirubin rarely exceeds 10 mg/dL (171 µmol/L). The absence of other potentially dangerous pregnancy-related liver diseases needs to be confirmed. Treatment consists of ursodeoxycholic acid, which usually controls the symptoms. Although considered benign, this disorder can be associated with increased risk of fetal distress and preterm delivery; therefore, management should involve an experienced obstetrician. Acute fatty liver of pregnancy usually occurs during the third trimester of pregnancy; it can be associated with high aminotransferase levels, increased bilirubin level, and evidence of fat on liver ultrasonography. Fetal mortality is high in the absence of delivery. The patient does not have evidence of acute hepatitis A, which often presents with very high aminotransferase levels (usually greater than 500 U/L) in a patient with likely exposure to the virus. The presence of the positive total hepatitis A virus antibody signifies a resolved infection. The HELLP syndrome is incorrect because the diagnostic criteria of this disorder—hemolytic anemia, significantly elevated aminotransferase levels, and thrombocytopenia—are not present. HELLP, a serious pregnancy-related disease also occurs during the third trimester and up to 48 hours postpartum. It is part of the spectrum of eclampsia or pre-eclampsia and should be considered in a pregnant patient who presents with hypertension, proteinuria, and acute liver disease. There is no treatment; therefore, prompt recognition along with urgent delivery of the fetus is the best therapy and often curative. If the disorder is not recognized and treated, the morbidity and the mortality rate of fetus and mother are significant.

Gastro 95 A 29-year-old woman is evaluated for a 6-week history of bright red blood per rectum along with mucoid stool and tenesmus. She has two or three bowel movements a day. The patient is otherwise healthy and takes no medications; her parents and two siblings are alive and well. On physical examination, the temperature is 36.8 °C (98.0 °F), the blood pressure is 110/76 mm Hg, the pulse rate is 76/min, and the respiration rate is 12/min. There is no scleral icterus. The abdomen is soft and not tender or distended with normal bowel sounds. Rectal examination reveals red blood in the rectal vault with normal rectal tone and no palpable masses. Laboratory studies, including serum C-reactive protein and erythrocyte sedimentation rate, are normal. Ileocolonoscopy shows mild to moderate erythema from the anal verge to 10 cm; the rest of the colon and terminal ileum are normal. Biopsy specimens show mildly active chronic colitis in the rectum. Which of the following is the most appropriate therapy for this patient? A Azathioprine B Infliximab C Mesalamine (topical) D Metronidazole E Prednisone

C Topical therapy with corticosteroids or mesalamine is appropriate for distal ulcerative colitis. Ulcerative colitis can be classified by both extent and severity of disease, and treatment is guided by both factors. This patient has mild to moderate ulcerative proctitis, with disease limited to the distal rectum. Therefore, the best treatment would be local therapy with suppositories or enemas. Options include cortisone foam and mesalamine or corticosteroid suppositories for proctitis and hydrocortisone or mesalamine enemas for left-sided colitis. Topical mesalamine is more effective than topical corticosteroids, and almost half of the dose of topical corticosteroids may be absorbed systemically, leading to adverse long-term side effects. Maintenance of remission can be achieved with topical mesalamine. Oral 5-aminosalicylates, including sulfasalazine, mesalamine, balsalazide, and olsalazine, are appropriate for distal disease that does not respond to topical therapy or for mild to moderate pancolitis. Oral prednisone is used when symptoms do not respond to 5-aminosalicylates. Because prednisone and other corticosteroids have many acute and chronic toxic effects that are dose- and duration-dependent, the lowest effective dose should be given for the shortest time. Azathioprine or 6-mercaptopurine (6-MP) may be used for patients who have incomplete disease remission while on corticosteroids. However, because both agents have delayed onset of action, concomitant administration of either azathioprine or 6-MP together with a 3- to 4-month course of prednisone is often necessary. The patient's mild and limited disease makes oral immunosuppressive therapy such as prednisone or azathioprine inappropriate. Controlled trials have not demonstrated a consistent benefit of antibiotic therapy in the treatment of ulcerative colitis. There is probably a role for antibiotics in the treatment of fulminant colitis to treat or prevent a life-threatening infection but not in this patient with mild to moderate disease.

Gastro 49 A 73-year-old woman who has been hospitalized for 3 weeks for urosepsis requiring parenteral nutrition is evaluated for 2 days of pain in the right upper quadrant of the abdomen and new-onset fever. The patient required mechanical ventilation but was recently weaned from the ventilator. Her medications include intravenous imipenem, furosemide, and omeprazole. On physical examination, the temperature is 38.7 °C (101.7 °F), the blood pressure is 140/80 mm Hg, the pulse rate is 98/min, and the respiration rate is 18/min; the BMI is 35. The abdomen is diffusely tender with increased discomfort in the right upper quadrant with guarding and rebound. Bowel sounds are hypoactive and there is no ascites. Laboratory studies: Leukocyte count 17,000/µL (17 × 109/L) with 15% band forms INR 1.3 Bilirubin (total) 2.4 mg/dL (41.0 µmol/L) Bilirubin (direct) 1.7 mg/dL (29.0 µmol/L) Aspartate aminotransferase 44 U/L Alanine aminotransferase 35 U/L Alkaline phosphatase 150 U/L Albumin 3.8 g/dL (38 g/L) Urinalysis Negative Ultrasonography of the abdomen shows a distended gallbladder with an edematous wall and pericholecystic fluid, pancreatic edema unchanged from previous imaging, and increased liver echotexture consistent with fatty infiltration; there is no cholelithiasis or sludge or dilatation of biliary duct. Which of the following is the most appropriate next step in the management of this patient? A Addition of an antifungal agent B Endoscopic retrograde cholangiopancreatography C Observation on current therapy D Percutaneous cholecystostomy

D Definitive therapy for acalculous cholecystitis is cholecystectomy; percutaneous cholecystostomy may be necessary in critically ill patients who cannot undergo cholecystectomy. This patient has evidence of cholecystitis on imaging, with gallbladder wall edema and the presence of pericholecystic fluid. However, she does not have any evidence of either cholelithiasis or choledocholithiasis. Therefore, the patient has acalculous cholecystitis. The clinical presentation of acalculous cholecystitis is variable. In critically ill patients, the only clues may be fever, leukocytosis, or nonspecific abdominal pain. In other patients, it may present similarly to calculous cholecystitis with right upper quadrant pain, fever, and right upper quadrant tenderness and a positive Murphy sign. Patients at risk for acalculous cholecystitis are those who are ill in the hospital with comorbidities, requiring long-term parenteral nutrition. These patients may be too ill for cholecystectomy and, therefore, require percutaneous drainage of the gallbladder until they are stable enough to undergo definitive therapy with surgery. Without intervention acalculous cholecystitis has a mortality rate of 10% to 50%, and, therefore, this patient cannot continue to be observed with ongoing antibiotic therapy. The patient does not have any evidence of choledocholithiasis as the cause of her pain or cholecystitis, and, therefore, endoscopic retrograde cholangiopancreatography will not be therapeutic and places the patient at increased risk of complications or worsening pancreatitis. Although antifungal therapy should be considered in patients with necrotizing pancreatitis whose condition deteriorates clinically on antibiotic therapy, this patient's pancreatitis is stable and not the cause of the current change in her status.

Gastro 25 A 68-year-old man with a history of alcoholism is evaluated in the emergency department for a 7-month history of diarrhea during which he has noted an increased volume of stool and decreased consistency. He has had intermittent abdominal pain but not severe enough to prevent him from eating or drinking. He is not taking any medications. On physical examination, he is afebrile; the blood pressure is 108/72 mm Hg, the pulse rate is 80/min, and the respiration rate is 16/min. The abdomen is soft with mild periumbilical tenderness but no distention. Laboratory studies: Aspartate aminotransferase 155 U/L Alanine aminotransferase 88 U/L Alkaline phosphatase 96 U/L Bilirubin (total) 1.1 mg/dL (18.8 µmol/L) Amylase 65 U/L Lipase 70 U/L CT scan of the abdomen shows calcifications but no mass. There is fat in the stool. Which of the following is the most appropriate management for this patient? A Fiber B Cholestyramine C Loperamide D Pancreatic enzymes

D The treatment of pancreatic insufficiency is enzyme replacement therapy. The patient has chronic pancreatitis, which results from alcohol use in approximately 70% of adult cases, and most such patients have consumed more than 150 g/d of alcohol over 6 to 12 years. He also has concomitant alcoholic liver disease. Although the use of pancreatic enzyme supplementation for pain relief has had mixed results in patients with alcohol-related chronic pancreatitis, its use for control of the steatorrhea that results from pancreatic insufficiency is well established. This patient's diarrhea will likely resolve by this intervention. Fiber will bulk-up the stool but will not address the malabsorptive cause of diarrhea. Cholestyramine will bind bile salts, which is not the patient's primary problem. Loperamide will slow down gut motility but he will continue to have fat malabsorption without pancreatic enzyme replacement.

Gastro 21 A 55-year-old man is evaluated for a 4-month history of frequent and urgent defecation with loose and bloody stool, mild abdominal cramping, and fatigue. He has up to eight bowel movements a day and often wakes at night with symptoms; before this episode he had one bowel movement a day with well-formed stool. He does not have fever, nausea, or vomiting, but he has lost 3 kg (7 lb). He has mild joint pain in his knees and ankles that also began 4 months ago, which is worse in the morning and resolves somewhat during the day. The patient is a former cigarette smoker but quit smoking 2 years ago. His medical history includes hypertension, and his only medication is hydrochlorothiazide. On physical examination, vital signs are normal. There is mild lower abdominal tenderness without rebound or guarding; there are no palpable abdominal masses. Examination of the rectum shows gross blood. Laboratory studies reveal hemoglobin 12.3 g/dL (123 g/L) with a mean corpuscular volume of 76 fL. Fecal leukocytes are present, but stool analysis is negative for infection. Colonoscopy shows continuous erythema, friability (Zerbrechlichkeit), and loss of vascular pattern from the rectum to the splenic flexure; the rest of the colon and terminal ileum is normal. Histology shows cryptitis, crypt abscesses, and crypt architecture distortion. Which of the following is the most likely diagnosis? A Crohn colitis B Infectious colitis C Ischemic colitis D Microscopic colitis E Ulcerative colitis

E Chronic histologic changes such as crypt architectural distortion are key distinguishing features of ulcerative colitis as compared to other causes of colitis, such as infection. This patient has mild to moderate left-sided ulcerative colitis based on his clinical presentation and endoscopic and histologic findings. His ex-smoking status, microcytic anemia, and the presence of arthritis, which is the most common extraintestinal manifestation of inflammatory bowel disease, further support the diagnosis. While many colitides can have overlapping clinical, endoscopic, and even histologic features, there are important differences to consider. Microscopic colitis presents with nonbloody diarrhea, and colonoscopy shows normal mucosa macroscopically and histology shows either increased intraepithelial lymphocytes (lymphocytic colitis) or an increased submucosal collagen layer (collagenous colitis). Bleeding is less often a feature of Crohn colitis, and endoscopic inflammatory changes are patchy and generally spare the rectum; histologic features, however, may be indistinguishable from those of ulcerative colitis. Infectious colitis usually presents with more acute symptoms, and chronic changes such as crypt architecture distortion are absent. Ischemic colitis also generally has a more acute course and spares the rectum because of the dual blood supply to this region.

Gastro 72 A 52-year-old man is evaluated for a 5-month history of three to four loose, bloody stools a day with mild urgency, abdominal cramping, and fatigue. He has not lost weight during this episode. The patient is a former cigarette smoker but quit smoking 18 months ago. He is otherwise healthy. On physical examination, vital signs are normal. There is mild lower abdominal tenderness without rebound or guarding; there are no palpable abdominal masses. Examination of the rectum shows gross blood. Laboratory studies reveal hemoglobin 12.9 g/dL (129 g/L) with a mean corpuscular volume of 78 fL. Stool culture is negative. Colonoscopy shows continuous mild erythema and loss of vascular pattern from the rectum to the proximal sigmoid colon; the rest of the colon and terminal ileum are normal. Biopsy specimens from the abnormal mucosa show cryptitis, crypt abscesses, and distortion of crypt architecture. Which of the following would be the most appropriate therapy for this patient? A Azathioprine B Ciprofloxacin C Metronidazole D Infliximab E Mesalamine

E First-line therapy for induction and maintenance of remission in mild to moderate ulcerative colitis is mesalamine or another 5-aminosalicylate agent. This patient has mild left-sided ulcerative colitis based on his clinical presentation and laboratory, endoscopic, and histologic findings. His ex-smoking status further supports this diagnosis. Mesalamine, a 5-aminosalicylate (5-ASA), is an effective agent to induce and maintain remission in this setting, and in mild cases, may be the only medication necessary. Its safety profile is excellent. Azathioprine, an immunomodulator, may require 2 to 3 months to have a therapeutic effect and is generally reserved for patients who require corticosteroids for a short period of time and then transition to this medication. Antibiotics, including both metronidazole and ciprofloxacin, have not been shown to be effective in ulcerative colitis. The role of infliximab, a chimeric antibody against tumor necrosis factor-α, in ulcerative colitis is evolving; in patients with severe disease or who do not respond to corticosteroid therapy for remission, infliximab may be effective, but it would not be an appropriate first-line medication in mild ulcerative colitis.

Gastro 45 A 25-year-old man is evaluated after being turned down as a blood donor because of abnormal liver chemistry tests. The patient is healthy, takes no medications, does not smoke, and drinks alcohol socially. His parents and siblings are alive and healthy; his maternal grandfather developed type 2 diabetes mellitus at age 75 years. The review of systems is normal. On physical examination, vital signs and BMI are normal. Laboratory studies: Hemoglobin 11.9 g/dL (119 g/L) Mean corpuscular volume 76 fL Cholesterol (total) 155 mg/dL (4.01 mmol/L) LDL cholesterol 85 mg/dL (2.2 mmol/L) HDL cholesterol 33 mg/dL (0.85 mmol/L) Bilirubin (total) 0.5 mg/dL (8.55 µmol/L) Aspartate aminotransferase 25 U/L Alanine aminotransferase 58 U/L Alkaline phosphatase 110 U/L Serologic tests for hepatitis virus infection are normal. Which of the following is the most appropriate diagnostic test for this patient? A Anti-tissue transglutaminase antibody B α1-Antitrypsin concentration C Blood alcohol level D Liver biopsy

A Measurement of serum anti-tissue transglutaminase antibodies has a sensitivity and specificity of approximately 90% for celiac disease. Celiac disease is a small-bowel disorder characterized by mucosal inflammation, villous atrophy, and crypt hyperplasia, which occur on exposure to gluten. The disease is rather common, affecting nearly 1% (1/133 persons) of the population. Although many affected patients have diarrhea and steatorrhea, as well as bloating, abdominal pain, and malabsorption of vitamins and minerals, other patients are asymptomatic at diagnosis. Such patients may be found to have anemia or osteoporosis as part of routine health maintenance testing. Still others present with neurologic symptoms, dermatitis herpetiformis, or elevated concentrations of liver enzymes. The elevated alanine aminotransferase in this patient should normalize with a gluten-free diet if he has celiac disease. The patient's elevated alkaline phosphatase concentration is likely the result of increased bone turnover as a result of vitamin D and calcium malabsorption. Celiac disease is one of the most common causes of osteoporosis in men. Measurement of serum anti-tissue transglutaminase antibodies has a sensitivity and specificity of approximately 90% for celiac disease. α1-Antitrypsin deficiency affects approximately 1 in 1600 persons, and although measurement of α1-antitrypsin is an appropriate test in the work-up of abnormal liver function tests, it is not the next best test because of the low prevalence of the disease compared to celiac disease, and it cannot explain the patient's anemia. The patient admits to social use of alcohol, and this is to be strictly avoided in the setting of liver disease. Unless the patient was actively drinking alcohol around the time of the testing, measuring the blood alcohol would not be helpful. Furthermore, in alcoholic liver disease, the aspartate aminotransferase concentration is usually greater than the alanine aminotransferase. Liver biopsy would be an extreme measure to take as a result of a mildly abnormal set of liver chemistry tests and is not indicated as an initial step.

Endo 7 A 71-year-old woman is evaluated for a 2-day history of progressive dyspnea on exertion. She has also had two episodes of black, tarry stool in the past week. She has not had fever, chills, cough, or abdominal pain or bright red rectal bleeding. The patient has a history of osteoarthritis for which she takes ibuprofen, 600 mg twice daily. On physical examination, the temperature is 37.0 °C (98.6 °F), the blood pressure is 136/84 mm Hg, the pulse rate is 78/min, and the respiration rate is 12/min; the BMI is 24. Cardiac examination shows a grade 2/6 early systolic murmur at the base and regular rhythm with normal heart sounds. The lungs are clear, and there is no peripheral edema. Rectal examination reveals brown stool that is positive for occult blood. Laboratory studies reveal hemoglobin of 9.8 g/dL (98 g/L) with a mean corpuscular volume of 80 fL; serum biochemistry tests, including liver chemistry tests, and prothrombin time, activated partial thromboplastin time, and INR are normal. Chest radiography and echocardiography are normal; esophagogastroduodenoscopy, colonoscopy, and push enteroscopy are normal. Small-bowel capsule endoscopy shows a nonbleeding white ulcer in the mid-ileum. Which of the following is the most appropriate next step in the management of this patient? A Discontinue ibuprofen therapy B Double-balloon enteroscopy C Estrogen/progesterone therapy D Mesenteric angiography E Octreotide therapy

A NSAID-induced injury to the bowel is a relatively common cause of small-bowel ulceration and may present with obscure gastrointestinal bleeding. Ulceration of the ileum may occur as a result of infection, inflammation, or neoplasia. Medication-induced injury of the small bowel is a well-recognized complication of NSAID therapy; and discontinuing the medication results in ulcer healing and cessation of bleeding. Mild NSAID-induced gastrointestinal side effects include abdominal pain, nausea, constipation, diarrhea, and dyspepsia; major complications include ulcers, bleeding, perforation, and the attendant morbidity and morality. After NSAID therapy is discontinued, no further investigation is warranted if the patient's bleeding ceases. Alternatively, ongoing blood loss or symptoms referable to the ulcer warrants further evaluation which may include directed biopsy and/or resection. The information from the capsule study regarding location of the bleeding site would direct the enteroscopy, which could be performed using the double-balloon technique to reach the site of injury. Although estrogen/progesterone therapy and octreotide therapy may be useful for management of bleeding from enteric angiodysplasia, these therapies would not aid in ulcer healing. Mesenteric angiography would not identify an ulcer that is not actively bleeding and therefore would not be indicated in this patient.

Gastro 87 A 68-year-old woman is evaluated after surgical resection for colon cancer. Three months ago, the patient presented with abdominal pain and rectal bleeding; colonoscopy revealed a mass in the mid-sigmoid colon, obstructing the lumen and preventing passage of the colonoscope. A left hemicolectomy was done, and she was found to have stage IIA disease. She has recovered well from surgery. Her medical history also includes coronary artery disease and hypercholesterolemia, and her medications are metoprolol, atorvastatin, and aspirin. On physical examination, vital signs are normal; BMI is 23.5. She has a well-healed abdominal scar; the abdomen is not tender and there is no organomegaly. The chest is clear. Laboratory results, including complete blood count, are normal; she had a normal serum carcinoembryonic antigen (CEA) level before surgery. Preoperative CT scan of the abdomen showed no lymphadenopathy or liver masses. In addition to CEA testing in 3 months, which of the following is the most appropriate management for this patient? A Colonoscopy now and again in 1 year B Colonoscopy in 3 years C CT scan in 3 and 12 months D Positron emission tomography scan now E Proctoscopy now

A Surveillance with colonoscopy, serial measurement of serum carcinoembryonic antigen, and CT scan is indicated in patients who have undergone curative resection for colon cancer. Patients diagnosed with colon cancer require a complete survey of the colon. The complete survey is usually done during the initial colonoscopy; however when a complete colonoscopy cannot be done preoperatively, it should be done postoperatively to determine whether synchronous lesions are present. Surveillance colonoscopy is then performed at 1, 3, and 5 years. Serum carcinoembryonic antigen (CEA) is usually measured every 3 months postoperatively, and CT scan is obtained at 1 year. Three meta-analyses showed increased survival with intensive follow-up after surgical resection for colorectal cancer. Recommendations from the American Society of Clinical Oncology suggest history and physical exams every 3 to 6 months for 3 years and then every 6 months for the next 2 years, CEA testing every 3 months for at least 3 years after diagnosis, and CT scans yearly for 3 years. FDG-PET may provide information regarding distant spread of disease, but it is not currently recommended as part of a routine surveillance program. Proctosigmoidoscopy surveillance is indicated for rectal cancers but not for more proximal lesions.

Gastro 15 A 56-year-old woman is evaluated for a 3-year history of progressive dysphagia for solids and liquids; she has had a 6.8-kg (15-lb) weight loss during this time. The dysphagia was initially intermittent, but recently swallowing almost all food or drink causes a feeling of chest tightness and discomfort with increasingly frequent regurgitation of undigested food. The dysphagia is sometimes alleviated by standing upright. Her medical history is significant only for hypertension, and her medications include lisinopril and a multivitamin. On physical examination, the patient appears uncomfortable and restless; she is thin but does not have thenar wasting. She is afebrile; the blood pressure is 142/92 mm Hg, the pulse rate is 96/min, and the respiration rate is 22/min. The BMI is 23. Barium esophagography shows a dilated esophagus with an air/fluid level and tapered narrowing of the distal esophagus. Esophagogastroduodenoscopy shows a dilated esophagus with retained food and a tight lower esophageal sphincter, which allowed passage of the endoscope. Which of the following is the most likely diagnosis? A Achalasia B Diffuse esophageal spasm C Peptic stricture D Scleroderma esophagus

A Achalasia is a primary motility disorder of the esophagus and requires manometric diagnosis. The patient's history is typical for achalasia, an uncommon but important primary motility disorder of the esophagus. The barium study and endoscopic appearance described are typical for achalasia, but the diagnosis is confirmed manometrically with esophageal motility studies. The manometric diagnosis of achalasia usually includes an elevated lower esophageal sphincter pressure, failure of the lower esophageal sphincter to relax with swallowing, and diminished or absent peristalsis of the esophageal body. Diffuse esophageal spasm typically presents with noncardiac chest pain. The diagnosis of diffuse esophageal spasm is made manometrically by the finding of more than 20% of swallows having simultaneous contractions in the distal esophagus. Peptic stricture would present with dysphagia, but would typically show a longer, non-tapered stricture on barium esophagography. Furthermore, peptic strictures seldom present with megaesophagus, as seen in this patient. Scleroderma esophagus leads to loss of esophageal motility and often severe reflux or distal esophageal strictures, not a dilated esophagus. Treatment of achalasia is usually pneumatic dilatation of the esophagus or surgical myomectomy, the latter of which can be done laparoscopically. Pneumatic dilatation, even in experienced hands, is associated with a 5% to 10% risk of esophageal perforation. Botulinum toxin injection can afford relief of achalasia in patients who are not considered candidates for endoscopic or surgical interventions.

Gastro 30 A 74-year-old man is evaluated in the hospital for severe diffuse abdominal pain. He was hospitalized 5 days ago for chest pain and was found to be in rapid atrial fibrillation and a myocardial infarction was diagnosed. He underwent cardiac catheterization and double stent placement after which he has had intermittent hypotension and has remained in atrial fibrillation with a controlled ventricular rate. At the bedside the patient is sweating, nauseated, and holding his abdomen. He cannot respond to questions. His wife says that he has never had any gastrointestinal problems, but that he has not had a bowel movement since he entered the hospital. The patient has chronic atrial fibrillation but he discontinued his anticoagulation therapy 6 months ago; he also has hyperlipidemia. His medications are heparin, metoprolol, simvastatin, clopidogrel, and aspirin. On physical examination, the temperature is 38.0 °C (99.5 °F), the blood pressure is 102/60 mm Hg, the pulse rate is 94/min, and the respiration rate is 25/min. There is mild, diffuse abdominal tenderness to palpation without rebound or guarding; there are no palpable abdominal masses. Laboratory studies reveal only a leukocyte count of 13,000/µL (13 × 109/L). Radiograph of abdomen shows no evidence of perforation or obstruction. Which of the following would be the most appropriate management for this patient? A CT arteriography B Colonoscopy C Intravenous famotidine D Lactulose

A CT angiography is the most sensitive diagnostic test for acute mesenteric ischemia. This patient most likely has acute mesenteric ischemia, a potentially fatal condition that typically affects elderly patients. The disorder may result from embolic or thrombotic occlusions of the splanchnic vessels or may be nonocclusive. Superior mesenteric artery embolism is the most common cause, accounting for approximately 50% of cases and usually developing from ventricular or left atrial thrombi in patients with atrial fibrillation. This patient's age, recent myocardial infarction, atrial fibrillation, history of discontinuing anticoagulation, and hypotension are all significant risk factors for this condition. His low-grade fever, hypotension, sudden onset of severe abdominal pain out-of-proportion to physical examination findings, as well as his elevated leukocyte count all support the diagnosis. CT arteriography is a sensitive test for acute mesenteric ischemia; the procedure can evaluate the mesenteric vessels and also assess for signs of bowel ischemia and rule out other potential causes of acute onset abdominal pain, such as perforation or obstruction. Surgical consultation should be done simultaneously because if the results of CT arteriography support the diagnosis of acute mesenteric ischemia, the patient may need emergent surgery. Traditional angiography is the gold standard for diagnosis of acute mesenteric ischemia and can also offer therapeutic options such as the use of vasodilators as well as balloon dilation and stent placement. Surgery is required for patients with peritoneal signs and/or evidence of bowel necrosis on imaging. Surgical interventions include peritoneal lavage, resection of necrotic and perforated bowel, thrombolectomy, patch angioplasty, endarterectomy, and bypass procedures. Successful thrombolytic therapy in stable patients without peritoneal signs has been reported. Colonoscopy would not be appropriate for this patient, because it does not evaluate the mesenteric vessels or the small intestine, only the mucosa of the colon. Intravenous famotidine therapy for possible dyspepsia would also not be appropriate in this patient with a potentially life-threatening condition. Likewise, although the patient may be somewhat constipated, giving him lactulose only is not correct.

Gastro 48 A 73-year-old man is evaluated for a 3-month history of progressive, dull (trüb), epigastric pain. The pain is constant and does not radiate. The patient has had early satiety during this time and has lost 13.5 kg (30 lb); he has had mild nausea but no vomiting. The patient has a history of peptic ulcer disease and occasional heartburn; his only medication is an over-the-counter antacid as needed. On physical examination, the patient appears cachectic; BMI is 19. There is tenderness and fullness in the epigastric region with hepatomegaly. Laboratory studies reveal hemoglobin 10.4 g/dL (104 g/L) with a mean corpuscular volume of 74 fL; bilirubin, aspartate aminotransferase, alanine aminotransferase, and alkaline phosphatase concentrations are normal. Esophagogastroduodenoscopy shows a large ulcerated mass in the gastric body with heaped-up edges; biopsy specimens show adenocarcinoma. Which of the following is the most appropriate next step in the management for this patient? A CT scan of the abdomen B Endoscopic ultrasonography C Helicobacter pylori stool antigen test D Positron emission tomography

A CT imaging is the next step after esophagogastroduoden- oscopy in the staging of newly diagnosed gastric adenocarcinoma to evaluate for metastatic disease. In a patient with a newly diagnosed gastric adenocarcinoma, the most important next step in staging would be to evaluate for metastatic disease. CT scan of the abdomen would assess for distant metastases, which would be concerning in this patient with profound weight loss and hepatomegaly. If no distant metastases are detected on initial staging, then endoscopic ultrasonography would be a helpful next test, because it can assess depth of invasion, whereas CT cannot accurately assess depth of early lesions. However, ultrasonography is less sensitive at determining metastatic disease, which is why it is not considered the initial test of choice for staging, especially considering most symptomatic patients such as this one have advanced disease at presentation. Testing for Helicobacter pylori and eradication of the organism if present are recommended in patients with early-stage gastric cancer but would not be of benefit in someone with advanced disease. Positron emission tomography (PET) scanning should be considered preoperatively to follow-up suspicious but indeterminate lesions on CT imaging, but it is not recommended as the initial step in staging.

Gastro 22 A 44-year-old man with a long history of alcohol abuse is evaluated on the sixth day of hospitalization for acute pancreatitis. On admission to the hospital, he was afebrile, the blood pressure was 150/88 mm Hg, the pulse rate was 90/min, and the respiration rate was 16/min. Abnormal findings were limited to the abdomen, which was flat and tender to palpation without peritoneal signs. Bowel sounds were normal. Plain abdominal and chest radiographs were normal. Abdominal ultrasonography revealed a diffusely enlarged, hypoechoic pancreas without evidence of gallstones or dilated common bile duct. He was treated with aggressive intravenous hydration and opioid analgesia. For the past 2 days, the patient has had repeated febrile episodes, persistent severe abdominal pain, and increasing shortness of breath. On physical examination, the temperature is 38.6 °C (101.5 °F), the blood pressure is 98/60 mm Hg, the pulse rate is 112/min, and the respiration rate is 22/min; oxygen saturation is 92% with the patient breathing oxygen 3 L/min. Breath sounds are decreased at the base of both lungs. The abdomen is distended and diffusely tender with hypoactive bowel sounds. Laboratory studies reveal leukocyte count of 19,800/µL (19.8 × 109/L), creatinine 1.4 mg/dL (106.8 µmol/L), amylase 388 U/L, and lipase 842 U/L. Which of the following is the most appropriate next step in the evaluation of this patient? A CT scan of the abdomen with intravenous contrast B Endoscopic retrograde cholangiopancreatography C Endoscopic ultrasonography D Stool chymotrypsin

A CT scan of the abdomen with intravenous contrast is the most sensitive test to diagnose pancreatic necrosis. Pancreatic necrosis should be suspected in a patient with severe acute pancreatitis whose condition is not improving or is worsening after 5 days or more of treatment. Pancreatic necrosis on CT scan can be identified as unenhanced areas of the pancreas. Neither endoscopic retrograde cholangiopancreatography nor endoscopic ultrasonography can detect the presence of pancreatic necrosis in the setting of acute pancreatitis. Stool chymotrypsin can be measured when chronic pancreatitis is suspected to help evaluate for decreased pancreatic function. Pancreatic necrosis is the most important predictor of poor outcome in acute pancreatitis. Patients who develop pancreatic necrosis should be given antibiotic prophylaxis, usually with imipenem. The necrosis should be sampled for the presence of infection, and if infection is present, surgical débridement is recommended.

Gastro 80 A 47-year-old woman is evaluated in the emergency department for 3 days of abdominal pain and distention; she has not had a bowel movement during that time. The patient has early satiety and decreased appetite; she has had nausea and vomited once. She has had similar symptoms previously. The patient has a history of systemic sclerosis. Her medications include nifedipine for Raynaud phenomenon and omeprazole for gastroesophageal reflux disease. On physical examination, pulse rate is 92/min; other vital signs are normal. The abdomen is distended with hypoactive bowel sounds and tympany. There is no succussion splash. There are no abdominal masses or organomegaly. Laboratory studies reveal hemoglobin 12.1 g/dL (121 g/L); serum thyroid-stimulating hormone, amylase, and liver chemistry tests are normal. Plain radiograph of the abdomen shows dilated loops of small bowel. After bowel decompression with a nasogastric tube and bowel rest, upper and lower gastrointestinal radiographic series shows no obstructive lesions. Which of the following is the most likely diagnosis? A Chronic intestinal pseudo-obstruction B Colonic inertia C Gastroparesis D Small-bowel obstruction

A Chronic intestinal pseudo-obstruction is characterized by chronic or recurrent episodes of abdominal pain, distention, nausea, vomiting and obstipation; the disorder may be due to myopathic causes (such as scleroderma) or neuropathic causes (such as amyloidosis, diabetes mellitus, and paraneoplastic syndromes). This patient has chronic intestinal pseudo-obstruction (CIPO), which is characterized by chronic or recurrent episodes of abdominal pain, distention, nausea, vomiting, and obstipation; also, in this patient abdominal imaging showing distended (aufgebläht) loops of small bowel supports the diagnosis. Systemic sclerosis, a connective tissue disorder, has been associated with CIPO. Other causes include neuropathic disorders such as amyloidosis, diabetes mellitus, and paraneoplastic syndromes. A hereditary cause is recognized as well. Gastrointestinal manometry may be used to confirm the diagnosis, but this test is available only at academic institutions with motility centers; full-thickness small-bowel biopsy can demonstrate neuronal loss and degenerative features as well as inflammatory enteric neuropathies, histologic features that may be seen in CIPO. Treatment of CIPO is generally supportive, with initial decompression of bowel via nasogastric tube and bowel rest. Medications that decrease intestinal motility (for example, narcotics, calcium channel blockers) should be reduced to the minimum necessary. In this patient's case, nitroglycerin paste might be substituted for nifedipine to manage the Raynaud phenomenon. An oral diet should be adhered to as tolerated; if gastroparesis coexists, medications that increase gastric motility such as metoclopramide may be used. Low-dose octreotide may improve motor activity of the small bowel, and antibiotics may be used when concomitant bacterial overgrowth is present. Laxatives may be used in cases with associated constipation. Colonic inertia often occurs concomitantly with CIPO, but in this patient, dilated loops of small bowel, rather than colon, are present. Although gastroparesis can also present with similar symptoms and/or coexist with CIPO, it is generally not an acute presentation, nor would patients stop having bowel movements or have distended loops of small bowel on radiography, although gastric distention may occur. Mechanical obstruction needs to be excluded as signs and symptoms are often very similar, and plain abdominal films can appear similar in these two diagnoses. In this patient, imaging studies did not show any obstructive lesion or mass.

Gastro 77 A 62-year-old woman with a 10-year history of gastroesophageal reflux symptoms undergoes screening esophagogastroduodenoscopy. Salmon-colored mucosa is seen in the distal 7 cm of esophagus. A biopsy of the abnormal esophageal mucosa shows intestinal metaplasia with goblet cells. The patient is asymptomatic, and her only medication is omeprazole, 40 mg/d. Which of the following is the most appropriate management for this patient? A Endoscopic/histologic surveillance B No further endoscopic evaluation C Surgical esophagectomy D Test and treat for Helicobacter pylori

A Endoscopic surveillance is recommended for patients with Barrett esophagus. Adenocarcinoma of the esophagus is one of the most rapidly increasing cancers in the United States. Because this malignancy often arises from metaplastic esophageal mucosa (Barrett esophagus), endoscopic surveillance is recommended for patients with Barrett esophagus. Although the details of when to begin surveillance and the frequency are not clear, the confirmed finding of high-grade dysplasia requires surgical removal or endoscopic ablation of the abnormal mucosa. In this case, long-standing reflux led to endoscopic evaluation. The "salmon-colored" columnar mucosa typical of Barrett esophagus was noted in the distal esophagus; metaplastic changes were confirmed by biopsy. In the absence of dysplasia, surgical esophagectomy is not indicated. Helicobacter pylori infection has not been implicated in the pathogenesis of Barrett esophagus. In fact, epidemiologic evidence points to a potential protective effect of H. pylori regarding development of Barrett esophagus.

Gastro 11 A 34-year-old woman is evaluated for continued severe mid-epigastric pain that radiates to the back, nausea, and vomiting 5 days after being hospitalized for acute alcohol-related pancreatitis. She has not been able eat or drink and has not had a bowel movement since being admitted. On physical examination, the temperature is 38.2 °C (100.8 °F), the blood pressure is 132/84 mm Hg, the pulse rate is 101/min, and the respiration rate is 20/min. There is no scleral icterus or jaundice. The abdomen is distended and diffusely tender with hypoactive bowel sounds. Laboratory studies: Leukocyte count 15,400/µL (15.4 × 109/L) Aspartate aminotransferase 189 U/L Alanine aminotransferase 151 U/L Bilirubin (total) 1.1 mg/dL (18.8 µmol/L) Amylase 388 U/L Lipase 924 U/L CT scan of the abdomen shows a diffusely edematous pancreas with multiple peripancreatic fluid collections, and no evidence of pancreatic necrosis. Which of the following is the most appropriate next step in the management of this patient? A Enteral nutrition by nasojejunal feeding tube B Intravenous imipenem C Pancreatic débridement D Parenteral nutrition

A Enteral feeding is the preferred route of providing nutrition in patients with severe acute pancreatitis. This patient has moderate to severe acute pancreatitis and after 5 days remains febrile, continues to be in pain, and cannot take in any oral nutrition. The patient will likely have an extended period before being able to take in oral nutrition. Two routes are available for providing nutrition in patients with severe acute pancreatitis: enteral nutrition and parenteral nutrition. Enteral nutrition is provided through a feeding tube ideally placed past the ligament of Treitz so as not to stimulate the pancreas. Parenteral nutrition is provided through a large peripheral or central intravenous line. Enteral nutrition is preferred over parenteral nutrition because of its lower complication rate, especially a lower infection rate. A meta-analysis of six studies with 263 participants compared enteral nutrition with total parenteral nutrition. Enteral nutrition was associated with a significantly lower incidence of infections, reduced surgical interventions to control complications of pancreatitis, and a reduced length of hospital stay. In another randomized, controlled trial, enteral nutrition showed a trend towards faster attenuation of inflammation, with fewer septic complications, and also was a dominant therapy in terms of cost-effectiveness. Imipenem therapy is only helpful in acute pancreatitis when there is evidence of pancreatic necrosis. Pancreatic necrosis is diagnosed by a contrast-enhanced CT scan that shows nonenhancing pancreatic tissue. In patients with noninfected pancreatic necrosis, antibiotics may decrease the incidence of sepsis, systemic complications (for example, respiratory failure), and local complications (for example, infected pancreatic necrosis or pancreatic abscess). Randomized, prospective trials have shown no benefit from antibiotic use in acute pancreatitis of mild to moderate severity but may lead to development of nosocomial infections with resistant pathogens. Similarly pancreatic débridement is recommended only in patients with pancreatitis and infected pancreatic necrosis.

Gastro 53 A 72-year-old man is evaluated for a 2-month history of epigastric discomfort associated in the past 6 weeks with a 4.6-kg (10 lb) weight loss; he has had 2 weeks of dark urine and light stool. The patient has no significant medical history and takes no medications. On physical examination, vital signs are normal. There is scleral icterus, visibly jaundiced skin, and mild epigastric tenderness. Laboratory studies are significant for total bilirubin of 8.2 mg/dL (140.2 µmol/L) and alkaline phosphatase of 648 U/L. CT scan of the abdomen shows fullness in the head of the pancreas and biliary dilation but no evident pancreatic mass. Which of the following is the most appropriate next step in the management of this patient? A Endoscopic ultrasonography B Measurement of CA 19-9 concentration C Surgical exploration D Ultrasonography of the abdomen E Visceral angiography

A Endoscopic ultrasonography is used commonly to diagnose pancreatic cancer, particularly in tumors of less than 2 cm. Patients with pancreatic cancer present most commonly with epigastric pain, weight loss, and in 70% of cases in which the tumor is in the head of the pancreas, with obstructive jaundice. Diagnosis is most frequently made by CT scan of the abdomen. However, smaller tumors may be missed by CT scan. Endoscopic ultrasonography is more sensitive than CT for tumors 2 cm or less. Endoscopic ultrasonography can also provide nodal and vascular staging and if desired can provide tissue through fine-needle aspiration. Angiography is no longer used for the diagnosis or staging of pancreatic cancer. Surgical exploration in suspected pancreatic cancer would be performed as an attempt at curative resection and would not be performed until the diagnosis of a pancreatic mass is established and the tumor is properly staged. Diagnostic procedures to confirm candidacy for exploratory laparotomy in patients with potentially resectable pancreatic cancer include diagnostic imaging with helical CT scans, endoscopic ultrasonography, and diagnostic laparoscopy. Although serum concentrations of CA 19-9, a Lewis blood group-related mucin, are frequently elevated in patients with this disease, this tumor marker's role has not been proved in the diagnosis or management of pancreatic cancer.

Gastro 65 A 34-year-old man is evaluated as a new patient. He is asymptomatic and has no significant medical history, but he is concerned about his risk for colon cancer. His 59-year-old mother had two adenomatous polyps detected at the age of 52 years. She has had surveillance colonoscopy every 3 years since then and has had two additional polyps detected. His father has no history of colon polyps, and there is no family history of colorectal cancer. On physical examination, vital signs are normal; BMI is 21.5; the rest of the physical examination is normal. Laboratory results are all within normal limits. Which of the following is the most appropriate screening strategy for this patient? A Colonoscopy at age 40 years B Colonoscopy at age 42 years C Colonoscopy at age 50 years D Fecal occult blood testing at age 40 years E Fecal occult blood testing at age 50 years

A Patients with a family history of colon cancer or adenoma should undergo a screening protocol at age 40 years or 10 years younger than the earliest diagnosis in the affected relative and with a shortened surveillance interval compared with those at average risk of colorectal cancer. Persons with a first-degree relative with a history of colonic adenoma or cancer have a 2- to 4-fold increased risk for colorectal cancer. This risk is modified by the age of onset of neoplasia in the affected relative. The current recommendation is that anyone with a first-degree relative with colon cancer or with an adenoma diagnosed before age 60 years should initiate screening at an earlier age than the general population and follow a shortened surveillance interval. Screening with colonoscopy is initiated at age 40 years or 10 years younger than the earliest diagnosis in the affected relative, whichever comes first. The recommended surveillance modality and interval is colonoscopy every 3 to 5 years. Surveillance colonoscopy is recommended rather than other modalities because it offers a structural assessment of the colon in its entirety and provides a means to find and to remove polyps throughout the colon in this population at increased risk of polyp and cancer development. Fecal occult blood testing is a recommended screening modality in the average-risk population but not in patients at increased risk.

Gastro 17 A 57-year-old woman is evaluated for a 1-month history of increased abdominal girth (Umfang). The patient has a 15-year history of alcohol abuse, drinking a bottle of wine a day. She also has type 2 diabetes mellitus, hypertension, and hyperlipidemia, and her medications are metformin, hydrochlorothiazide, propranolol, simvastatin, and aspirin. On physical examination, the temperature is 37.1 °C (98.8 °F), the blood pressure is 90/50 mm Hg, the pulse rate is 99/min, and the respiration rate is 13/min; the BMI is 21. Examination reveals scleral icterus; bulging (ausgewölbte) flanks; a small umbilical hernia; caput medusae; spider angiomata of the face, arms, and chest; and mild asterixis. There is no abdominal tenderness. Laboratory studies: Leukocyte count 5200/µL (5.2 × 109/L) with a normal differential Platelet count 65,000/µL (65 × 109/L) INR 2.2 Bilirubin (total) 3.4 mg/dL (58.1 µmol/L) Bilirubin (direct) 1.8 mg/dL (30.8 µmol/L) Aspartate aminotransferase 120 U/L Alanine aminotransferase 65 U/L Alkaline phosphatase 196 U/L Albumin 2.7 g/dL (27 g/L) Creatinine 2.7 mg/dL (206.0 µmol/L) Urinalysis Normal Blood cultures are negative. Ultrasonography of the abdomen shows massive ascites, patent vessels, no ductal dilatation, and a shrunken liver with no masses. Which of the following is the most appropriate management for this patient? A Diagnostic paracentesis B Cefotaxime and albumin C Furosemide and spironolactone D Transjugular intrahepatic portosystemic shunt

A An ascitic fluid absolute polymorphonuclear cell count greater than 250/mL is indicative of spontaneous bacterial peritonitis. This patient should have a diagnostic paracentesis to confirm the etiology of the ascites and to rule out spontaneous bacterial peritonitis (SBP). All patients who present with ascites should have a diagnostic paracentesis to categorize the ascites as secondary to portal hypertension or to other causes. This patient should have a small amount of fluid sampled for cell count with differential. An absolute polymorphonuclear cell count greater than 250/mL is indicative of SBP. Also total protein should be measured as well as albumin to calculate the serum albumin to ascites gradient (SAAG). An SAAG greater than 1.1 is consistent with portal hypertension; an SAAG less than 1.1 can occur in patients with malignant ascites. Patients do not always have symptoms suggestive of SBP such as fever, abdominal pain, or leukocytosis; therefore, their absence should not preclude a paracentesis to rule out SBP. In addition this patient has an elevated serum creatinine, which poses the possibility of hepatorenal syndrome for which SBP is a significant risk. Therefore, it is important to determine whether she has SBP. Furosemide and spironolactone therapy is not indicated in the setting of an elevated serum creatinine and possible hepatorenal syndrome, because diuresis can precipitate worsening hemodynamic status or renal failure. Ceftriaxone and albumin are appropriate therapy for spontaneous bacterial peritonitis but are not indicated unless a diagnostic paracentesis can confirm the diagnosis. TIPS is not first-line therapy for new-onset ascites.

Gastro 71 A 68-year-old man is evaluated in the emergency department for a 6-hour history of nausea and vomiting, with some bright red emesis. For the past 2 hours he has felt lightheaded and weak. On physical examination, the temperature is 37.0 °C (98.6 °F), the blood pressure is 88/51 mm Hg, the pulse rate is 114/min, and the respiration rate is 18/min. Nasogastric aspiration shows a mixture of coffee grounds and dark blood. The abdomen is not tender, and bowel sounds are normal. Laboratory studies reveal hemoglobin of 9.4 g/dL (94 g/L); all other tests are normal. Omeprazole therapy is begun. After volume repletion with saline and transfusion of two units of packed erythrocytes, esophagogastroduodenoscopy is performed and reveals a duodenal ulcer with a small visible vessel. Which of the following is the best management option for this patient? A Endoscopic therapy B Immediate surgical intervention C Observation D Octreotide infusion E Ranitidine infusion

A An ulcer with a visible vessel as a cause of upper gastrointestinal bleeding has an approximately 50% risk of rebleeding if not treated endoscopically. Upper endoscopy should be performed at the time of an upper gastrointestinal bleed after appropriate volume resuscitation to provide a diagnosis as to the cause of bleeding, provide a prognosis, and perform endoscopic guided therapy if required. An ulcer with a visible vessel has an approximately 50% risk of rebleeding if not treated endoscopically. These ulcers can be effectively treated with injection therapy, thermal coagulation via endoscopic probes, or mechanical modalities such as endoclips. Clean-based ulcers rebleed in less than 5% of cases and do not require endoscopic therapy. Prospective trials have shown a 5% to 10% rebleeding rate for endoscopic hemostasis. In these patients, endoscopic therapy may be repeated if the patient remains hemodynamically stable and if the endoscopist thinks that the bleeding lesion is amenable to endoscopic therapy. If repeat endoscopy is unsuccessful or the bleeding vessel is inaccessible or too large, surgical consultation should be obtained. However, in this case endoscopic intervention is the first management choice for an accessible and small visible vessel. Intravenous omeprazole has been shown to reduce the risk of recurrent upper gastrointestinal bleeding in peptic ulcers after endoscopic hemostasis. Oral omeprazole also may decrease rebleeding. A meta-analysis showed that adjuvant high-dose proton pump inhibitor therapy following endoscopic hemostasis for ulcers at high risk of rebleeding reduces rebleeding, surgery, and mortality. Octreotide may have a marginal benefit by decreasing the rate of nonvariceal bleeding but is inferior to intravenous proton pump inhibitors. Ranitidine is inferior to proton pump inhibitors as an adjunct to endoscopic therapy.

Gastro 86 A 35-year-old man is evaluated in the emergency department for the acute onset of abdominal pain and nausea; he does not have diarrhea or other changes in bowel habits. The patient has a 17-year history of Crohn disease, which was managed with long-term corticosteroid therapy until he began therapy with azathioprine 4 months ago; his disease has been stable since that time until the current episode. He takes no other medications. On physical examination, the temperature is 37.0 °C (98.5 °F), the blood pressure is 108/76 mm Hg, the pulse rate is 110/min, and the respiration rate is 19/min. There is no scleral icterus. There is mild epigastric tenderness, but the rest of the examination is normal. Right upper quadrant abdominal ultrasonography is normal. Laboratory studies: Hemoglobin 17.2 g/dL (172 g/L) Leukocyte count 4500/µL (4.5 × 109/L) Aspartate aminotransferase 20 U/L Alanine aminotransferase 22 U/L Bilirubin (total) 0.9 mg/dL (15.4 µmol/L) Amylase 1500 U/L Lipase 1630 U/L Which of the following is the most likely diagnosis? A Azathioprine-induced pancreatitis B Crohn disease flare C Gallstone pancreatitis D Helicobacter pylori-associated gastritis E Peptic ulcer disease

A Azathioprine can cause pancreatitis in about 2% of patients taking the drug; the drug should be discontinued and not resumed if pancreatitis occurs. In patients with Crohn disease, not every episode of acute abdominal pain is related to the underlying inflammatory bowel disease. Patients with the disease who are refractory to or dependent on corticosteroids are a challenging problem. Since surgery for Crohn disease is associated with a high rate of recurrence, medical therapy with immunomodulators, such as 6-mercaptopurine and azathioprine, and biologic agents, such as infliximab and adalimumab, remain important therapies for the disorder. However, idiosyncratic pancreatitis may occur in up to 2% of patients treated with azathioprine, and the drug must be discontinued and not restarted in affected patients. The patient's symptoms, relatively benign abdominal examination, and the markedly increased concentrations of pancreatic enzymes make pancreatitis the most likely diagnosis. The patient does not have an increase in the frequency of bowel movements, and therefore, a Crohn disease flair is unlikely and cannot explain the elevated serum amylase and lipase concentrations. The patient's normal aminotransferase and bilirubin concentrations make gallstone pancreatitis unlikely. The patient's markedly increased amylase and lipase concentrations make peptic ulcer disease and Helicobacter pylori-associated gastritis unlikely.

Gastro 6 A 44-year-old woman is evaluated for a 6-month history of dyspepsia, regurgitation of sour fluid, and eructation. There is no associated fever, chills, weight loss, or vomiting. The condition failed to respond to a 6-week trial of omeprazole therapy. The patient's medical history includes hypertension, type 2 diabetes mellitus, and obesity (BMI 36); her medications are lisinopril, metformin, and insulin glargine. On examination, vital signs are normal; there is mild epigastric tenderness without rebound, and stool is negative for occult blood. Which of the following is the most appropriate next diagnostic step in the evaluation of this patient? A Ambulatory esophageal pH monitoring B Barium esophagography C CT scan of the chest D Esophageal manometry

A In patients with a high pretest probability of gastroesophageal reflux disease who fail to respond to a therapeutic trial of a proton pump inhibitor, ambulatory esophageal pH monitoring is most useful. This patient's clinical presentation is highly suggestive of gastroesophageal reflux disease (GERD). The typical symptoms of pyrosis, along with obesity, make the pretest probability of GERD high, although the lack of response to proton pump inhibitor therapy argues against the diagnosis. In patients with a high pretest probability of GERD who fail to respond to a therapeutic trial, ambulatory esophageal pH monitoring is most useful. Ambulatory pH monitoring employs a narrow-gauge cannula placed transnasally into the distal esophagus. Simultaneous pH measurement is made by several detectors. An endoscopically placed ambulatory monitor is becoming more commonly used. Abnormal esophageal acid exposure is determined by the percentage of time that esophageal pH is less than 4.0, esophageal acid exposure time, number of reflux episodes, mean duration and number of reflux episodes, and longest duration of reflux. The patient can record time of meals, episodes of reflux symptoms or pain, supine position, and other parameters to correlate symptoms to actual abnormal esophageal acid exposure. Barium esophagography is a useful initial test in the evaluation of oropharyngeal reflux and to determine the presence of strictures but is less useful in determining significant episodes of gastroesophageal reflux or their relationship to symptoms. CT scan of the chest is performed in cases of dysphagia to investigate potential masses in the region of the gastric cardia. Esophageal manometry is useful in esophageal motility disorders such as diffuse esophageal spasm, achalasia, and hypotonic motility disorders. Newer techniques of esophageal function include esophageal impedance monitoring, which provides information on esophageal bolus transit as opposed to manometry's ability to detect esophageal contractile activity.

Gastro 44 A 55-year-old woman is evaluated for elevated liver chemistry tests detected on examination for life insurance. She has no symptoms of liver disease and no history of jaundice, ascites, lower extremity edema, or encephalopathy. She used recreational injection drugs between the ages of 20 and 25 years. She has no significant medical history and takes no medications. She drinks about six cans of beer a day. On physical examination, vital signs are normal. There are spider angiomata on the upper body and the presence of a nodular liver edge and splenomegaly. Laboratory studies: Platelet count 88,000/µL (88 × 109/L) INR 1.4 Bilirubin (total) 1.1 mg/dL (18.8 µmol/L) Aspartate aminotransferase 48 U/L Alanine aminotransferase 96 U/L Alkaline phosphatase 186 U/L Albumin 3.6 g/dL (36 g/L) Hepatitis B surface antigen Negative Hepatitis C virus antibody Positive HCV RNA in serum 500,000 copies/mL Ultrasonography shows coarsened hepatic echotexture; CT scan shows changes in the liver consistent with cirrhosis and splenomegaly. Which of the following is the most appropriate next step in the management for this patient? A Esophagogastroduodenoscopy B Evaluation for liver transplantation C Lamivudine D Pegylated interferon and ribavirin

A Variceal hemorrhage occurs in 25% to 40% of patients with cirrhosis; upper endoscopy is indicated in patients with newly diagnosed cirrhosis. This patient presented with asymptomatic elevation of hepatic biochemical tests; further testing demonstrated the presence of hepatitis C virus infection and signs of cirrhosis (splenomegaly, nodular liver contour, thrombocytopenia). Variceal hemorrhage occurs in 25% to 40% of patients with cirrhosis, and since mortality from variceal bleeding is high, screening for varices is a high priority task in all patients with newly discovered cirrhosis. Esophagogastroduodenoscopy is the next best management option to exclude the presence of esophageal varices. Depending on the size and appearance of the varices and the patient's Child-Pugh grade, prophylactic treatment may be indicated. Patients with hepatitis C may be candidates for treatment with pegylated interferon and ribavirin; however, in this patient, cirrhosis and its potential complications must be assessed before therapy is begun. Moreover, treatment efficacy is lower in patients with cirrhosis than in patients without cirrhosis. Liver transplantation is the treatment of choice for suitable candidates with end-stage liver disease, fulminant hepatic failure, and certain metabolic disorders. Although the patient appears to have cirrhosis, there is no ascites, jaundice, or hepatic dysfunction that is severe enough to warrant consideration of transplantation. Treatment is recommended for all patients with chronic hepatitis B who have abnormal serum alanine aminotransferase values, HBV DNA levels >105 copies/mL, and liver biopsy findings of active disease. The oral agents lamivudine, adefovir, and entecavir are safe and well tolerated and may improve liver function in patients with decompensated hepatitis B. However, there is no indication that this patient has hepatitis B and therapy with lamivudine is not indicated.

Gastro 1 A 38-year-old man is evaluated for persistent dyspepsia 2 months after a duodenal ulcer was detected and treated. He originally presented with new-onset epigastric pain, and esophagogastroduodenoscopy showed a duodenal ulcer; biopsy specimens showed the presence of Helicobacter pylori. The patient, who does not use NSAIDs and is penicillin-allergic, completed a 10-day course of therapy with omeprazole, metronidazole, and clarithromycin. At this time, urea breath testing for H. pylori shows persistent infection. In addition to a proton pump inhibitor, which of the following regimens is indicated for this patient? A Amoxicillin and levofloxacin B Bismuth subsalicylate, metronidazole, andtetracycline C Clarithromycin and amoxicillin D Clarithromycin and metronidazole E Trimethoprim-sulfamethoxazole and erythromycin

B Bismuth-based quadruple therapy should be considered in a patient in whom initial proton pump inhibitor-based triple therapy has failed to eradicate Helicobacter pylori. This patient has persistent Helicobacter pylori infection despite initial therapy with a proton pump inhibitor, clarithromycin, and metronidazole, an appropriate regimen for this patient with penicillin allergy. The most likely reason for failure of treatment in most patients is either noncompliance with therapy or antibiotic resistance; antibiotic resistance (probably to clarithromycin) is a likely cause of treatment failure in this patient. Therefore, an additional treatment regimen, one that does not contain clarithromycin, needs to be given. In the United States, bismuth-based quadruple therapy is indicated in a patient whose infection has failed to respond to proton pump inhibitor-based triple therapy. Levofloxacin-based triple therapy is also used in patients with persistent infection, but this regimen has not been validated in the United States. Regimens containing amoxicillin would not be indicated in this patient with penicillin allergy. Therapy with the same regimen that failed initially to eradicate the organism because of likely antibiotic resistance would not be appropriate. H. pylori is naturally resistant to trimethoprim, and the regimen of trimethroprim-sulfamethoxazole and erythromycin is not an approved regimen for eradication of the organism.

Gastro 26 A 74-year-old woman is evaluated for 3 years of progressive dysphagia, first for solid foods and now for both solid foods and liquids; she has had frequent episodes of regurgitation of undigested food and has lost 6.8 kg (15 lb) during the past 6 months. Her medical history includes stenting of the left anterior descending coronary artery 1 year ago after which she has had symptomatic residual distal stenosis. She had a cerebrovascular accident 2 years ago and still has mild residual right hemiparesis. Her medications include metoprolol, clopidogrel, enalapril, aspirin, and hydrochlorothiazide. On physical examination, the patient is thin (BMI 20) and appears ill, although not in distress. Vital signs are normal. Chest radiograph shows a dilated esophagus with an air/fluid level and changes of chronic aspiration in the right lung base. Barium esophagography shows "bird beak" narrowing of the distal esophagus and mega-esophagus with retained fluid in the esophageal body. Esophageal manometry shows aperistalsis of the esophageal body and incomplete lower esophageal sphincter relaxation with swallowing. On esophagogastroduodenoscopy, the endoscope passes through the lower esophageal sphincter without resistance; there are no masses in the esophagus or the gastric cardia. Which of the following is the most appropriate therapy for this patient? A Anticholinergic therapy B Botulinum toxin injection C Laparoscopic myotomy D Pneumatic dilatation

B Botulinum toxin injection can afford relief of achalasia in patients who because of age or comorbidities are not candidates for endoscopic or surgical intervention. This patient's history is typical for achalasia, an uncommon but important primary motility disorder of the esophagus. Although endoscopic evaluation is required to investigate the possibility of a mass lesion leading to partial esophageal obstruction, the diagnosis of achalasia is made manometrically using esophageal motility studies. The manometric diagnosis usually includes an elevated lower esophageal sphincter resting pressure, failure of the lower esophageal sphincter to relax with swallowing, and diminished or absent peristalsis of the esophageal body. Radiologic findings of note in achalasia include a "bird's beak" abnormality of the distal esophagus, widening of the esophagus, and less commonly, megaesophagus with an air/fluid level. CT of the chest can be employed to differentiate achalasia from pseudoachalasia, the latter mimicking the true motility disorder but caused by a mass lesion at the distal esophagus or gastric cardia. Treatment of achalasia is usually pneumatic dilatation of the esophagus or surgical myomectomy, the latter of which can be done laparoscopically. Pneumatic dilatation, even in experienced hands, is associated with a 5% to 10% risk of esophageal perforation. Botulinum toxin injection can afford relief of achalasia in patients like this one who because of age or comorbidities are not candidates for endoscopic or surgical intervention. Botulinum toxin inhibits the release of acetylcholine from nerve endings and has been used successfully for decades to treat certain spastic disorders of skeletal muscle such as blepharospasm and torticollis. Anticholinergic therapy is not indicated for achalasia.

Gastro 96 A 45-year-old man is evaluated for a 6-month history of recurrent ascites refractory to salt restriction and diuretic therapy. He is managed with serial large-volume paracentesis. The patient has a history of hepatitis C, and cirrhosis was detected on liver biopsy 5 years ago. The patient's medications are spironolactone and furosemide. On physical examination, the patient is afebrile; the blood pressure is 85/60 mm Hg, the pulse rate is 80/min, and the respiration rate is 16/min. The BMI is 26. Examination reveals jaundice, cutaneous angiomata, cachexia, large-volume ascites, edema, and asterixis. The abdomen is not tender to palpation and a liver edge is not palpable. Laboratory studies: INR 3.5 Creatinine 1.4 mg/dL (106.8 µmol/L) Bilirubin (total) 6.5 mg/dL (111.2 µmol/L) Aspartate aminotransferase 49 U/L Alanine aminotransferase 50 U/L Albumin 2.4 g/dL (24 g/L) α-Fetoprotein 10 ng/mL (10 µg/L) The serum creatinine has been stable for the past 4 months and he continues to make approximately 800 mL of urine daily. Serum electrolytes are normal. Ultrasonography shows a nodular liver without space-occupying lesions and massive ascites. Which of the following is the most appropriate next step in the management of this patient? A Hemodialysis B Liver transplantation C Peritoneovenous shunt D Transjugular intrahepatic portosystemic shunt (TIPS)

B Diuretic-resistant ascites is a common and important indication for referral for liver transplantation. The patient has diuretic-resistant ascites, which portends a poor survival. Diuretic-resistant ascites is a common indication for liver transplantation and constitutes a reason to refer the patient for transplant evaluation. True refractory ascites is uncommon and can be managed with serial large-volume paracentesis or transjugular intrahepatic portosystemic shunt (TIPS). Randomized trials suggest that both therapeutic procedures are effective in controlling refractory ascites, although TIPS may be associated with a higher complication rate and increased risk for encephalopathy, and patients with poorly compensated liver disease may experience further decompensation after the procedure. The Practice Guidelines Committee of the American Association for the Study of Liver Diseases state that although TIPS will decrease the need for repeated large-volume paracentesis in patients with refractory ascites associated with cirrhosis, it should be used only in those patients who are intolerant of repeated large-volume paracentesis. Patients undergoing paracentesis of more than 5 L of ascitic fluid should be given intravenous albumin at the time of fluid removal in order to prevent post-paracentesis circulatory dysfunction. Peritoneovenous shunt is rarely used because of an appreciable rate of complications and the lack of evidence that it prolongs patient survival, which may be several years in patients with normal or near normal hepatic and renal function tests but less than 6 weeks in patients with the hepatorenal syndrome. Although diuretic-resistant ascites is present, the patient has a stable serum creatinine, electrolytes are normal, he is not anuric and therefore, dialysis is not indicated.

Gastro 47 A 45-year-old man is brought to the emergency department by fire rescue after his wife found him in bed lethargic and disoriented. The previous 2 days, the patient had been agitated and irritable and his speech was slurred. The patient has a history of cirrhosis complicated by esophageal variceal bleeding, ascites, and lower extremity edema, and his medications are furosemide, spironolactone, propranolol, and lactulose. He has a long history of alcohol dependence but has been sober for 1 year. On physical examination, the patient is somnolent but arousable although not responsive to commands; he is afebrile. The blood pressure is 100/78 mm Hg, the pulse rate is 65/min, and the respiration rate is 12/min; the BMI is 26. There are no focal neurologic deficits; the pupils are equal and reactive to light. There is shifting abdominal dullness and 2+ lower extremity edema. The stool is negative for occult blood. Laboratory studies: Leukocyte count 5600/µL with normal differential (5.6 × 109/L) Glucose (random) 112 mg/dL (6.21 mmol/L) Sodium 135 meq/L (135 mmol/L) Potassium 3.5 meq/L (3.5 mmol/L) Chloride 100 meq/L (100 mmol/L) Bicarbonate 28 meq/L (28 mmol/L) Bilirubin (total) 4.0 mg/dL (68.4 µmol/L) Bilirubin (direct) 2.3 mg/dL (39.3 µmol/L) Aspartate aminotransferase 78 U/L Alanine aminotransferase 45 U/L Alkaline phosphatase 167 U/L Albumin 2.7 g/dL (27 g/L) Ammonia 230 µg/dL (135 µmol/L) Blood alcohol Negative Urinalysis is negative. Dipstick is positive for 3+ leukocyte esterase and nitrites. CT scan of the head is normal. A diagnostic peritoneal fluid tap excludes spontaneous bacterial peritonitis. Empiric antibiotic therapy is started. Which of the following is the most appropriate management for this patient? A Corticosteroids B Increase lactulose therapy C Intravenous albumin D Transjugular intrahepatic portosystemic shunt (TIPS)

B First-line therapy for hepatic encephalopathy is lactulose. This patient has severe encephalopathy manifested by worsening somnolence. The patient began manifesting signs of increasing encephalopathy days prior to the admission, when his wife noticed that he was becoming more agitated and slurring his speech. Encephalopathy progresses from subtle findings, such as reversal of the sleep-wake cycle or mild mental status changes, to irritability, confusion, slurred speech, and ultimately coma if not recognized and treated. There can be multiple inciting causes of encephalopathy in patients with cirrhosis, including dehydration, infection (especially spontaneous bacterial peritonitis), diet indiscretions, gastrointestinal bleeding, and medications. This patient likely became worse with the development of the urinary tract infection. The best course of management is to treat the infection and to discontinue the diuretics and increase the lactulose to respond to the encephalopathy. The dose of lactulose should be titrated to achieve two to three soft stools per day with a pH below 6.0. Approximately 70% to 80% of patients with hepatic encephalopathy improve on lactulose therapy, and treatment is usually well tolerated. Corticosteroids have no role in the reversal of hepatic encephalopathy. Although albumin therapy is often instituted in patients hospitalized with infections, especially spontaneous bacterial peritonitis and associated dehydration, it is not a primary treatment of encephalopathy. Transjugular intrahepatic portosystemic shunt (TIPS) is also not appropriate because placement of TIPS is likely to precipitate worsening hepatic encephalopathy as more blood is bypassed through the shunt rather than processed by the liver.

Gastro 10 A 32-year-old man is evaluated in the emergency department for a 5-day history of worsening crampy abdominal pain and eight to ten loose bowel movements a day. The patient has a 5-year history of ulcerative colitis treated with azathioprine and topical mesalamine; before this episode, he had one or two bowel movements of well-formed stool a day. The patient had sinusitis recently, which resolved with antibiotic therapy. He has otherwise been healthy and has not traveled recently, had contact with sick persons, or been noncompliant with medication. On physical examination, the temperature is 38.3 °C (101 °F), the blood pressure is 130/76 mm Hg sitting and 105/60 mm Hg standing, the pulse rate is 90/min sitting and 120/min standing, and the respiration rate is 18/min. The abdomen is diffusely tender without rebound or guarding. Laboratory studies reveal hemoglobin 12.3 g/dL (123 g/L), leukocyte count of 28,000/µL (28 × 109/L) with 15% band forms, and platelet count of 234,000/µL (234 × 109/L). Intravenous fluids are started and stool studies are obtained. Which of the following is the most appropriate next step in the management of this patient? A Increase dosage of azathioprine B Start oral vancomycin C Start oral mesalamine D Small-bowel radiographic series

B Infectious causes should be considered in exacerbations of diarrhea in patients with inflammatory bowel disease. This patient likely has Clostridium difficile antibiotic-associated colitis complicating his underlying inflammatory bowel disease. C. difficile is an anaerobic gram-positive rod that produces two toxins, both capable of damaging the mucosa of the colon and causing pseudomembranous colitis. Infectious diarrhea associated with C. difficile has emerged as a major public health concern and can be seen in patients with underlying inflammatory bowel disease. Whenever a patient with inflammatory bowel disease presents with a new flair, stool studies, including C. difficile toxin assay, should be done. This patient's recent history of antibiotic use greatly increases his risk of C. difficile infection. The fever, orthostasis, leukocytosis, and abdominal tenderness in the setting of chronic immunosuppression are all signs that he needs to be hospitalized for further investigations (for example, CT scan to rule out toxic megacolon) and to start empiric therapy. Optimal therapy is orally administered metronidazole or vancomycin and should be initiated promptly for severely ill patients. It would be unwise to increase his immunosuppression either by adding prednisone or increasing the azathioprine in the setting of possible infection. There is no role for evaluation of the small bowel mucosa with a small-bowel series in order to diagnose small-bowel inflammation.

Gastro 9 A 38-year-old woman is evaluated for elevated results of liver chemistry tests detected in an evaluation for new-onset fatigue, joint pains, and jaundice. The patient recently started a job in a hospital and received a hepatitis B vaccination. She has a history of hypothyroidism, and her only medications are levothyroxine and a multivitamin. She has never used illicit drugs and does not drink alcohol. Her mother has rheumatoid arthritis. On physical examination, the patient is afebrile; the blood pressure is 130/75 mm Hg, the pulse rate is 80/min, and the respiration rate is 14/min. The BMI is 26. There is scleral icterus; the rest of the examination is normal. Laboratory studies: Leukocyte count 3400/µL (3.4 × 109/L) with a normal differential Bilirubin (total) 6.0 mg/dL (102.6 µmol/L) Bilirubin (direct) 3.6 mg/dL (61.6 µmol/L) Aspartate aminotransferase 890 U/L Alanine aminotransferase 765 U/L Alkaline phosphatase 120 U/L Antinuclear antibody Titer 1:40 (n < 1:80) Anti-smooth muscle antibody Titer 1:640 (erhöht) Antimitochondrial antibody Negative Viral serologic tests are negative. Which of the following is the most likely diagnosis? A Acute cholecystitis B Autoimmune hepatitis C Drug-induced liver injury D Primary biliary cirrhosis E Primary sclerosing cholangitis

B Laboratory findings in patients with autoimmune hepatitis include elevated serum aminotransferase values, hypergammaglobulinemia, mild hyperbilirubinemia, elevated serum alkaline phosphatase values, and the presence of autoantibodies. This patient has autoimmune hepatitis, a disorder that occurs most commonly in girls and young women. Like this patient with hypothyroidism, many affected patients have other autoimmune disorders and a family history of autoimmunity. These patients usually present with vague symptoms. Fatigue, which occurs in 85% of patients, is the most common presenting symptom, followed by jaundice (46%), anorexia (30%), myalgias (30%), and diarrhea. On physical examination, 78% of patients have an enlarged liver. Others have a normal examination despite the presence of advanced disease. The results of liver chemistry tests can be quite elevated, with aminotransferase concentrations into the thousands, but typically less than 500 IU at presentation, with elevated bilirubin, often with near-normal alkaline phosphatase. Autoimmune serologic tests, specifically antinuclear antibodies, anti-smooth muscle antibodies, and antibody to liver/kidney microsome type 1 (anti-LKM1), may be positive but are not detected in up to 25% of patients. Antibody levels are not prognostic of the disease course. Primary biliary cirrhosis is a chronic progressive cholestatic liver disease of unknown cause. It is an autoimmune disorder that occurs predominantly in women (80% to 90% of cases) between 40 and 60 years of age. The diagnostic triad associated with primary biliary cirrhosis includes a cholestatic liver profile, positive antimitochondrial antibody titers, and compatible histologic findings on liver biopsy. Serum alkaline phosphatase level is usually elevated ten times or more above normal. The patient's near normal alkaline phosphatase concentration and negative antimitochondrial antibody essentially rule out primary biliary cirrhosis. Although drug-induced liver injury can cause similar liver test abnormalities, the patient has not taken any new medications recently, making this diagnosis unlikely. She has no pain to suggest cholecystitis. Primary sclerosing cholangitis is a chronic cholestatic liver disease of unknown cause that is characterized by progressive bile duct destruction and may lead to secondary biliary cirrhosis. Laboratory findings include a cholestatic liver profile, with serum alkaline phosphatase levels three to five times greater than normal and mild hyperbilirubinemia. This patient's alkaline phosphatase level is minimally elevated making primary sclerosing cholangitis unlikely.

Gastro 51 A 30-year-old woman is evaluated for her risk for colon cancer. Her father and her sister both have colon cancer; her father was diagnosed at age 51 years and her sister at age 26 years. Her paternal uncle was diagnosed with colon cancer at age 57 years and with transitional cell cancer of the ureter at age 61 years. The patient had a screening colonoscopy 3 years ago that was normal; she is healthy and takes no medications. On physical examination, vital signs are normal; BMI is 21.5. Physical examination is normal. Blood testing for germline mutations of the MLH1 and MSH2 genes is negative. Which of the following is the most appropriate next step in the management of this patient? A Abdominal CT scan B Colonoscopy C Mammography D Repeat genetic testing

B Management of hereditary nonpolyposis colorectal cancer (HNPCC) requires regular testing for colon cancer and HNPCC-associated cancers starting at an early age. This patient's family history meets the Amsterdam II clinical criteria for hereditary nonpolyposis colorectal cancer (HNPCC), which use the 3-2-1 rule: three relatives with an HNPCC-associated cancer, two generations affected, one person diagnosed before age 50 years. She should therefore be considered to be at risk for cancers associated with the disorder despite the negative genetic tests. The patient was tested negative for germline mutations MLH1 and MSH2, but because there is no information whether a mutation had been identified in an affected member (her father, sister, or uncle), it is not clear whether the test represents a true negative in which case she is not at risk for HNPCC or if the test is uninformative. A genetic test for HNPCC must be interpreted along with consideration for the clinical and genetic data within the kindred. The strong family history of HNPCC-associated cancers suggests that the patient remains at risk for HNPCC, and she should be treated as if she is affected. Recommendations included colonoscopy every 1 to 2 years from age 20 to 25 years and surveillance for gynecologic malignancy every 1 to 2 years from age 30 years. Abdominal CT scan is not part of the consensus screening guideline for HNPCC. Breast cancer is not associated with HNPCC. False-negative germline testing is very rare and repeating the blood testing is not indicated.

Gastro 94 A 50-year-old man is evaluated after having had his first screening colonoscopy. The colonoscopy revealed a 12-mm polyp in the sigmoid colon and a 7-mm polyp in the transverse colon; both polyps were tubular adenomas. He has no personal or family history of inflammatory bowel disease, colon polyps, or colon cancer. The patient is healthy and takes no medications. On physical examination, vital signs are normal; BMI is 25.5. When should the next colonoscopy be scheduled for this patient? A 1 year B 3 years C 5 years D 10 years

B Patients who had high-risk colon polyps (three or more adenomas, high-grade dysplasia, villous features, or >1 cm in size) should have a repeat colonoscopy in 3 years, whereas those who had lower risk polyps (one or two <1-cm tubular adenomas with no high-grade dysplasia) can wait 5 to 10 years, and those who had only hyperplastic polyps can wait 10 years. This patient has had two adenomas identified on colonoscopy. The largest adenoma is greater than 1 cm in diameter and therefore requires a repeat colonoscopy in 3 years. Predictors for recurrent advanced adenoma on surveillance examinations include the presence of large polyps (>1 cm), more than three adenomas, villous histology, and adenoma with advanced dysplasia. If only one or two adenomas are detected, and they are less than 1 cm in diameter, 5-year colonoscopic surveillance is appropriate. If an initial colonoscopy were normal or reveals only hyperplastic polyps in a patient with no family history of colon cancer, a 10-year interval would be appropriate.

Gastro 50 A 37-year-old man is evaluated for several weeks of progressive dysphagia, primarily for solid foods but also for liquids, associated with regurgitation of undigested food, nausea and vomiting, chest pain, and a 6.8-kg (15-lb) weight loss. The patient's only significant medical history is Hodgkin disease diagnosed 15 years ago, which was treated with mantle radiation. On physical examination, the patient appears anxious but not in distress. Vital signs and physical examination are normal. Barium esophagography shows a dilated atonic esophagus with a tapered narrowing at the gastroesophageal junction. Esophageal manometry shows incomplete relaxation of a hypertensive lower esophageal sphincter with aperistalsis of the esophageal body. CT scan of the chest and abdomen confirms the dilated esophagus but shows no masses or lymphadenopathy. Esophagogastroduodenoscopy shows a mildly dilated esophagus with retained secretions and a 3-cm mass at the gastroesophageal junction. Biopsy specimens of the mucosa are negative for cancer. Which of the following is the most appropriate management for this patient? A Botulinum toxin injection of the distal esophagus B Endoscopic ultrasonography of the esophagus with needle biopsy C Pneumatic dilatation of the esophagus D Repeat mucosal biopsy E Thoracoscopic exploration

B Patients with pseudoachalasia tend to be older than patients with achalasia, have a shorter duration of symptoms, and have greater weight loss. This patient presents with classic symptoms of achalasia: relatively short history of dysphagia, dilated esophagus, "bird's beak" abnormality on barium study, and typical manometric criteria for the diagnosis. However, in the presence of a mass lesion of the distal esophagus, the diagnosis of pseudoachalasia must be considered. Pseudoachalasia, also known as secondary achalasia, is not a primary motility disorder of the esophagus; rather it is a result of obstruction or neuronal infiltration at the gastroesophageal junction. The lesion is often neoplastic. Differentiation of pseudoachalasia from true achalasia can be problematic. Patients with pseudoachalasia tend to be older than patients with achalasia, have a shorter duration of symptoms, and have greater weight loss. Neither barium esophagography nor esophageal manometry can distinguish between primary and secondary achalasia. In this case, a history of lymphoma and the presence of a submusosal lesion require further evaluation. Endoscopic ultrasonography will allow delineation of the depth and invasiveness of the lesion, as well as permit transluminal needle biopsy of the mass. Repeat mucosal biopsy is unlikely to be of sufficient depth to make a tissue diagnosis. Surgical exploration is not necessary to retrieve an adequate histologic sample of the lesion. Pneumatic dilatation and botulinum toxin injection are reserved for patients with primary achalasia, not pseudoachalasia, which frequently responds to treatment of the underlying disorder.

Gastro 34 A 64-year-old man is evaluated for a 3-month history of abdominal bloating and mid-epigastric discomfort associated with a 6.8-kg (15-lb) weight loss. The patient has no significant medical history and takes no medications. On physical examination, vital signs are normal, and the only significant finding is mild epigastric tenderness. Laboratory studies: Complete blood count Normal Aspartate aminotransferase 55 U/L Alanine aminotransferase 67 U/L Amylase 184 U/L Lipase 382 U/L Helical CT scan of the abdomen shows a 2.8-cm pancreatic body mass. There are no liver lesions and no invasion into surrounding major vessels. Endoscopic ultrasonography confirms the presence of an approximately 3-cm lesion without vascular invasion. Fine-needle aspiration specimen is positive for adenocarcinoma. Which of the following is the most appropriate next step in the management of this patient? A Combined radiation therapy and chemotherapy B Distal pancreatectomy C Palliative care consultation D Pancreatic enzyme supplementation

B Surgery is the only treatment that provides a potential cure in patients with localized pancreatic cancer, with a 5-year survival rate of 10% to 30%. At the time of diagnosis, about 80% to 85% of pancreatic cancers are unresectable because of distant metastases or invasion or encasement of the major blood vessels. Treatment of pancreatic cancer that has not metastasized nor spread to the local vasculature is surgical resection, with distal pancreatectomy being the preferred procedure for lesions of the pancreatic body. Evaluation of whether the tumor is resectable pre-operatively is performed with a combination of helical CT of the abdomen and endoscopic ultrasonography. Even with surgery and complete resection of the tumor, the 5-year survival rate is only 10% to 30%. Concurrent radiation therapy and chemotherapy alone delays disease progression and may improve survival in patients with localized unresectable pancreatic cancer but will not provide a cure in patients with localized resectable pancreatic cancer. Pancreatic enzymes are used in patients with chronic pancreatitis or after pancreatic surgery to treat pancreatic malabsorption. In this otherwise healthy patient with a localized lesion and a potential for curative resection, palliative care is not indicated.

Gastro 73 A 34-year-old woman is evaluated for a 6-month history of nausea and vomiting. The symptoms were initially mild but have become progressively severe, and she now has daily symptoms that are exacerbated by eating. She has early satiety and postprandial bloating; she vomits partially digested food a few hours after eating. She has mild epigastric burning but no other features of abdominal pain. She has lost 6.8 kg (15 lb) since the beginning of this episode; her most recent menses was 1 week ago. The patient's medical history includes endometriosis and migraine, and her only medication is as-needed ibuprofen. Her mother has scleroderma and her father has type 2 diabetes mellitus. On physical examination, the patient is thin (BMI 20); the abdomen is distended with a succussion splash and mild epigastric tenderness; there are no palpable abdominal masses or hepatosplenomegaly. Laboratory studies, including liver enzymes and fasting blood glucose, are normal. Which of the following is the most appropriate next step in the evaluation of this patient? A Biliary ultrasonography B Esophagogastroduodenoscopy C Gastric scintigraphy D Measurement of anticentromere and anti-scl-70 antibodies E Oral glucose tolerance test

B The initial step in the evaluation of delayed gastric emptying is esophagogastroduodenoscopy to rule out mechanical obstruction as a cause of symptoms. This patient has clinical and examination findings suggestive of delayed gastric emptying, but it is not possible to distinguish mechanical obstruction from gastroparesis from the history and examination alone. The patient takes ibuprofen regularly, and, therefore, peptic ulceration causing gastric outlet obstruction needs to be considered; an esophagogastroduodenoscopy would be the next step to exclude mechanical obstruction. If the initial evaluation is negative for obstruction, gastric scintigraphy would be a reasonable test to evaluate for gastroparesis. Biliary ultrasonography could be considered if abdominal pain were a prominent feature of the clinical presentation, although her symptoms are not compatible with biliary colic and it would not explain the historical or examination features of delayed gastric emptying. After gastroparesis has been established based on an abnormal scintigraphic study, an evaluation to determine an underlying cause may include screening for diabetes mellitus and serologic tests to detect an underlying autoimmune rheumatologic condition, which may be reasonable given her family history; however, these tests would not be recommended as the next step in this patient's evaluation.

Gastro 43 A 32-year-old man is evaluated for a 2-week history of nausea, malaise, low-grade fever, vomiting, and jaundice. He has no other significant medical history and takes only ibuprofen for headache and fever. On physical examination, the temperature is 37.6 °C (99.7 °F), the blood pressure is 110/75 mm Hg, the pulse rate is 90/min, and the respiration rate is 22/min; the BMI is 25. Examination reveals scleral icterus, jaundice, 1+ pitting lower extremity edema, hepatomegaly, mild asterixis, and somnolence. There are no stigmata of chronic liver disease. Laboratory studies: Bilirubin (total) 17.5 mg/dL (393.3 µmol/L) Bilirubin (direct) 7.2 mg/dL (123.1 µmol/L) Aspartate aminotransferase 8790 U/L Alanine aminotransferase 7650 U/L Alkaline phosphatase 195 U/L INR 2.3 Hepatitis B surface antigen Positive Hepatitis B core antigen (IgM) Positive Hepatitis C antibody Negative Hepatitis A total antibody Positive Blood alcohol Negative Acetaminophen Undetectable Ultrasonography shows hepatomegaly and increased echogenicity, a normal spleen, and perihepatic ascites. There is no ductal dilatation. Which of the following is the most appropriate management for this patient? A Begin interferon therapy B Evaluation for liver transplantation C Endoscopic retrograde cholangiopancreatography D Corticosteroids

B About 5% patients with acute hepatitis B virus infection develop acute progressive hepatitis B with fulminant hepatic decompensation and need urgent liver transplantation. The markedly elevated aminotransferase levels, positive hepatitis B surface antigen, and IgM antibody to hepatitis B core antigen establish the diagnosis of acute hepatitis B. Patients at greatest risk for exposure to hepatitis B virus (HBV) infection are those with a history of multiple sexual partners and injection drug users. The course of HBV infection depends mainly on the age at which a patient is exposed. Most patients who are first exposed as adults develop flu-like symptoms with malaise, nausea, vomiting, and diarrhea with associated jaundice. Children on the other hand most often do not develop symptoms after exposure to HBV. Despite the presentation of an acute infection, most adult patients will clear their infection after a few months. Therefore, most patients can be monitored as outpatients and treated for symptoms only. However, about 5% patients develop acute progressive hepatitis B with hepatic decompensation and need urgent liver transplantation. These patients tend to have an elevated INR and a rising bilirubin level and may develop encephalopathy, a marker of fulminant hepatic failure. This subgroup of patients needs to be recognized, admitted to the hospital, and started on antiviral therapy. Telbivudine, lamivudine, adefovir, and entecavir are acceptable options. Transplantation evaluation should be pursued. Transplantation should be performed when a poor outcome is anticipated but before the patient develops uncontrolled sepsis or prolonged periods of increased intracranial pressure that would preclude recovery even if a functioning liver is transplanted. In the United States, patients with fulminant hepatic failure are given highest priority for transplantation and often receive a transplanted liver within 1 week of evaluation. Interferon should be avoided in patients with acute hepatitis B and particularly in fulminant hepatic failure due to hepatitis B because of the increased risk of hepatic necroinflammation. The patient has no evidence of ductal dilatation on ultrasonography, and therefore, there is a low yield and high risk to proceed with endoscopic retrograde cholangiopancreatography. Corticosteroids have been studied as a treatment for fulminant hepatic failure and have been found to be not effective. In addition, corticosteroid use is associated with an increased risk for infectious complications.

Gastro 83 A 67-year-old woman is evaluated for a 2-month history of epigastric discomfort, a burning sensation that does not radiate and is not associated with eating. She has had episodic mild nausea and intermittent bloating but no acid regurgitation, heartburn, dysphagia, or odynophagia. She has one bowel movement of well-formed stool a day, and her weight is stable. The patient has a history of osteopenia and gallstone pancreatitis for which she had a cholecystectomy; her medications include calcium and vitamin D supplements. Her mother had gastric cancer. On physical examination, vital signs are normal. There is mild epigastric tenderness, normal bowel sounds with no bruits, no rebound or guarding, and no hepatomegaly or lymphadenopathy. Complete blood count and serum chemistry tests, including liver enzymes, amylase, and lipase, are normal. Which of the following is the most appropriate management of this patient? A Endoscopic retrograde cholangiopancreatography B Esophagogastroduodenoscopy C Helicobacter pylori stool antigen assay D Nortriptyline at bedtime E Proton pump inhibitor empiric trial

B Alarm features associated with dyspepsia include age greater than 55 years with new-onset symptoms, family history of gastric cancer, weight loss, gastrointestinal bleeding, dysphagia, odynophagia, iron-deficiency anemia, vomiting, and abnormal examination findings. This patient has new-onset dyspepsia, and although she does not have any worrisome clinical features, the new onset of symptoms at her age and her mother's history of gastric cancer constitute alarm features, and therefore, further evaluation with esophagogastroduodenoscopy is warranted. Alarm features associated with dyspepsia include the following: age greater than 55 years with new-onset symptoms, family history of gastric cancer, weight loss, gastrointestinal bleeding, dysphagia, odynophagia, iron-deficiency anemia, vomiting, or abnormal examination findings. Although pancreatic disease, such as adenocarcinoma of the pancreas, may present with similar clinical symptoms, evaluation with endoscopic retrograde cholangiopancreatography would not precede upper endoscopy or cross-sectional imaging, especially because of the increased invasiveness of this test. If the patient had not had any alarm features, testing for Helicobacter pylori and treating if positive would be recommended. In this patient with dyspepsia and alarm features, it would be inappropriate to simply diagnose functional dyspepsia or treat her with a neuromodulating agent before a fuller evaluation. An empiric trial with a proton pump inhibitor would be reasonable in a patient with features suggestive of gastroesophageal reflux, but should not replace further evaluation in this elderly patient with new symptoms.

Gastro 13 A 60-year-old man hospitalized for advanced cirrhosis complicated by ascites and encephalopathy is evaluated for massive hematemesis and hypotension. The patient's medications are spironolactone, furosemide, and lactulose. On physical examination, the temperature is 35.6 °C (96 °F), the blood pressure is 80/50 mm Hg, the pulse rate is 146/min, and the respiration rate is 20/min. The patient has just vomited red blood and has large-volume ascites; the stool is brown and positive for occult blood. Laboratory studies show hemoglobin of 9 g/dL (90 g/L), platelet count of 60,000/µL (60 × 109/L), and INR of 3. In addition to rapid volume resuscitation, which of the following is the most appropriate management of this patient? A Arteriography B Esophagogastroduodenoscopy C Intravenous nadolol D Mesocaval shunt E Transjugular intrahepatic portosystemic shunt

B Antibiotics, endoscopic variceal band ligation, and intravenous octreotide are the first-line therapies for acute esophageal variceal bleeding. The first step in the management of acute variceal hemorrhage is the restoration of the intravascular volume using a large bore peripheral intravenous line or a central line. Packed erythrocytes are used as need to replace blood loss and clotting factors are replaced as needed. Platelet transfusions may be indicated if values fall below 50,000/µL (50 × 109/L). In addition, this patient should undergo urgent esophagogastroduodenoscopy and band ligation of esophageal varices. Clinical studies have shown that sclerotherapy was superior to balloon tamponade alone, vasopressin alone, and a combination of vasopressin and balloon tamponade in controlling active variceal hemorrhage, preventing early rebleeding, and improving survival in patients with esophageal and gastroesophageal varices. Band ligation has been shown to be as effective as sclerotherapy for preventing early rebleeding. Therapy should also be started with intravenous octreotide, which reduces portal venous blood inflow through inhibition of the release of vasodilatory hormones and is more effective for controlling bleeding than placebo; however, its ultimate effect on survival is unknown. Bacterial infections are present in a sizable percentage of hospitalized patients with variceal bleeding and are associated with a high mortality rate. Clinical trials have demonstrated that the use of prophylactic antibiotics in these patients results in a reduction in infectious complications and possibly mortality. Arteriography is not first-line therapy in patients with a variceal bleed from venous portal hypertension. Arteriography is reserved for patients with a presumed arterial source of bleeding as can be seen in peptic ulcer disease or tumors anywhere along the gastrointestinal tract. In such cases, arteriography can be used to identify and embolize the specific vessel involved. This method is usually reserved for cases in which the patient is actively bleeding and either endoscopic therapy has failed to stop the bleeding or the presence of active bleeding interferes with identification of the bleeding site and the patient is unstable. Mesocaval shunt is a surgical shunt that decompresses the portal system by diverting portal venous flow into the inferior vena cava, thus bypassing the liver. This type of shunt is rarely used for acute variceal bleeding as it carries a high intraoperative risk in an acutely decompensated patient with underlying liver disease and coagulopathy. Intravenous nadolol is not appropriate because this patient is hypotensive and needs endoscopic intervention rather than medical therapy. Lastly, transjugular intrahepatic portosystemic shunting (TIPS) should not be used without first performing an upper endoscopy with esophageal band ligation. Many variceal bleeds can be controlled with endoscopic therapy alone, obviating the need for urgent TIPS which can have significant comorbidities including hepatic decompensation, bleeding complications, or resultant encephalopathy.

Gastro 90 A 30-year-old man is evaluated in the emergency department for a 3-week history of malaise, fatigue, and jaundice. The patient is an injection drug user and was diagnosed with chronic hepatitis B virus infection 4 years ago. He drinks beer occasionally and does not use any medications, including over-the-counter medications. On physical examination, the temperature is 36.1 °C (97.0 °F), the blood pressure is 120/65 mm Hg, the pulse rate is 100/min, and the respiration rate is 16/min. Abdominal examination reveals mild tenderness to palpation in the right upper quadrant. Laboratory studies: INR 1.2 Bilirubin (total) 3.5 mg/dL (59.9 µmol/L) Bilirubin (direct) 1.8 mg/dL (30.8 µmol/L) Aspartate aminotransferase 1100 U/L Alanine aminotransferase 1450 U/L Hepatitis B surface antigen Positive Hepatitis B core antibody (IgG) Positive Hepatitis B core antibody (IgM) Negative Hepatitis B virus DNANegative Hepatitis C virus antibodyNegative AcetaminophenNegative Ultrasonography shows heterogeneous hepatic echotexture with no masses. Which of the following is the most likely diagnosis? A Acute alcoholic hepatitis B Hepatitis delta virus superinfection C Hepatocellular carcinoma D Reactivated hepatitis B virus infection

B Chronic carriers of hepatitis B are susceptible to superinfection with hepatitis delta virus, which is a defective virus that requires the presence of hepatitis B virus to exist. This patient likely has a superimposed hepatitis delta virus (HDV) infection, which is most common among injection drug users. HDV is a defective virus that requires the presence of hepatitis B virus (HBV) to exist. The outer lipoprotein envelope of the hepatitis D virus is made of the surface antigen of the HBV (HBsAg). Therefore, detectable hepatitis B surface antigen is necessary for the diagnosis of HDV infection. Two major patterns of serologic response may be seen depending upon the type of infection. In acute HBV/HDV D coinfection, the presence of IgM antibody to hepatitis B core antigen (anti-HBc IgM) is present. This simultaneous infection is clinically indistinguishable from acute HBV infection, although it may be more severe and is associated with a higher prevalence of acute liver failure. In acute HDV superinfection, the IgM antibody to hepatitis B core antigen is negative but the IgG antibody is positive. This type of infection may present clinically as either an apparent worsening of a previously established chronic HBV infection or an acute infection of viral hepatitis in a person with previously unrecognized chronic HBV infection. In this situation, the superimposed HDV infection suppresses hepatitis B replication and the HBV DNA level becomes undetectable. The reason for the suppression of HBV virus replication is unknown. Patients with HDV superinfection may be at greater risk for progression to cirrhosis. This patient's clinical presentation of an acute worsening of a chronic hepatitis B infection is most compatible with a superinfection with HDV. It is possible to diagnose HDV infection by using total anti-HDV antibodies, which would be the next diagnostic step for this patient; IgM and IgG can be detected by enzyme immunoassay or radioimmunoassay. Acute alcoholic hepatitis is unlikely given the lack of a history of alcohol ingestion and the degree of elevation and pattern of the aminotransferases. In alcohol injury, the aspartate aminotransferase concentration tends to be higher than the alanine aminotransferase but rarely greater than 300 U/L. Ultrasonography did not show a space-occupying liver lesion, and therefore, hepatocellular carcinoma is unlikely. Furthermore, hepatocellular carcinoma is associated with cirrhosis, which is not present in this patient. Reactivation of hepatitis B can occur, but in this case the anti-HB core IgM and DNA should be positive.

Gastro 93 A 38-year-old man is evaluated for a 2-month history of progressive mid-epigastric pain that is worse after eating, postprandial nausea, and a 4.6-kg (10 lb) weight loss. The patient has a 5-year history of chronic pancreatitis and has six alcohol-containing drinks a day. His medications are amitriptyline, oral morphine, and pancreatic enzyme supplements. On physical examination, the patient is thin (BMI 20) and appears to be in mild distress. There is epigastric tenderness without rebound or guarding. The liver is slightly enlarged, but there are no palpable masses. Laboratory studies reveal a normal complete blood count and serum amylase of 175 U/L and lipase of 333 U/L. CT scan of the abdomen and pelvis shows multiple pancreatic calcifications, a calcified stone in the head of the pancreas within the main pancreatic duct, as well as dilation of the duct in the body and tail of the gland. In addition to alcohol cessation, which of the following is the most appropriate management for this patient? A Celiac nerve block B Endoscopic retrograde cholangiopancreatography with removal of stones C Increasing the dose of pancreatic enzymes D Pancreatoduodenectomy

B Endoscopic treatment of pain in chronic pancreatitis is performed by removing pancreatic duct stones and placing stents in pancreatic duct strictures to decrease pancreatic duct pressure. Patients with chronic pancreatitis must avoid alcohol. Patients who continue to drink alcohol have an increase in painful attacks and mortality. Pain in chronic pancreatitis results from chronic inflammation, chronic noxious stimulation of the nerves to the pancreas, and increased pancreatic intraductal pressure secondary to pancreatic duct stones, calcifications, or strictures. Large stones in the pancreatic duct can be crushed with extracorporeal shock wave lithotripsy; endoscopic retrograde cholangiopancreatography can then remove the stones and place stents in pancreatic duct strictures to decrease pancreatic duct pressure. Studies have documented symptom improvement in 11% to 75% of patients and resolution of stricture in 10% to 50%. A surgical pancreatoduodenectomy (Whipple procedure) can be performed to relieve pain but is effective only in patients who have disease limited to the head of the pancreas and who have failed to respond to medical and endoscopic therapy. A surgical procedure to divert the pancreatic duct into the small intestine has been used widely in the treatment of patients with a chronic pancreatitis and a dilated pancreatic duct and is effective in many patients. The procedure involves removing pancreatic tissue that overlies the ductal system in the head of the pancreas. A less invasive procedure is preferred to surgical intervention as the next management step. Celiac nerve block has been used to treat chronic pancreatitis pain but is considered by many experts to be an unproved therapy. Even in patients who respond, pain returns in 2 to 6 months and significant procedural complications have been reported. Furthermore, it would not be the first procedure of choice in a patient with a pancreatic ductal stone and evidence of obstruction. Pancreatic enzyme supplements are not consistently effective for pain control in chronic pancreatitis.

Gastro 56 A 39-year-old woman is evaluated for a 6-week history of nausea and intermittent vomiting, which began after a 3-day episode of severe nausea, vomiting, and diarrhea. At the same time, her school-age children had a similar episode that resolved spontaneously. However, since that time, the patient has had daily nausea worsened by eating, early satiety, bloating, and vomiting of large volumes of nonbilious food. She does not have abdominal pain, but in the past 6 weeks, she has lost 4.5 kg (10 lb). The patient's medical history includes a transsphenoidal resection of a nonfunctioning pituitary adenoma 3 years ago and two deliveries by cesarean section. Her mother has depression, and her father has primary sclerosing cholangitis. On physical examination, she is thin (BMI 19); vital signs are normal. She responds appropriately and makes normal eye contact; there is no jaundice or scleral icterus. There is mild abdominal distention and normal bowel sounds without succussion splash or tenderness. Laboratory studies, including liver enzymes, are normal. Esophagogastroduodenoscopy reveals retained gastric contents but no obstruction. Radiograph of the small bowel is normal. Which of the following is the most appropriate next step in the evaluation of this patient? A Biliary ultrasonography B Gastric scintigraphy C MRI of the head D Referral for biofeedback training E Viral culture of stool

B Four-hour gastric scintigraphy of a solid meal is the test of choice for establishing the diagnosis of gastroparesis. This patient has the typical clinical features of delayed gastric emptying after an episode of infectious gastroenteritis. The abrupt onset of this syndrome makes postviral gastroparesis likely, especially because mechanical obstruction has been excluded. The test of choice to substantiate the diagnosis of gastroparesis is a 4-hour gastric scintigraphy of a solid meal. In the absence of abdominal pain and with normal liver chemistry studies, the utility of biliary ultrasonography would be low in the evaluation of her symptoms. Although central nervous system lesions should always be considered in a patient with nausea and vomiting of unknown cause, the presence of her early satiety, bloating, and retained gastric contents makes central nervous system pathology unlikely, and, therefore, MRI of the head would not be indicated. Patients with cyclical vomiting syndrome or bulimia may benefit from biofeedback training, but neither condition is likely in this patient. Patients with cyclical vomiting tend to have discrete periods of intense vomiting, often feeling well between episodes, whereas patients with bulimia are unlikely to present for an evaluation of vomiting and do not have an abrupt onset as in this patient. Even though this patient likely has postviral gastroparesis, stool cultures for a viral infection would not be helpful because the acute viral infection has resolved.

Gastro 29 A 55-year-old woman is evaluated in the hospital for a 2-day history of epigastric abdominal pain, nausea and vomiting, and anorexia. The patient has no significant medical history and takes no medications. On physical examination, the temperature is 38.0 °C (100.5 °F), the blood pressure is 124/76 mm Hg, the pulse rate is 99/min, and the respiration rate is 16/min. There is scleral icterus and a slight yellowing of the skin. There is mid-epigastric and right upper quadrant tenderness. There is no palmar erythema, spider angiomata, or other evidence of chronic liver disease. Laboratory studies: Leukocyte count 14,900/µL (14.9 × 109/L) Aspartate aminotransferase 656 U/L Alanine aminotransferase 567 U/L Bilirubin (total) 5.6 mg/dL (95.8 µmol/L) Amylase 1284 U/L Lipase 6742 U/L Abdominal ultrasonography shows a biliary tree with a dilated common bile duct of 12 mm and cholelithiasis but no choledocholithiasis. Which of the following is the most appropriate next step in the management of this patient? A CT scan of the abdomen and pelvis with pancreatic protocol B Endoscopic retrograde cholangiopancreatography C Hepatobiliary iminodiacetic acid (HIDA) scan D Magnetic resonance cholangiopancreatography

B In patients with gallstone pancreatitis and evidence of biliary obstruction, endoscopic retrograde cholangiopancreatography and stone removal reduces morbidity and mortality by reducing the risk of biliary sepsis. This patient has a classic presentation of acute pancreatitis with the acute onset of epigastric abdominal pain, nausea, and vomiting associated with markedly elevated pancreatic enzymes. The presence of stones in the gallbladder, a dilated bile duct, and elevated aminotransferase levels highly suggest gallstones as the cause of pancreatitis. The scleral icterus, jaundice, and elevated bilirubin level suggest continuing bile duct obstruction. Abdominal ultrasonography has a sensitivity of only 50% to 7lllll5% for choledocholithiasis, and a common duct stone should be suspected in the correct clinical situation even when ultrasonography does not show a stone. Endoscopic retrograde cholangiopancreatography (ERCP) with sphincterotomy and stone removal is the most appropriate procedure in patients with acute gallstone pancreatitis and with imaging and biochemical evidence of biliary obstruction from a common duct stone. The procedure can document the diagnosis of choledocholithiasis and remove the gallstones, which lessens the morbidity and mortality due to biliary sepsis. CT scan will likely show evidence of acute pancreatitis and magnetic resonance cholangiopancreatography (MRCP) will show pancreatitis and the presence of a common duct stone, with sensitivities for CT being approximately 75% for stones and MRCP having sensitivities ranging from 80% to 100% as compared with ERCP for the diagnosis of choledocholithiasis. Biliary scintigraphy may show obstruction of the cystic or common bile duct but will not determine the cause. However, CT, biliary scintigraphy, and MRCP will not be therapeutic for bile duct stones.

Gastro 97 A 32-year-old woman is evaluated in the hospital. She was hospitalized 2 days ago because of nausea, vomiting, and dehydration as a result of documented gastroparesis secondary to long-standing, difficult-to-control type 1 diabetes mellitus. Her symptoms had been controlled with oral erythromycin therapy, but the therapy became ineffective with time. She recently began taking metoclopramide, which controlled the nausea and vomiting but caused tardive dyskinesia, and the therapy was stopped. Oral antiemetic therapy has not prevented vomiting, and modifying her diet by eating multiple, small-volume, low-residue meals or limiting her diet to liquid supplements only has not been effective. Over the past 3 months, she has lost 5.8 kg (15 lb) and been hospitalized four times. In addition to type 1 diabetes mellitus, her medical history includes erosive esophagitis, and her medications include insulin glargine and insulin lispro and pantoprazole. On admission she was given intravenous fluids, and her volume status improved over 2 days. On physical examination, she is afebrile, the blood pressure is 105/68 mm Hg with no orthostatic changes, and the pulse rate is 88/min; the BMI is 21. The patient has intermittent dystonic movements of the mouth; the mucous membranes are moist, skin turgor is intact, and abdominal examination is normal. Laboratory studies reveal a plasma glucose of 160 mg/dL (8.88 mmol/L); all other studies, including complete blood count, serum creatinine, liver chemistry tests, thyroid-stimulating hormone, calcium, tissue transglutaminase, and cortisol, and spot urine for microalbumin, are normal. Which of the following is the most appropriate next step in the management of this patient? A Metoclopramide at a reduced dose B Nasojejunal feeding trial C Percutaneous gastrostomy tube placement D Total parenteral nutrition

B In patients with gastroparesis who fail to meet nutritional needs, alternative modalities for enteral feeding need to be considered. This patient with gastroparesis has had weight loss, dehydration, and recurrent hospital admissions, and diet modification and pharmacologic therapy have failed to control her symptoms. Because she cannot meet her nutritional needs, a nasojejunal feeding trial would be recommended to provide enteral nutrition support beyond the stomach. If such a feeding trial is successful, a more permanent feeding jejunostomy with or without a venting gastrostomy could be considered. Because she developed tardive dyskinesia while taking metoclopramide, restarting the drug, even at a reduced dose, would not be appropriate. Placing a percutaneous gastrostomy tube and feeding into the stomach will likely offer no additional clinical benefit because the patient did not tolerate liquid supplements by mouth; therefore, feeding beyond the stomach into the jejunum would be recommended. Although total parenteral nutrition may be needed in some patients with gastroparesis, it has long-term risks and is, therefore, reserved for patients who cannot tolerate enteral feeding.

Gastro 69 A 20-year-old woman is evaluated in the emergency department for a 2-week history of malaise, fatigue, and mild jaundice. The patient has no significant medical history, but she uses injection drugs. She drinks alcohol socially. On physical examination, the temperature is 37.8 °C (100.0 °F), the blood pressure is 128/70 mm Hg, the pulse rate is 100/min, and the respiration rate is 16/min. Examination reveals slight scleral icterus, needle puncture marks in the antecubital fossae, and hepatomegaly; there is no splenomegaly, cutaneous angiomata, ascites, or asterixis. Laboratory studies: Bilirubin (total) 4.6 mg/dL (78.7 µmol/L) Aspartate aminotransferase 580 U/L Alanine aminotransferase 750 U/L Alkaline phosphatase 145 U/L Albumin 4.2 g/dL (42 g/L) Hepatitis B surface antigen Negative Hepatitis B core antibody (IgG and IgM) Negative Hepatitis C virus antibody Negative Hepatitis A virus antibody (IgG and IgM) Negative Drug and alcohol screens Negative Ultrasonography shows hepatomegaly. In addition to screening for HIV infection, which of the following is the most appropriate next diagnostic test? A Hepatitis B virus DNA B Hepatitis C virus RNA C Liver biopsy D MRI of the liver

B In possible acute hepatitis C virus infection in seronegative patients, the most sensitive diagnostic test is measurement of hepatitis C viral RNA. The patient has signs of an acute hepatitis, including elevated liver test results with concurrent fatigue, malaise, and jaundice, and a recent exposure putting her at risk for viral hepatitis (injection drug use). Acute hepatitis C virus (HCV) infection may occur as a result of injection drug use, and although most infected persons do not develop a clinically apparent acute hepatitis, approximately 20% develop an acute infectious episode. Infected patients may remain seronegative for longer than 8 weeks, and therefore, in this clinical setting with a possibility of recent HCV infection, the most appropriate test would be measurement of HCV RNA. HCV RNA can be measured by PCR-based methods or signal amplification technologies. Hepatitis B virus (HBV) DNA should be measured only if the patient is positive for hepatitis B surface antigen (HBsAg). HBsAg appears in serum 1 to 10 weeks after an acute exposure to hepatitis B virus and is present in the blood prior to the onset of symptoms or liver aminotransferase elevation. IgM anti-HBc can be the only serologic evidence of HBV infection during the period between the disappearance of HBsAg and the appearance of anti-HBs. Since the serologic tests for HBV are negative in this symptomatic person with elevated liver chemistry tests, there is no value in measuring HBV DNA. Acute hepatitis C is associated with characteristic histopathologic features including steatosis, lymphoid aggregates, and bile duct damage. However, these findings are not specific for HCV and similar findings are associated with acute HBV infection. Liver biopsy in this setting will not lead to a specific diagnosis without complementary serologic testing. MRI of the liver will detail hepatic morphology but will not contribute any more to the diagnosis than the ultrasonography.

Gastro 91 A 57-year-old woman with cirrhosis is evaluated for worsening ascites and abdominal discomfort. The patient has hepatitis C virus infection, which failed to respond to antiviral therapy. She has been followed with regular cross-sectional imaging. Her medications are furosemide, spironolactone, nadolol, lactulose, and acetaminophen. On physical examination, the patient is afebrile; the blood pressure is 90/50 mm Hg, the pulse rate is 80/min, and the respiration rate is 14/min. The BMI is 26. Examination reveals scleral icterus, a firm palpable liver edge, bulging flanks with shifting dullness, spider angiomata, caput medusae, and 2+ lower extremity edema. Palpation does not elicit Murphy sign, and there is no guarding, rebound, or peritoneal signs. Laboratory studies Hemoglobin 16.4 g/dL (164 g/L) Platelet count 45,000/µL (45 × 109/L) INR 1.7 Creatinine 1.4 mg/dL (123.76 µmol/L) Bilirubin (total) 1.8 mg/dL (30.78 µmol/L) Bilirubin (direct) 0.9 mg/dL (15.39 µmol/L) Aspartate aminotransferase 67 U/L Alanine aminotransferase 45 U/L Alkaline phosphatase 120 U/L Albumin 2.6 g/dL (26 g/L) α-Fetoprotein 200 ng/mL (200 µg/L) (n = bis 10 ng/ml) Amylase 75 U/L Lipase 58 U/L MRI of the abdomen shows patent hepatic vessels, cirrhosis, and a new 4.2-cm enhancing mass in the right hepatic lobe. There is no tumor thrombus, and CT scan of the chest shows no pulmonary nodules. Which of the following is the most appropriate management for this patient? A Biopsy of the liver mass B Evaluation for liver transplantation C Resection of the liver mass D Systemic chemotherapy

B Liver transplantation has become the primary treatment of choice for many patients with cirrhosis and hepatocellular carcinoma. This patient has radiographic evidence of cirrhosis with recent worsening hepatic decompensation manifested by worsening of ascites. Patients who develop hepatocellular carcinoma can, like this patient, present with new hepatic decompensation. The hemoglobin concentration is also elevated, a finding that can be a paraneoplastic manifestation of hepatocellular carcinoma. Furthermore, the findings on MRI are consistent with hepatocellular carcinoma in a patient with underlying cirrhosis. The patient has a single hepatocellular carcinoma in the absence of vascular involvement or extrahepatic spread; the stage of the tumor (T2) is within the Milan criteria for transplantation. Therefore, the next best management option is evaluation for liver transplantation. When the imaging suggests hepatocellular carcinoma and there is either an elevation of the serum α-fetoprotein concentration or another image finding suggestive of hepatocellular carcinoma, diagnostic biopsy is not necessary, and there is a small but real risk for tumor seeding with biopsy of this lesion. Hepatic resection is reserved for a select group of patients with hepatocellular carcinoma. Such patients need to have stable disease with good hepatic reserve. Thus hepatic resection is often limited either to patients without underlying cirrhosis or patients who have stable cirrhosis but no evidence of portal hypertension or hepatic dysfunction, which would lead to hepatic decompensation and increased patient morbidity after resection. Although systemic chemotherapy may be considered for patients who have extensive tumors who do not meet the Milan criteria, the preferred therapy in this patient is transplantation rather than systemic chemotherapy.

Gastro 68 A 65-year-old woman is evaluated for a 6-month history of watery, nonbloody diarrhea; she has from 3 to 20 bowel movements a day. She also has abdominal cramps, bloating, and occasional fecal incontinence and has lost 2.2 kg (5 lb) since the beginning of the episode. She had been previously healthy. She has not traveled recently or used antibiotics. On physical examination, the temperature is 37.2 °C (99.0 °F), the blood pressure is 118/82 mm Hg, the pulse rate is 76/min, and the respiration rate is 14/min. The heart rate is regular and the chest is clear. The abdomen is soft with slight distention. Which of the following is the most likely diagnosis? A Clostridium difficile colitis B Microscopic colitis C Tropical sprue D Ulcerative colitis

B Microscopic colitis is characterized by chronic watery diarrhea without bleeding; the diagnosis must be made by histologic examination of colonoscopic biopsy specimens. This patient most likely has microscopic colitis, which is characterized by chronic watery diarrhea without bleeding. There are two types of microscopic colitis: collagenous colitis and lymphocytic colitis. The average age of onset for collagenous colitis is in the sixth decade of life and it tends to affect more women than men. The average age of onset for lymphocytic colitis is in the seventh decade of life, and women seem to be affected slightly more often than men. The cause of microscopic colitis is unknown. One theory is that the use of NSAIDs may contribute to the development of the disorder. Another theory is that it is caused by an autoimmune response. Colonoscopy in affected patients is grossly normal; to make a diagnosis, several biopsies must be taken from the colon. In collagenous colitis, biopsy specimens show more than normal amounts of collagen beneath the lining of the colon. In lymphocytic colitis, the specimen may also show an increased number of lymphocytes. Loperamide, diphenoxylate, and bismuth subsalicylate, either alone or in combination, are effective and well-tolerated when used as initial therapy. Ulcerative colitis is characterized by bloody diarrhea associated with rectal discomfort, which the patient does not have. Fever, weight loss, tachycardia, dehydration, and significant abdominal tenderness or rebound indicate more severe disease. The patient has not taken any antibiotics and does not have other established risk factors for Clostridium difficile colitis, including recent hospitalization, advanced age, and severe illness. She has not traveled out of the country recently, and therefore, is not at risk for tropical sprue.

Gastro 35 A 37-year-old woman is evaluated for diffuse musculoskeletal pain at her 1-year follow-up after a Roux-en-Y gastric bypass for medically complicated obesity. Before surgery she had type 2 diabetes mellitus for which she required insulin therapy; after surgery the diabetes was manageable with diet modification alone. She also had hyperlipidemia, for which she took a statin, but after surgery her hyperlipidemia resolved and she discontinued statin therapy 6 months ago. She has lost 36.4 kg (80 lb) since surgery but she thinks that her weight has stabilized over the past few months. She has two to three bowel movements of well-formed stool a day. She has some diffuse musculoskeletal pain but otherwise feels well. Her medical history includes the gastric bypass, a cholecystectomy, diet-controlled type 2 diabetes mellitus, and hypothyroidism. Her medications include levothyroxine, a multivitamin containing iron, vitamin B12 by injection, and over-the-counter calcium and vitamin D. On physical examination, vital signs are normal; the BMI is 29. There is redundant skin over the arms and trunk and mild thyromegaly. There is no jaundice or scleral icterus. Abdominal examination reveals no tenderness or hepatomegaly; there is normal muscle strength with no focal muscle tenderness. Laboratory studies show a serum alkaline phosphatase of 314 U/L; all other tests, including complete blood count, aminotransferases, bilirubin, γ-glutamyltranspeptidase, serum thyroid-stimulating hormone, free thyroxine, and calcium, are normal. Which of the following is the most appropriate next step in the evaluation of this patient? A Anti-smooth muscle antibody test B Measurement of 25-hydroxyvitamin D C Measurement of serum creatine kinase D Measurement of triiodothyronine E Ultrasonography of the liver

B Nutritional deficiencies are common after bariatric surgery, and patients typically need ongoing supplementation with iron, vitamin B12, calcium, and vitamin D. Patients who have bariatric surgery are at risk for various vitamin deficiencies. After such surgery, most patients require ongoing therapy with a multivitamin containing iron, along with vitamin B12, calcium, and vitamin D. Other requirements include folic acid and vitamin A, which are often adequately replaced through the use of a multivitamin, although additional supplementation may be required. In addition, thiamine should be given for the first 6 postoperative months. This patient is taking calcium and vitamin D supplementation, but over-the-counter vitamin D supplementation may not be of sufficient quantity to sustain the requirements in this patient. Her diffuse musculoskeletal discomfort and elevated serum alkaline phosphatase are likely the result of vitamin D deficiency, with resultant osteomalacia, and, therefore, serum 25-hydroxyvitamin D should be measured. Anti-smooth muscle antibody is a serologic marker of autoimmune hepatitis; although this disorder may produce abnormal liver chemistry studies, it usually is considered a hepatitic condition with predominant elevation of the aminotransferases rather than cholestasis. Serum creatine kinase may be a reasonable test in a patient who has myalgias while taking a statin, but this patient's musculoskeletal pain began months after she stopped taking the drug. Although thyroid dysregulation can cause both myalgias and abnormal liver chemistry results, the patient's thyroid-stimulating hormone and thyroxine levels are within normal limits while she is taking thyroid hormone replacement; therefore, further thyroid function testing is unnecessary. Ultrasonography of the liver would be the next best step in a patient who has serum chemistry results indicative of cholestasis, but with alkaline phosphatase being the only test elevated and without clinical features of biliary disease, nonhepatic causes of an elevated alkaline phosphatase need to be considered.

Gastro 85 A 57-year-old man is evaluated for elevated aminotransferase concentrations detected on evaluation for fatigue. The patient has a history of osteoarthritis, type 1 diabetes mellitus, and hypertension, and his medications are ibuprofen, glargine and lispro insulins, lisinopril, simvastatin, and low-dose aspirin. The patient does not use illicit drugs, alcohol, or tobacco; he has never had a blood transfusion. His mother died of heart failure and a maternal aunt died of cirrhosis of unknown cause. On physical examination, the patient is afebrile; the blood pressure is 135/81 mm Hg, the pulse rate is 53/min, and the respiration rate is 14/min. The BMI is 27. Cardiopulmonary examination is normal; there are no stigmata of chronic liver disease. Laboratory studies: Hemoglobin 14 g/dL (140 g/L) Glucose (fasting) 145 mg/dL (8.0 mmol/L) Bilirubin (total) 1.2 mg/dL (20.5 µmol/L) Bilirubin (direct) 0.4 mg/dL (6.8 µmol/L) Aspartate aminotransferase 75 U/L Alanine aminotransferase 88 U/L Alkaline phosphatase 120 U/L Albumin 4.2 g/dL (42 g/L) Ferritin 1267 ng/mL (1267 mg/L) Transferrin saturation 75% Serologic tests for viral hepatitis, antinuclear antibody, anti-smooth muscle antibody, and antimitochondrial antibody are negative. Chest radiography and electrocardiography are normal. Which of the following is the most appropriate next step in the management for this patient? A Deferoxamine therapy B Genetic testing for mutations of the HFE gene C MRI of the abdomen D Phlebotomy

B Patients who have abnormal liver chemistry tests and an abnormal transferrin saturation should be screened for hereditary hemochromatosis. Although the classic description of hereditary hemochromatosis includes the findings of cirrhosis, type 1 diabetes mellitus, and skin hyperpigmentation ("bronze diabetes"), it is now uncommon for patients to present with this triad because of earlier detection of the disease. The most common presenting symptoms today are fatigue, impotence, and destructive arthropathy, characteristically of the second and third metacarpophalangeal joints. However, many patients are asymptomatic and are diagnosed after abnormal results are noted during routine laboratory testing. Patients who have abnormal liver chemistry tests and an abnormal transferrin saturation should be screened for hereditary hemochromatosis, an autosomal recessive disorder in which mutations in the HFE gene cause increased intestinal absorption of iron; the disorder is a common genetic disorder especially in the white population. The genetic mutation is located on chromosome 6 and is most commonly associated with the C282Y homozygous state or the compound heterozygote C282Y/H63D. This patient not only has elevated transferrin saturation but a serum ferritin greater than 1000 ng/mL (1000 mg/L) which has been shown to be predictive of advanced liver fibrosis in patients with hereditary hemochromatosis. Furthermore, he has diabetes mellitus and arthritis. In addition, his family history is also suggestive of the diagnosis. Generally, a transferrin saturation of greater than 55% in a man and greater than 50% in a woman should raise suspicion for the disorder. Identifying an affected patient is also important for family members who then should be screened for the disorder as well, generally starting with a simple transferrin saturation and evaluating further those in whom an elevated saturation is found. Identifying patients before they develop disease is important because it can generally be managed with therapeutic phlebotomy until iron stores are depleted. Therapy with deferoxamine, an iron chelator, is indicated only for patients who cannot tolerate phlebotomy, such as those with significant anemia. Iron chelation therapy is significantly less effective at removing iron than is phlebotomy and must be administered by subcutaneous or intravenous infusion. MRI is not diagnostic for hemochromatosis.

Gastro 18 A 25-year-old man is evaluated for a 6-month history of eight loose, nonbloody stools a day; he also has abdominal pain and small-joint arthritis. The patient has a 5-year history of Crohn disease, initially treated with corticosteroids but then maintained on azathioprine therapy. Small-bowel radiographic series shows no stricturing disease but some active jejuno-ileitis. Azathioprine metabolites were recently measured and were found to be at therapeutic concentration. Which of the following is the most appropriate additional therapy for this patient? A 5-Aminosalicylate B Anti-tumor necrosis factor α-inhibitor C Calcineurin inhibitor D Corticosteroids

B Patients whose Crohn disease becomes refractory to immunomodulator therapy should be given anti-tumor necrosis factor-α inhibitor therapy. The patient's Crohn disease had been well controlled on therapy with the immunomodulator azathioprine but is no longer responding; the small-joint arthritis may also be a complication of azathioprine. The next agent that should be used is the biologic agent infliximab an anti-tumor necrosis factor-α inhibitor. This potent biologic agent can bring the disease into remission quickly while sparing the effects of systemic corticosteroids. However, there is an increased risk of lymphoma and solid tumors associated with infliximab therapy. Recently it has been reported that combined infliximab and azathioprine is associated with an increased risk of hepatosplenic T-cell lymphoma in pediatric patients. Therefore, combination therapy is not preferred in patients who are receiving fixed-interval infliximab (every 8 weeks). Azathioprine dosage should be tapered and therapy with this agent eventually discontinued. Early use of infliximab has been shown to be more effective than conventional management with corticosteroids for induction of remission. Corticosteroids are not used as maintenance therapy in Crohn disease primarily because of their numerous short- and long-term complications. The patient's disease has now failed to respond to the steroid-sparing immunomodulator, and there is no role for corticosteroids. Calcineurin inhibitors have not been shown to be efficacious in Crohn disease. Mesalamines are not effective in immunomodulator-refractory Crohn disease.

Gastro 42 A 51-year-old man is evaluated for an 8-month history of mid-epigastric pain that is worse after eating, six to eight bowel movements a day usually occurring after a meal, and loss of 6.8 kg (15 lb) over the past 6 months. The patient drinks six to eight cans of beer a day. He takes no medications. On physical examination, the patient is thin (BMI 21) and has normal bowel sounds, mid-epigastric tenderness, but no evidence of hepatosplenomegaly or masses. Rectal examination reveals brown stool that is occult blood negative. The remainder of the examination is normal. Plain radiograph of the abdomen shows a normal bowel gas pattern and is otherwise normal. Laboratory studies: Leukocyte count 6800/µL (6.8 × 109/L) Platelet count 69,000/µL (69 × 109/L) Fasting plasma glucose 104 mg/dL (5.77 mmol/L) Aspartate aminotransferase 191 U/L Alanine aminotransferase 82 U/L Amylase 122 U/L Lipase 289 U/L Which of the following tests is most likely to establish the diagnosis in this patient? A Colonoscopy B CT scan of the abdomen C Measurement of serum antiendomysial antibodies D Stool for leukocytes, culture, ova, and parasites

B Patients with chronic pancreatitis present with abdominal pain and in more severe cases with malabsorption and endocrine insufficiency. This patient has chronic pancreatitis secondary to alcohol abuse, which has resulted in malabsorption. The three classic findings in chronic pancreatitis are abdominal pain that is usually mid-epigastric, postprandial diarrhea, and diabetes mellitus secondary to pancreatic endocrine insufficiency. Malabsorption occurs in patients with chronic pancreatitis when approximately 80% of the pancreas is destroyed. Malabsorption presents with diarrhea and steatorrhea, weight loss, and deficiencies of fat-soluble vitamins because the damaged pancreatic gland is no longer producing the pancreatic exocrine enzymes to absorb food. Patients with a typical presentation may not need additional testing. However, most patients with chronic pancreatitis have only nonspecific abdominal pain and require diagnostic radiographic imaging studies. The presence of pancreatic calcifications on radiographs confirms the diagnosis. Plain films of the abdomen will show pancreatic calcifications in approximately 30% of patients. Most patients, however, require abdominal CT scans, which are able to detect pancreatic calcification in up to 90% of patients. CT scanning can also exclude other causes of pain. Radiographic evidence of pancreatic ductal dilation, pseudocysts, or mass lesions may also help identify the cause of pain and determine the type of therapy. Antiendomysial antibodies are a marker for celiac disease, which is unlikely in this patient with an evident history of pancreatic malabsorption. Although colonoscopy is indicated as a screening tool for average risk asymptomatic patients beginning at the age of 50 years and for patients with a change in bowel habits and weight loss, this patient's history suggests pancreatic malabsorption and colonoscopy is less likely than abdominal CT scan to confirm the diagnosis. Stool studies are appropriate for determining the cause of an acute infectious diarrhea, but this patient has had diarrhea for 8 months and infectious diarrhea is not usually associated with such a degree of weight loss.

Gastro 81 A 63-year-old man is evaluated for a 3-month history of a 9-kg (20-lb) weight loss and 1 month of painless jaundice and decreased appetite. The patient was diagnosed with ulcerative colitis at age 21 years and underwent a colectomy with ileostomy. He was diagnosed with primary sclerosing cholangitis 2 years ago. On physical examination, the patient is thin and cachectic; he is afebrile. The blood pressure is 130/75 mm Hg, the pulse rate is 80/min, and the respiration rate is 14/min; the BMI is 22. Examination reveals scleral icterus, but no ascites or right upper quadrant tenderness on deep palpation; Murphy sign was not elicited. Laboratory studies: Hemoglobin 14.5 g/dL (145 g/L) Leukocyte count 3400/µL (3.4 × 109/L) with normal differential Platelet count 225,000/µL (225 × 109/L) Bilirubin (total) 8.0 mg/dL (136.8 µmol/L) Bilirubin (direct) 3.6 mg/dL (61.56 µmol/L) Aspartate aminotransferase 120 U/L Alanine aminotransferase 145 U/L Alkaline phosphatase 456 U/L CA 19-9 1200 U/mL (1200 kU/L) α-Fetoprotein 9.0 ng/mL (9.0 µ/L) Magnetic resonance cholangiopancreatography shows dilated intrahepatic bile ducts with new irregular strictures at the common hepatic duct with a periductal irregular mass surrounding the stricture; the pancreatic duct has a normal caliber. The gallbladder appears normal without edema or wall thickening. Which of the following is the most likely diagnosis? A Acute cholecystitis B Cholangiocarcinoma C Choledocholithiasis D Progressive primary sclerosing cholangitis

B Patients with primary sclerosing cholangitis are at risk for developing cholangiocarcinoma (lifetime prevalence of 10% to 30%). Primary sclerosing cholangitis is a chronic cholestatic liver disease of unknown cause that is characterized by progressive bile duct destruction and may lead to secondary biliary cirrhosis. Up to 80% of patients with primary sclerosing cholangitis have an inflammatory bowel disease (most often ulcerative colitis), although less than 5% develop primary sclerosing cholangitis. Patients with primary sclerosing cholangitis are at risk for developing cholangiocarcinoma (lifetime prevalence of 10% to 30%). Patients with advanced primary sclerosing cholangitis and cirrhosis are also at risk for developing hepatocellular carcinoma. This patient has a history of primary sclerosing cholangitis and now presents with onset of painless jaundice and weight loss, which should raise the suspicion for cholangiocarcinoma. The detection of an irregular stricture in the hepatic duct further raises concern that a malignancy has developed. Additional pertinent findings are the elevated CA 19-9 level, which although not specific to cholangiocarcinoma, in the correct clinical setting is helpful in raising the likelihood of the diagnosis. The patient has no findings suggesting acute cholecystitis on physical examination, the leukocyte count is normal, and on imaging the gallbladder appears normal without any wall thickening or edema, which would be expected findings in a patient with acute cholecystitis. Furthermore, biliary ductal dilatation would be uncommon in cholecystitis. Choledocholithiasis usually presents with biliary colic in the absence of weight loss. The lack of cholelithiasis, which does not entirely exclude this diagnosis, in the presence of the other clinical findings makes it less likely. Furthermore, a filling defect in the ducts would be expected rather than an irregular stricture, which makes malignancy more likely. Progressive primary sclerosing cholangitis, although possible given the patient's disease, should not exclude the more likely diagnosis of cholangiocarcinoma in this patient who now presents with painless jaundice, weight loss, an irregular new stricture, and a very elevated CA 19-9.

Gastro 23 A 55-year-old man is hospitalized for a 2-week history of jaundice and altered mental status. The patient has a 10-year history of alcohol dependence and has failed several attempts to stop drinking. His family reports that he had been drinking heavily every day until about 3 weeks ago. On physical examination, the patient is confused and lethargic; the temperature is 38.0 °C (100.0 °F), the blood pressure is 90/60 mm Hg, the pulse rate is 120/min, and the respiration rate is 30/min. The BMI is 24. Examination reveals scleral icterus. There is no guarding on palpation of the abdomen. The liver edge is tender and easily palpable 3 cm below the right costal margin. There is no ascites, edema, or evidence of bleeding. Laboratory studies: Leukocyte count 17,000/µL (17 × 109/L) Platelet count 103,000/µL (103 × 109/L) Prothrombin time 26.2 s INR 4.0 Bilirubin (total) 37.0 mg/dL (632.7 µmol/L) Bilirubin (direct) 17.0 mg/dL (290.7 µmol/L) Aspartate aminotransferase 98 U/L Alanine aminotransferase 50 U/L Alkaline phosphatase 230 U/L Albumin 2.0 g/dL (20 g/L) Ammonia 110 µg/dL Chest radiograph is normal. Ultrasonography shows an enlarged, fatty liver, with no nodules, ascites, pericholecystic fluid, or bile duct dilatation. Blood and urine cultures are negative. In addition to enteral nutrition, which of the following is the most appropriate management for this patient? A Ceftriaxone B Methylprednisolone C Fresh frozen plasma D Liver transplantation

B Patients with severe alcoholic hepatitis, as defined by a discriminant function score of 32 or more, benefit from corticosteroid therapy. This patient has severe alcoholic hepatitis. Excessive alcohol intake may cause liver disease directly or may increase the risk of an unfavorable outcome in patients with pre-existing liver disease. This patient with chronic alcohol abuse has a history of recent heavy alcohol use, elevated serum aspartate aminotransferase (AST) and alanine aminotransferase values (usually greater than 300 U/L) and a serum aspartate aminotransferase concentration that is greater than the alanine aminotransferase concentration in roughly a 2 to 1 ratio, elevated alkaline phosphatase concentration, jaundice, coagulopathy, and encephalopathy. Moreover, other causes of acute and chronic liver disease have been excluded. The severity of the presentation and poor prognosis are underscored by the presence of the very high serum bilirubin concentration, coagulopathy, and encephalopathy. The mortality risk in this setting is calculated by the discriminant function as follows: (4.6 × [prothrombin time − control prothrombin time]) + serum bilirubin. A discriminant function score of greater than 32 identifies patients with a 50% mortality rate within 30 days and has been used to identify patients who have a survival benefit from corticosteroid therapy. In addition, nutrition therapy has been shown to improve survival in severely malnourished hospitalized alcoholic patients. The presentation of alcoholic hepatitis may resemble infection because of the fever and leukocytosis; however, this patient's chest radiograph and blood and urine cultures were negative. In the absence of active bleeding, fresh frozen plasma is not indicated. Although many patients with alcoholic hepatitis recover with appropriate therapy, it would be inappropriate to offer liver transplantation to an alcoholic patient who is not involved in rehabilitation counseling. Some transplant programs require abstinence of at least 6 months. This not only confirms a commitment to abstaining from alcohol but also allows time for improvement of the alcoholic hepatitis to the point that transplantation may not be needed. Antibiotic therapy has no role in alcoholic hepatitis. Fresh frozen plasma is not a priority in a patient with alcoholic hepatitis and a coagulopathy unless active bleeding is present.

Gastro 64 A 76-year-old man is evaluated in the emergency department after having passed a large amount of red and maroon blood per rectum. After this episode, the patient felt dizzy and stumbled but did not lose consciousness or injure himself. In the ambulance on the way to the emergency department, he passed more blood, and the blood pressure was 100/60 mm Hg and the pulse rate was 110/min. He has not had abdominal pain, nausea, vomiting, fever, or weight loss. He had a colonoscopy 1 year ago that showed a benign polyp and diverticulosis. His medical history includes hypertension and hypercholesterolemia, and his medications are hydrochlorothiazide, lisinopril, simvastatin, and aspirin. On physical examination, the patient is pale; the blood pressure is 105/65 mm Hg and the pulse rate is 100/min. Abdominal examination is normal; there is dried blood in the perianal area, and rectal examination reveals normal tone but fresh blood on the examination glove. Laboratory studies reveal hemoglobin 10.1 g/dL (101 g/L) and normal biochemical studies, including blood urea nitrogen. Leukocyte count is 5600/µL (5.6 × 109/L) and platelet count is 348,000/µL (348 × 109/L); prothrombin time and activated partial thromboplastin time are normal. Which of the following is the most appropriate next step in the management of this patient? A Colonoscopy B Esophagogastroduodenoscopy C Intravenous access D Placement of a nasogastric tube with lavage E Technetium-labeled red blood cell scan

C Diverticular bleeding is one of the most common sources of lower gastrointestinal bleeding, but diagnosis and therapy via colonoscopy are secondary to volume resuscitation and achieving hemodynamic stability. The likely source of gastrointestinal bleeding in this patient is lower, with diverticulosis and vascular ectasia being most likely. Although a brisk upper gastrointestinal bleeding source is possible, generally an elevated blood urea nitrogen level and symptoms referable to the upper gastrointestinal tract would be seen. The patient's recent colonoscopy and lack of other symptoms and large-volume bleeding make a neoplasm unlikely. Colonic ischemia generally presents with abdominal pain and only a small amount of bleeding. Regardless of the source of bleeding at this point in the emergency department, however, the first rule of management is to achieve hemodynamic stability. This patient is volume-depleted and not hemodynamically stable. Therefore, the most appropriate next step is to ensure proper intravenous access, generally via two large-bore peripheral catheters or a central line for volume repletion with crystalloid fluids and blood products as necessary. A nasogastric tube may be considered after volume resuscitation if an upper gastrointestinal bleeding source were suspected, but given his lack of upper gastrointestinal symptoms and normal blood urea nitrogen concentration, this is unlikely. Further, if an upper gastrointestinal source were highly suspected, an esophagogastroduodenoscopy should be performed regardless of the lavage results, which have a high false-negative rate. Although a colonoscopy is the diagnostic test of choice to evaluate for diverticular bleeding, vascular ectasias, neoplasms, internal hemorrhoids, and other possible sources of lower gastrointestinal bleeding, again this should not occur before volume resuscitation. A bleeding scan would be indicated only if endoscopic evaluation is not revealing and the patient continues to bleed. Up to 90% of diverticular bleeding resolves spontaneously. If a bleeding diverticulum is detected on colonoscopy, it can be treated with epinephrine injection and/or thermal coagulation.

Gastro 67 A 51-year-old man is evaluated for a 6-month history of nausea, bloating, and diarrhea. He typically has nausea and bloating within an hour after eating and then has an explosive bowel movement; during the episode he is tachycardic. He feels reasonably well between episodes and at night. The episodes are worst in the morning even though he consumes only coffee and a breakfast drink. High-fat foods are better tolerated and do not seem to provoke symptoms. He has no abdominal pain, vomiting, or weight loss. His medical history includes a distal gastrectomy 9 months ago for a perforated NSAID-induced peptic ulcer. He has no other medical problems and takes no medications. On physical examination, the BMI is 24 and vital signs are normal. He has hyperactive bowel sounds. The heart, lungs, and abdomen are normal. Laboratory studies, including complete blood count and routine biochemistry tests, are normal. Esophagogastroduodenoscopy shows changes of a distal gastrectomy with a widely patent anastomosis. The mucosa is normal throughout, and biopsy specimens and aspirates of the small bowel are normal. Which of the following is the most likely diagnosis? A Carcinoid syndrome B Chronic mesenteric ischemia C Dumping syndrome D Gastrinoma E Irritable bowel syndrome

C Early dumping syndrome is associated with nausea, bloating, diarrhea, and tachycardia that often occur within 30 minutes after a meal. This patient has recently undergone a distal gastrectomy and now has classic features of early dumping syndrome. Patients with early dumping syndrome often have diarrhea, nausea, bloating, and tachycardia that occur within 30 minutes of eating as a result of hyperosmolar substances being rapidly emptied into the small bowel in the absence of the pylorus; therefore, carbohydrates and liquid nutrient drinks are often poorly tolerated as this case demonstrates. Late dumping syndrome occurs 1 to 3 hours after a meal and is characterized by neuroglycopenic symptoms. Although carcinoid syndrome can be associated with diarrhea, flushing, and tachycardia, the diarrhea is often secretory in nature and continuous, including nocturnal episodes, unlike the features in this patient, which seem to be osmotically driven. Patients with chronic mesenteric ischemia also tend to have postprandial symptoms, but abdominal pain is often the predominant feature associated with weight loss secondary to fear of eating (sitophobia), neither of which was present in this patient. A gastrinoma should be considered in a patient with recurrent or complicated peptic ulcer disease that cannot be explained by NSAID use (as in this patient) or Helicobacter pylori infection. Patients with a gastrinoma usually have diarrhea secondary to fat malabsorption because the low pH from excess acid production inactivates pancreatic lipase. This patient notes his symptoms are actually better on a higher-fat diet, which would be more compatible with dumping syndrome because the food will have a longer phase of gastric digestion given that ingested fat can slow gastric emptying. Although irritable bowel syndrome is common in the general population, other diagnoses need to be considered first in this middle-aged man with new-onset symptoms following a surgical procedure.

Gastro 37 A 32-year-old woman is evaluated for a 5.2-cm mass in the right lobe of the liver detected during evaluation for possible appendicitis, which was ruled out. The patient has a history of migraine and a 6-year history of irregular menses, and her medications include sumatriptan as needed and an oral contraceptive pill. On physical examination, the patient is afebrile; the blood pressure is 110/65 mm Hg, the pulse rate is 80/min, and the respiration rate is 14/min. The BMI is 21. There are no palpable abdominal masses; bowel sounds are normal, and Murphy sign is not present. Laboratory tests are all normal. CT scan of the abdomen shows a well-circumscribed lesion in the right lobe of the liver with an enhancing central scar; the rest of the liver appears to be normal, and there is no intra- or extrahepatic duct dilatation. Which of the following is the most appropriate management for this patient? A Discontinuation of oral contraceptive pills B Evaluation for liver transplantation C Observation D Referral for surgical resection

C Focal nodular hyperplasia is the most common nonmalignant hepatic tumor not of vascular origin and is usually stable and uncomplicated requiring no immediate management. This young woman has focal nodular hyperplasia, the most common nonmalignant hepatic tumor that is not of vascular origin. This tumor occurs more commonly in women and usually causes no symptoms unless it is very large, in which case an affected patient may experience discomfort from mass effect on nearby organs. The tumors are caused by aberrant blood supply to an area of the liver and classically are differentiated by the appearance of a central scar. Unlike (verschieden) adenomas, focal nodular neoplastic tumors are not estrogen-sensitive, and therefore, the patient does not need to discontinue oral birth control. In addition, these tumors do not have a malignant potential and do not need to be resected unless they are large and exert mass effect on other organs or cause focal ductal dilatation. This patient, therefore, does not need to be referred for resection. Because this lesion is radiographically consistent with focal nodular hyperplasia rather than hepatocellular cancer, the patient does not have to be referred for evaluation for liver transplantation. Focal nodular hyperplasia is usually stable and uncomplicated, and therefore, no immediate management is required for this patient.

Gastro 31 A 35-year-old woman with a history of chronic hepatitis C virus infection is evaluated 4 weeks after starting therapy with pegylated interferon and ribavirin. The patient also has a history of hypothyroidism, and her medications include interferon, ribavirin, and levothyroxine. The patient's mother has rheumatoid arthritis and hypothyroidism. On physical examination, the patient is afebrile; the blood pressure is 100/78 mm Hg, the pulse rate is 65/min, and the respiration rate is 12/min. The BMI is 26. There is scleral icterus; there is no edema, ascites, or spider angioma. Laboratory studies: 1 Month Ago Current Bilirubin (total, mg/dL) 1.2 (20.5 µmol/L) 4.0 (68.4 µmol/L) Bilirubin (direct, mg/dL) 0.3 (5.1 µmol/L) 2.3 (39.3 µmol/L) Aspartate aminotransferase (U/L) 37 770 Alanine aminotransferase (U/L) 39 890 Alkaline phosphatase (U/L) 90 167 Albumin (g/dL) 4.0 (40 g/L) 3.6 (36 g/L) Antinuclear antibody Negative Titer 1:80 Anti-smooth muscle antibody Negative Titer 1:640 Antimito-chondrial antibody Negative Negative Hepatitis C virus RNA 580,000 copies/mL 540,000 copies/mL Ultrasonography of the liver is normal. Which of the following is the most appropriate next step in the management of this patient? A Continue interferon and ribavirin at current dosages B Continue interferon and hold ribavirin C Discontinue interferon and ribavirin D Discontinue interferon and ribavirin for 1 week

C Interferon therapy can precipitate autoimmune hepatitis, and therapy should be discontinued in affected patients. This patient is experiencing a flare of autoimmune hepatitis precipitated by interferon therapy. Such a flare can occur in patients with quiescent autoimmune hepatitis or as new-onset disease induced by interferon therapy. Other autoimmune diseases can develop or be exacerbated during interferon therapy, including rheumatoid arthritis, sarcoidosis, dermatitis, and type 1 diabetes mellitus. Accordingly, interferon therapy should be used cautiously in patients with autoimmune disease and is contraindicated in patients with pre-existing autoimmune hepatitis. This patient has worsening liver disease as well as established hypothyroidism; therefore, the interferon and ribavirin therapy should be discontinued and the patient monitored closely. If the autoimmune hepatitis does not stabilize, treatment for this disorder will have to be considered. This patient should not be rechallenged with hepatitis C therapy because it is likely to cause worsening liver disease. Decreasing the dosage of the interferon and ribavirin is not an option, because the autoimmune response is not dose-related. Although some patients receiving therapy for hepatitis C experience a flare of hepatitis before clearing the virus, this patient has persistent viral replication, and given the newly positive antinuclear antibody and anti-smooth muscle antibody test results suggestive of autoimmune hepatitis and personal and family history of autoimmune disorders, therapy should be discontinued to avoid worsening hepatic decompensation.

Gastro 70 A 63-year-old woman is evaluated for heartburn, sour regurgitation, and substernal chest pain, symptoms that have occurred several times a week for the past 5 years. She does not have fever, nausea, vomiting, or weight loss. The patient has a history of hypertension, and her medications include an angiotensin receptor blocker and hydrochlorothiazide. On physical examination, vital signs are normal; BMI is 28.5. The abdomen is soft without tenderness or distention; bowel sounds are normal. Which of the following is the most appropriate management of this patient? A Ambulatory esophageal pH monitoring B Esophageal manometry C Esophagogastroduodenoscopy D Oral calcium channel blocker therapy

C Long-standing symptoms of gastroesophageal reflux disease should prompt an esophagogastroduodenoscopy to evaluate for Barrett esophagus. Long-standing gastroesophageal reflux is associated with risk for Barrett esophagus, a premalignant condition. Barrett esophagus is diagnosed endoscopically: the presence of columnar-appearing mucosa in the distal esophagus suggests the disorder, and it is confirmed histologically by detection of specialized intestinal epithelium. The presence of Barrett esophagus increases the risk for esophageal adenocarcinoma by approximately 30- to 100-fold. Endoscopic and histologic surveillance of patients with Barrett esophagus for dysplastic changes is recommended, although the frequency of surveillance is controversial. Ambulatory pH monitoring is the most sensitive procedure for detection of increased esophageal acid exposure, but such a finding would be expected in this patient from the history alone. Esophageal manometry is useful in elucidating the cause of noncardiac chest pain or esophageal dysphagia. An oral calcium channel blocker may be prescribed in spastic motility disorders of the esophagus, but this patient's symptoms are typical of gastroesophageal reflux disease.

Gastro 92 A 67-year-old woman is evaluated for a 9-month history of postprandial abdominal pain. The pain is diffuse but slightly more epigastric and periumbilical. She has not had heartburn, nausea, vomiting, or a change in bowel movements. She avoids eating to minimize episodes of pain and has lost 9 kg (20 lb). The patient had a myocardial infarction 2 years ago and underwent carotid endarterectomy 5 years ago. She has a 30-pack-year smoking history and still smokes. Her medications are simvastatin, lisinopril, metoprolol, and aspirin. On physical examination, the blood pressure is 140/90 mm Hg; other vital signs are normal. BMI is 20. There is an epigastric bruit but no abdominal tenderness, masses, or organomegaly. Laboratory studies reveal hemoglobin 13.5 g/dL (135 g/L); amylase, metabolic profile, and liver chemistry tests are normal. Abdominal ultrasonography, CT scan of the abdomen and pelvis, and esophagogastroduodenoscopy are normal. The patient had a normal colonoscopy one and a half years ago. Which of the following is the most appropriate next step in the evaluation of this patient? A Colonoscopy B Gastric emptying study C Magnetic resonance angiography of the abdomen D Small-bowel radiographic series

C Magnetic resonance arteriography is a highly sensitive test to diagnose chronic mesenteric ischemia. This patient has classic signs and symptoms of chronic mesenteric ischemia: postprandial abdominal pain associated with sitophobia (fear of eating) and weight loss in the setting of vascular disease and a history of smoking. The clinical history is an essential part of the diagnosis of the disorder. Other possible causes of her abdominal pain have been evaluated, including biliary disease, peptic ulcer disease, pancreatitis, and malignancy, by the ultrasonography, upper endoscopy, CT scan, and laboratory tests. Although the gold standard for evaluating stenoses of the mesenteric vessels is traditional angiography, CT or magnetic resonance angiography both have high sensitivity in the diagnosis of chronic mesenteric ischemia. Two vessels are involved in 90% of affected patients, and 50% have significant stenosis of all three vessels (celiac artery, superior mesenteric artery, inferior mesenteric artery). Doppler ultrasonography may also be useful in evaluating these vessels. Although colonoscopy would be indicated in this patient to screen for colon cancer if she had not had one within the last 10 years, this test should not be prioritized at this time. Colonoscopy is normal in chronic mesenteric ischemia as it does not involve the colon. A small bowel series would not be helpful in this setting because it evaluates only the lumen and not the vasculature of the small intestine. A gastric emptying study measures the half-time for the stomach to empty its contents and can be helpful in diagnosing gastroparesis; although the symptoms of gastroparesis and chronic mesenteric ischemia overlap, given the patient's history of vascular disease and extreme weight loss as well as the importance of timely diagnosis to avoid a superimposed life-threatening acute ischemic event, a gastric emptying study would not be the most appropriate next test.

Gastro 74 A 37-year-old woman is evaluated for a 4-month history of recurrent bloody stool described as bright red blood on the outside of the stool and on the toilet paper. She does not have blood mixed in the stool. She has no pain on defecation, and no diarrhea, constipation, fever, or chills. She has been taking a fiber supplement for the past month without effect. The patient is otherwise healthy and takes no medications. There is no family history of colon cancer. On physical examination, vital signs are normal; BMI is 19.5. Abdominal and rectal examinations are normal; stool is positive for occult blood. Laboratory studies, including complete blood count, are normal. Which of the following is the most appropriate next step in the management of this patient? A Colonoscopy now B Continued high-fiber diet C Flexible sigmoidoscopy D Oral prednisone therapy

C Patients at low risk of colonic neoplasia may undergo evaluation for hematochezia with flexible sigmoidoscopy rather than colonoscopy. The strongest clinical predictor of colonic neoplasia is the presence of blood in the stool. The likelihood of identifying colon polyps or cancer increases with age, and therefore, a patient younger than 40 years with bleeding should undergo flexible sigmoidoscopy as the first step in evaluation. The likelihood of proximal colonic neoplasia is low in this setting and, therefore, the risks associated with complete colonoscopy are not warranted. Oral prednisone would not be indicated. A high fiber diet may be helpful for treatment for benign anorectal sources of bleeding; however, further evaluation to exclude neoplasia is indicated because of the multiple occurrences of bleeding.

Gastro 14 A 48-year-old man is evaluated 3 weeks after presenting with epigastric pain and early satiety with no weight loss. He underwent an esophagogastroduodenoscopy that showed thickened and nodular gastric folds in the body of the stomach with patchy erosions in the antrum. Biopsy specimens of the nodularity showed low-grade B-cell lymphoma; specimens of the antrum showed chronic active gastritis and no Helicobacter pylori. Endoscopic ultrasonography showed no invasion through the wall of the stomach; CT scan of the abdomen showed only gastric wall thickening. The patient's medical history includes intermittent heartburn and occasional headache; his medications include as-needed antacids and an NSAID. On physical examination, vital signs are normal. Examination of the abdomen reveals mild epigastric tenderness but no hepatosplenomegaly or lymphadenopathy. Complete blood count and serum chemistry tests are normal. Which of the following is the most appropriate next step in the management of this patient? A Evaluation for HIV infection B Initiation of systemic chemotherapy C Helicobacter pylori stool antigen test D Measurement of serum tissue transglutaminase antibody E Surgical resection of the tumor

C Patients with mucosa-associated lymphoid tissue (MALT) lymphoma should be tested for Helicobacter pylori infection. This patient has a mucosa-associated lymphoid tissue (MALT) lymphoma, a low-grade B-cell lymphoma arising from the mucosal lymphoid tissue of the gastrointestinal tract. Half of MALT lymphomas are indolent, and most are located within the stomach. MALT lymphomas are thought to arise from clonal lymphocyte expansion in patients with chronic active gastritis secondary to Helicobacter pylori infection; eradication therapy for H. pylori infection alone results in lymphoma regression in up to 80% of affected patients; therefore, testing for H. pylori by stool antigen test is indicated in this patient. Patients with HIV infection are at increased risk for Burkitt lymphoma, which can occur in the gastrointestinal tract; however, these tumors tend to be large and are usually located in the terminal ileum or rectum rather than the stomach and are not associated with chronic active gastritis. Patients with MALT lymphoma who have increased extent of disease as evidenced by ulceration, invasion through the gastric wall, or lymphadenopathy, which were not present in this case, are more likely to require standard chemotherapy for lymphoma. Although intestinal lymphoma is a malignant complication of celiac disease, this form of lymphoma is of T-cell origin (also known as enteropathy-associated T-cell lymphoma), and therefore, measurement of serum tissue transglutaminase antibody is not necessary. Surgical resection would not be required in a patient with early-stage MALToma who may fully respond to H. pylori eradication alone.

Gastro 79 A 64-year-old woman is evaluated in the emergency department for a 3-week history of postprandial right upper quadrant abdominal pain that has become increasingly intense. The pain is now constant and radiates to her right shoulder and is accompanied by fever, nausea, and vomiting. The patient has a history of type 2 diabetes mellitus and obesity, and her medications are metformin, glimepiride, and aspirin. On physical examination, the patient is uncomfortable; the temperature is 38.3 °C (101.0 °F), the blood pressure is 110/65 mm Hg, the pulse rate is 110/min, and the respiration rate is 20/min. The BMI is 42. There is pain on palpation in the right upper quadrant of the abdomen with a Murphy sign. There is no rebound or tenderness or jaundice. Laboratory studies: Leukocyte count 16,000/µL (16 × 109/L) with 20% band forms Bilirubin (total) 2.0 mg/dL (34.2 µmol/L) Aspartate aminotransferase 40 U/L Alanine aminotransferase 120 U/L Alkaline phosphatase 120 U/L Amylase 58 U/L Lipase 36 U/L Ultrasonography shows pericholecystic fluid with echogenic stones in the gallbladder. The wall of the gallbladder is thickened (6 mm), and there is no dilatation of the bile ducts. In addition to antibiotics, which of the following is the most appropriate management for this patient? A CT scan of the abdomen B Endoscopic retrograde cholangiopancreatography C Cholecystectomy D Magnetic resonance cholangiopancreatography

C The classic findings of acute cholecystitis on ultrasonography are pericholecystic fluid and a thickened gallbladder wall of 3 to 4 mm, and ultrasonographic Murphy sign further confirms the diagnosis. The patient likely has acute cholecystitis; she has a history of biliary colic, including pain that radiates to the right shoulder, a Murphy sign elicited on examination, fever, leukocytosis, mild bilirubin and aminotransferase elevation, gallstones and pericholecystic fluid, and thickening of the gallbladder wall on ultrasonography. When ultrasonography reveals gallstones and a positive ultrasonographic Murphy sign, the positive predictive value for acute cholecystitis is 92%. When the patient has the additional findings of gallstones and gallbladder wall thickening (≥3 mm), the positive predictive value is 95% for acute cholecystitis. Ultrasonography can be less sensitive in patients with ascites, hypoalbuminemia, hepatitis, obesity, and heart failure. Surgical cholecystectomy is advisable once gallstones lead to such complications as acute cholecystitis. Early cholecystectomy is associated with a shorter recovery period and fewer complications, such as gangrene and empyema of the gallbladder. While definitive evidence regarding the use of antibiotics in acute cholecystitis is lacking, it would seem prudent to consider using antibiotics in toxic-appearing patients. Endoscopic retrograde cholangiopancreatography would be indicated if there were evidence of bile duct obstruction, which is not present on ultrasonography in this case. Abdominal CT scan would not be useful in this case; it can be useful when complications of acute cholecystitis (such as emphysematous cholecystitis or gallbladder perforation) are suspected. The presence of a sepsis would suggest the diagnosis of gallbladder gangrene and perforation usually occurs after the development of gangrene. MRCP could confirm bile duct obstruction, if it were suspected on ultrasonography.

Gastro 76 A 42-year-old man is evaluated in the hospital for a 1-year history of postprandial abdominal pain that radiates to the back and that is worse after eating and is associated with nausea. He has not had vomiting, weight loss, or change in bowel habits. The patient has had at least five alcohol-containing drinks a day for 20 years; he has reduced his intake in the past year because of continued abdominal pain. On physical examination, vital signs are normal; BMI is 24. There is mild epigastric tenderness with no guarding or rebound and normal bowel sounds. Laboratory studies reveal normal complete blood count, fasting glucose, and liver chemistry tests; amylase is 221 U/L and lipase 472 U/L. Radiography, ultrasonography, and CT scan of the abdomen are normal, as is esophagogastroduodenoscopy. Which of the following is the most appropriate next step in the evaluation of this patient? A Biliary scintigraphy B Colonoscopy C Endoscopic retrograde cholangiopancreatography D Measurement of stool elastase

C Endoscopic retrograde cholangiopancreatography is the most sensitive imaging test for chronic pancreatitis. Establishing the diagnosis of chronic pancreatitis in a patient with early disease can be difficult. No blood or stool tests are currently available for the accurate diagnosis of early chronic pancreatitis. This patient's pain is most likely secondary to chronic pancreatitis with minimally elevated pancreatic enzymes and a history of harmful drinking. The patient has no evidence of exocrine or endocrine insufficiency and thus likely has early chronic pancreatitis. Normal liver enzymes, normal upper endoscopy, and a normal abdominal ultrasonography and CT scan of the abdomen make biliary causes and peptic ulcer disease less likely the cause of pain. Endoscopic retrograde cholangiopancreatography (ERCP) has a sensitivity of nearly 95% for chronic pancreatitis and can show ductal dilation, strictures, and irregularity in both the main duct and its side branches. CT scan of the abdomen has a sensitivity of up to 90% for diagnosing chronic pancreatitis and should be ordered with thin cuts of the pancreas to improve sensitivity. Endoscopic ultrasonography may also be used to diagnose chronic pancreatitis with sensitivities equal to ERCP for moderate and advanced chronic pancreatitis but with lower sensitivity and specificity for mild and early chronic pancreatitis. Magnetic resonance cholangiopancreatography does not have sensitivities or specificities that match ERCP in the diagnosis of chronic pancreatitis at this time and cannot be routinely recommended. Biliary scintigraphy is used to diagnose acute cholecystitis and does not have a role in diagnosing chronic pancreatitis. Stool elastase can be abnormal in patients with more advanced chronic pancreatitis, particularly those who have malabsorption. However, stool elastase has poor sensitivity in patients with early chronic pancreatitis. Colonoscopy has a low yield in patients with upper abdominal pain.

Gstro 63 A 45-year-old man is evaluated 6 months after undergoing liver transplantation for hepatitis C virus-related cirrhosis. The infection had never been treated and the patient was viremic at the time of transplantation. His medications are tacrolimus, mycophenolate, trimethoprim-sulfamethoxazole, and ursodeoxycholic acid. On physical examination, the patient is afebrile; the blood pressure is 100/78 mm Hg, the pulse rate is 65/min, and the respiration rate is 12/min. The BMI is 26. There is a healed midline abdominal incision but no ascites, lower extremity edema, or jaundice. Laboratory studies: Bilirubin (total) 1.4 mg/dL (23.9 µmol/L) Bilirubin (direct) 0.9 mg/dL (15.4 µmol/L) Aspartate aminotransferase 75 U/L Alanine aminotransferase 95 U/L Albumin 3.7 g/dL (37 g/L) Hepatitis C virus RNA >500,000 U/mL Genotype 1A Hepatitis A total antibody Positive Ultrasonography of the liver shows patent vessels, no bile duct dilatation, and normal echotexture. Which of the following is the most appropriate management of this patient? A Discontinue mycophenolate B Pegylated interferon and ribavirin therapy C Liver biopsy D Observation; reevaluation in 3 months

C Evaluation of abnormal liver test results after liver transplantation often requires a liver biopsy especially to differentiate between recurrent hepatitis C virus infection and rejection. This patient has abnormal liver chemistry tests after receiving a liver transplant for hepatitis C virus (HCV)-related cirrhosis, and a liver biopsy is indicated to distinguish between rejection and recurrent HCV infection. HCV is known to recur in the transplanted liver almost immediately after surgery if the patient was viremic at the time of transplant. As a result, some degree of inflammation is to be expected and in some patients early histologic injury as the HCV establishes itself in the new liver. In a small percentage of patients, the HCV can recur very quickly and aggressively a few months after liver transplantation, a condition termed rapidly recurring cholestatic hepatitis C, which carries a poor prognosis. Fortunately, most patients have a much more benign course, with approximately 30% still developing recurrent cirrhosis within 5 to 10 years after transplantation. Identifying those patients who show histologic injury in the liver graft and initiating therapy in these patients is important. Therefore, this patient should undergo a liver biopsy to assess histology and also to rule out other causes of post-transplantation abnormal liver chemistry tests, such as acute cellular rejection, which would clearly be managed quite differently from recurrent HCV infection. Pegylated interferon and ribavirin are effective in clearing the virus in about 30% to 40% of patients with post-transplant hepatitis C and are indicated for patients with progressive histologic changes. Prophylactic treatment has not yet been shown to be effective. Because liver chemistry tests after transplantation are expected to be normal, conservative management with observation is not appropriate, and since this patient may have either recurrent HCV or rejection as a cause of his liver test abnormalities, it is incorrect to simply discontinue mycophenolate or start anti-HCV therapy with pegylated interferon and ribavirin without a histologic diagnosis.

Endo 4 A 27-year-old woman with an 8-year history of ulcerative colitis is evaluated during a follow-up examination. The initial colonoscopy after diagnosis showed pancolitis. She has been treated with mesalamine since diagnosis and has had episodes of bloody diarrhea two or three times a year but has otherwise done well. Her most recent colonoscopy 1 year ago when she had increased diarrhea and bleeding showed no progression of disease. Since then she has been clinically stable. The patient's medical history includes nephrolithiasis, and her only medications are mesalamine, 2.4 g/d, and a multivitamin. There is no family history of inflammatory bowel disease or colorectal cancer. On physical examination, vital signs are normal; BMI is 20.5. There is mild abdominal tenderness in the right lower quadrant without rebound or guarding. The rest of the physical examination is normal. Laboratory studies reveal a normal complete blood count, including leukocyte differential, and a serum C-reactive protein level of 0.8 mg/dL (8 mg/L). Which of the following is the most appropriate management of this patient's risk for colorectal cancer? A Annual capsule endoscopy B Annual flexible sigmoidoscopy C Colonoscopy now and annually thereafter D Increasing the dose of mesalamine to 3.6 g/d

C Patients with inflammatory bowel disease should initiate screening for colorectal cancer after 8 years of duration of disease. This patient has pancolitis of 8 years' duration. The inflammation involves the ileum and proximal colon. The colon cancer risk in patients with ulcerative colitis or Crohn disease reaches a significant level after 8 years of inflammation; the annual cancer risk is estimated to be 1% to 2% per year after 8 years. The cancer risk is slightly delayed for patients with inflammation limited to the distal colon. The recommendation is to initiate a surveillance program with colonoscopy 8 years after onset of her disease, with follow-up colonoscopy every 1 to 2 years thereafter. Biopsies are performed in four-quadrant fashion throughout the entire colon. The patient's disease is reasonably well controlled on her current dose of mesalamine, and treatment with mesalamine does not in itself prevent colon cancer. There is no recommendation for standard screening for small-bowel carcinoma in the setting of ulcerative colitis or Crohn disease, and therefore, capsule endoscopy is not indicated. Flexible sigmoidoscopy would not reach the at-risk colonic mucosa in the proximal colon beyond the reach of the sigmoidoscope.

Gastro 98 A 70-year-old man is evaluated for an 8-year history of elevated liver enzyme concentrations without jaundice, ascites, encephalopathy or hospitalizations for complications of cirrhosis or portal hypertension. The medical history also includes type 2 diabetes mellitus, hyperlipidemia, and obesity, and his medications include metformin, glyburide, rosiglitazone, hydrochlorothiazide, metoprolol, simvastatin, aspirin, and fish oil capsules; he does not drink alcohol. On physical examination, the temperature is 36.0 °C (97.0 °F), the blood pressure is 130/80 mm Hg, the pulse rate is 70/min, and the respiration rate is 14/min; the BMI is 30. Examination reveals hepatomegaly but no jaundice, ascites, edema, splenomegaly, or asterixis. Laboratory studies: Glucose (fasting) 101 mg/dL (5.6 mmol/L) Hemoglobin A1c 6% Bilirubin (total) 1.4 mg/dL (23.9 µmol/L) Aspartate aminotransferase 80 U/L Alanine aminotransferase 150 U/L Alkaline phosphatase 200 U/L Albumin 4.0 g/dL (40 g/L) Cholesterol (total) 140 mg/dL (3.63 mmol/L) HDL cholesterol 50 mg/dL (1.3 mmol/L) Triglycerides 100 mg/dL (1.13 mmol/L) Ferritin 255 ng/mL (255 mg/L) Transferrin saturation 30% Ultrasonography shows fatty infiltration. Which of the following is the most appropriate next step in management for this patient? A Gemfibrozil therapy B Insulin therapy C Liver biopsy D MRI of the abdomen E Observation

C Patients with metabolic syndrome and persistently elevated liver enzyme concentrations, despite adequate treatment of its components, should undergo liver biopsy. Nonalcoholic fatty liver disease includes nonalcoholic steatohepatitis (NASH) and steatosis (fatty degeneration). Liver biopsy would differentiate between the two disorders. NASH, the most likely finding in this patient, generally occurs in obese patients with dyslipidemia and diabetes mellitus or glucose intolerance. This patient has the metabolic syndrome with persistently elevated liver chemistry tests despite adequate management of the diabetes and lipid disorder that are components of the syndrome. However, he remains obese. The patient's evaluation excludes viral hepatitis, space-occupying lesions, and hemochromatosis, and the remaining possibilities include NASH and drug-induced liver injury. The liver biopsy specimen would likely show features of NASH, which include macrovesicular steatosis, ballooning degeneration, and lobular inflammation. Also, the presence of fibrosis would further identify this patient as being potentially at risk for progressive fibrosis attributable to NASH. Treatment of NASH depends on the cause. Drugs known to be associated with NASH should be discontinued. If this is not possible, the dose should be reduced. Weight loss is the cornerstone of treatment for patients with obesity, diabetes mellitus, and dyslipidemia. The patient's metabolic syndrome is adequately controlled, and therefore, therapy with insulin or a fibric acid derivative is not necessary and has not been shown to affect NASH. Because the only abnormality shown on ultrasonography is fatty infiltration, MRI would add little to the patient's evaluation.

Gastro 27 A 56-year-old woman is evaluated for iron-deficiency anemia detected during her annual examination. An esophagogastroduodenoscopy showed diffuse linear antral erosions and small clean-based ulcers; gastric biopsy specimens were negative for Helicobacter pylori, and small-bowel biopsy specimens were normal. A subsequent colonoscopy was normal. The patient's medical history includes long-standing rheumatoid arthritis and diarrhea-predominant irritable bowel syndrome, and her medications include methotrexate, hydroxychloroquine, nabumetone (an NSAID); daily calcium, vitamin D, and iron supplements; and loperamide. The patient has taken NSAIDs for many years; she once tried to switch to celecoxib, but it did not provide sufficient pain relief. On physical examination, vital signs are normal; she has slight joint swelling of the metacarpophalangeal and proximal interphalangeal joints consistent with rheumatoid arthritis. Abdominal examination is normal. Laboratory studies reveal hemoglobin of 10.6 g/dL (106 g/L). Which of the following is the most appropriate management for this patient? A Add an H2-receptor antagonist B Add misoprostol C Add a proton pump inhibitor D Add sucralfate

C Proton pump inhibitors are the drugs of choice in NSAID-induced gastric injury whether or not NSAID therapy is discontinued. This patient has gastric erosions and ulcerations consistent with NSAID-induced gastropathy, which would account for her iron-deficiency anemia. In most patients with NSAID-induced gastric injury, discontinuation of the NSAID in addition to starting proton pump inhibitor therapy, would be reasonable, but in certain patients (like this one who requires NSAID therapy for her rheumatoid arthritis), the best option is to start proton pump inhibitor therapy, which has been found to heal peptic ulcerations whether or not NSAID therapy is ongoing. In such patients, reducing the dose of the NSAID versus switching to either a selective COX-2 inhibitor, such as celecoxib, or an NSAID with a safer side effect profile, such as nabumetone, etodolac, or meloxicam, should be considered. This patient did not receive sufficient pain relief with celecoxib and she is already taking nabumetone; therefore, simply adding a proton pump inhibitor is indicated. Although high-dose H2-receptor antagonists such as famotidine are thought to be helpful in preventing ulcers, these agents have not been first-line therapy since the advent of proton pump inhibitors because they are not as effective. Misoprostol can reduce NSAID-related ulcers, but it has a dose-related side effect of diarrhea and would therefore not be indicated in this patient with diarrhea-predominant irritable bowel syndrome. Sucralfate has been used for ulcer prophylaxis, but there is no good evidence for its use in patients with long-term NSAID use and current ulcers.

Gastro 24 A 64-year-old woman is evaluated in the emergency department for her second episode of painless bloody stool. Four weeks ago, she was evaluated in the hospital for maroon stool; the hemoglobin at that time was 2 g/dL (20 g/L) lower than previous complete blood counts; esophagogastroduodenoscopy was normal; colonoscopy showed some old blood but no active bleeding. The patient was observed for 2 days and discharged with instructions for outpatient follow-up. Her current episode consisted of two maroon stools, one this morning and one 2 hours ago. Nasogastric lavage yields coffee grounds that clear with 1 L of saline. The patient had a colonoscopy 2 years ago at which time a single adenomatous polyp was detected and removed. She has no personal or family history of bleeding or gastrointestinal malignancy and is otherwise healthy; she has no upper gastrointestinal symptoms, does not use alcohol, and her only medication is a multivitamin. On physical examination, the temperature is 37.1 °C (98.7 °F), the blood pressure is 98/62 mm Hg, the pulse rate is 94/min, and the respiration rate is 14/min; the BMI is 23.5. There is no scleral icterus; examinations of the heart and lungs are normal. The abdomen is soft with increased bowel sounds but no hepatosplenomegaly. Rectal examination reveals the presence of maroon and red blood but no palpable masses or hemorrhoids. Laboratory studies reveal hemoglobin of 9.4 g/dL (94 g/L) with a mean corpuscular volume of 86 fL; leukocyte count with differential and platelet count are normal. Prothrombin time, activated partial thromboplastin time, and INR are normal, as are liver chemistry tests. Serum blood urea nitrogen is 34 mg/dL (12.1 mmol/L), and creatinine is normal. Which of the following is the most appropriate next step in the evaluation of this patient? A Colonoscopy B Double-balloon enteroscopy C Esophagogastroduodenoscopy D Technetium red blood cell scan E Wireless capsule endoscopy

C Repeat upper endoscopy in a patient with obscure upper gastrointestinal bleeding will identify a bleeding source in a significant proportion of patients. This patient presents with obscure overt bleeding in that blood loss is clinically apparent because of the maroon stool, but the source is not identified despite endoscopic investigation. The most appropriate next test would be to repeat upper endoscopy. Studies have shown that between one third and two thirds of sources of obscure upper gastrointestinal bleeding are found within the reach of the upper endoscope on repeat endoscopy. The missed lesions are most often bleeding erosions associated with hiatus hernia, gastroduodenal angiodysplasia, Dieulafoy lesions, gastric antral vascular ectasias, and peptic ulcers. Moreover, endoscopy offers therapeutic options if a lesion is found in addition to establishing a diagnosis. A colonoscopy is less likely to be helpful because the blood detected on nasogastric lavage suggests an upper gastrointestinal source. Wireless capsule endoscopy may be helpful if a repeat upper endoscopy is normal to assess the remainder of the small bowel; however, in contrast to endoscopy, capsule endoscopy does not offer therapeutic options. A radiolabeled red blood cell scan would not be the next test of choice; the results are nonspecific and would delay diagnosis and therapy that could be delivered more rapidly with endoscopy. In double-balloon enteroscopy, deep intubation of the small bowel is achieved by using a two-balloon technique to anchor the endoscope and assist in advancement. The procedure is lengthy and requires prolonged sedation, although it offers diagnostic and therapeutic capabilities. This test is reserved until a more proximal source of bleeding has been excluded and wireless capsule endoscopy has been performed. The capsule would direct whether the double-balloon enteroscope is to be introduced via the mouth (for jejunal abnormalities) or anus (ileal abnormalities).

Gastro 33 A 32-year-old man is evaluated in the emergency department for chest discomfort of 2 hours' duration and that occurred after he ate a large meal. He has difficulty swallowing and feels as though something is stuck in his chest; he is barely able to swallow his saliva and frequently spits into a cup. He has had two similar but less severe episodes in the past 6 months. The patient's only significant medical history is a 20-year history of seasonal allergic rhinitis treated with nasal corticosteroids and oral antihistamines. On physical examination, he is well developed and well nourished but uncomfortable. Esophagogastroduodenoscopy shows multiple esophageal rings with raised white specks, longitudinal furrows, and friable esophageal mucosa. Histologic examination of the mucosa shows intense inflammation of the lamina propria with more than 15 eosinophils per high-power field. No strictures are detected. Which of the following is the most appropriate management for this patient? A Endoscopic esophageal dilatation B Leukotriene receptor antagonist therapy C Oral topical (swallowed) corticosteroid therapy D Proton pump inhibitor therapy

C Swallowed liquid corticosteroids are the first-line therapy for eosinophilic esophagitis. Eosinophilic esophagitis is characterized by eosinophilic infiltration of the esophageal mucosa. Clinically, the disease most commonly presents with dysphagia or food impaction in atopic men in their third to fourth decades of life. For many years considered a pediatric disorder, the disease (or at least its recognition) appears to be rapidly increasing in adults. Diagnosis is suggested from clinical context, supported by endoscopic findings of mucosal furrowing or raised white specks (thought to represent eosinophilic microabscesses), and confirmed by histologic examination of the esophageal mucosa. Besides disimpaction of a food bolus, initial treatment is medical because of concerns about mucosal friability and esophageal perforation in the setting of dilatation. Swallowed topical corticosteroids (fluticasone propionate or beclomethasone) have been shown to produce clinical remission in up to 50% of adults. Proton pump inhibitors do not appear to be effective. Case reports citing the efficacy of leukotriene receptor antagonists have yet to be confirmed in randomized trials.

Gastro 99 A 63-year-old man is evaluated for a 2-day history of left lower quadrant abdominal pain. The pain is constant and is not relieved by a bowel movement or by positional changes. The patient is slightly nauseated and has no appetite but is not vomiting. He has never had a similar episode. The patient's medical history includes hypertension, and his only medication is hydrochlorothiazide. On physical examination, the temperature is 38.0 °C (101.1 °F), the blood pressure is 125/85 mm Hg, the pulse rate is 95/min, and the respiration rate is 14/min. There is fullness and tenderness of the left lower quadrant with no rebound or guarding; bowel sounds are decreased. Rectal examination is normal; examination of stool for occult blood is negative. Leukocyte count is 14,000/µL (14 × 109/L); all other laboratory results are normal. A plain abdominal radiograph is unremarkable, and a chest radiograph shows no free air beneath the diaphragms. Which of the following is the most appropriate next step in the evaluation of this patient? A Barium enema B Colonoscopy C Contrast-enhanced CT scan of the abdomen and pelvis D Small-bowel radiographic series

C The best imaging modality to confirm suspected diverticulitis and evaluate for extraluminal complications is a contrast-enhanced CT scan. This patient's left lower quadrant pain, fever, and elevated leukocyte count are classic symptoms and signs of diverticulitis. The most sensitive imaging modality to confirm this diagnosis as well as evaluate for any complications such as perforation, abscess, obstruction, and fistula is a contrast-enhanced CT scan of the abdomen and pelvis. A number of prospective investigations have reported a sensitivity of 69% to 95% and specificity of 75% to 100% for CT scan in acute diverticulitis. The presence of severe disease found on CT scan was prognostically very useful by accurately predicting failure of medical treatment and risk of secondary complications. Colonoscopy is generally avoided during an episode of acute diverticulitis for concern of increased risk of perforation with air insufflation; furthermore, colonoscopy would miss the extraluminal complications. A small-bowel series evaluates the small intestine, which is not affected in diverticulitis. Before the availability of CT scanning, barium enema was used to diagnose diverticulitis, but it is more cumbersome for both the patient and radiologist and like colonoscopy, presents a risk for perforation.

Gastro 5 A 71-year-old man is evaluated for chronic epigastric discomfort, heartburn, and diarrhea of 4 years' duration. His weight has been stable during this time period. The patient has no significant medical history and takes no medications. On physical examination, there is mild epigastric tenderness but no rebound or guarding. Rectal examination reveals brown stool that is positive for occult blood. Laboratory studies reveal hemoglobin of 12.3 g/dL (123 g/L) with a mean corpuscular volume of 75 fL. Test for serum Helicobacter pylori antibody is negative. Esophagogastroduodenoscopy shows prominent gastric folds, mild linear erosions, and multiple ulcers in the stomach and the duodenum. Which of the following is the most appropriate next step in the evaluation of this patient? A CT scan of the abdomen and pelvis B Endoscopic ultrasonography C Measurement of serum gastrin D Somatostatin receptor scintigraphy

C The first step in the evaluation of suspected gastrinoma is measurement of serum gastrin; serum gastrin concentration greater than 1000 pg/mL (1000 ng/L) is highly suggestive of gastrinoma. Gastrinoma is a neuroendocrine tumor that secretes the hormone gastrin, which in turn results in hypersecretion of acid. Gastrinoma should be suspected when ulcers are present in a patient who has a negative test for Helicobacter pylori and who does not take NSAIDs. The initial test when gastrinoma is suspected is measurement of serum gastrin; a gastrin concentration of 1000 pg/mL (1000 ng/L) or greater is highly suggestive of a gastrinoma, but gastrinoma may be present with gastrin levels as low as 150 to 200 pg/mL (150 to 200 ng/L). After the diagnosis of gastrinoma is made by detecting hypergastrinemia, various tests are used to localize the tumor and to evaluate for metastases. CT scan of the abdomen and pelvis is often the first test but may miss small pancreatic gastrinomas. Endoscopic ultrasonography is the most sensitive test for pancreatic endocrine tumors, including gastrinoma, but may miss tumors that are not localized within the pancreas. Somatostatin receptor scintigraphy is the most sensitive procedure for identifying the primary gastrinoma and detecting whether metastatic disease is present.

Gastro 88 A 72-year-old man is evaluated in the hospital for 6 weeks of progressive dysphagia, initially for solid foods and now for both solid foods and liquids. He has had a 6.8-kg (15-lb) weight loss during this episode. He has not had fever, chills, sweats, nausea, or vomiting. The patient has hypertension, which is controlled with amlodipine therapy. He also takes a daily aspirin. On physical examination, he appears ill and is thin (BMI 19); vital signs are normal. Laboratory findings include hemoglobin of 10.3 g/dL (103 g/L), mean corpuscular volume 88 fL, creatinine 1.5 mg/dL (114.5 µmol/L), and sodium 133 meq/L (133 mmol/L). CT scan shows a right lower lobe pulmonary infiltrate, thickening of the lower esophagus, multiple enhancing hepatic lesions, and celiac lymphadenopathy. Esophagogastroduodenoscopy shows a circumferential mass in the lower esophagus; the endoscope can pass through the mass. Biopsy specimen of the mass shows adenocarcinoma. Which of the following is the most appropriate management for this patient? A Esophagectomy B Esophagectomy with hepatic wedge resection C Neoadjuvant chemotherapy D Palliative self-expanding metal stent

D Endoscopic stenting therapy in advanced esophageal carcinoma can provide significant improvements in quality of life. This patient has advanced esophageal adenocarcinoma. Although clinical trials have shown survival benefit with aggressive therapy in early-stage esophageal adenocarcinoma, the management options in advanced disease are largely palliative. Patients with advanced stages of esophageal cancer or those that are poor candidates for surgery can be offered stenting as a palliative treatment for dysphagia. Stenting can also be used for palliation of patients with postoperative tumor recurrence. Both plastic and metal stents can be used for palliation of obstructing esophageal adenocarcinoma, although complications are more frequent with plastic stents. In this case, the patient presents with very advanced disease, including hepatic metastases. Esophagectomy is a highly morbid procedure, which will offer no survival advantage to this individual. Hepatic wedge resection is not indicated in such advanced disease.

Gastro 54 A 63-year-old man is evaluated in the hospital 3 days after having undergone a left hemicolectomy for adenocarcinoma of the colon. The patient has bloating and mild diffuse abdominal discomfort. He has not defecated or passed gas since surgery. He is nauseated but has not vomited. His medical history also includes hypertension and type 2 diabetes mellitus, and his medications are oxycodone, lisinopril, and glyburide. On physical examination, the abdomen is distended with mild diffuse tenderness, tympany, and hypoactive bowel sounds but without rebound or guarding; there are no abdominal masses or organomegaly. Laboratory studies reveal random glucose 290 mg/dL (16.1 mmol/L), potassium 3.1 meq/L (3.1 mmol/L), and magnesium 1.1 mg/dL (0.45 mmol/L); complete blood count, serum thyroid-stimulating hormone, liver chemistry tests, and amylase are normal. Plain radiograph of the abdomen shows diffusely dilated loops of small bowel; CT scan of the abdomen and pelvis also shows dilated loops of bowel without evidence of obstruction. In addition to glucose control and correction of electrolytes, which of the following is the most appropriate next step in the management of this patient? A Insertion of a nasogastric tube to gravity drainage B Insertion of a rectal tube C Intravenous metoclopramide D Minimization of oxycodone

D Ileus is a functional obstruction of the bowel that commonly complicates the postoperative period; treatment is generally supportive, including the discontinuation of medications that slow gut motility and correction of electrolyte abnormalities. This patient has a postoperative ileus. An ileus is a functional, as opposed to mechanical, gut obstruction that occurs when the normal bowel motility is inhibited. Metabolic derangements, medications such as narcotic analgesics and anticholinergic agents, and infection may exacerbate or contribute to the development of an ileus. The pathogenesis of a postoperative ileus likely relates to impaired gut electrical, neurologic, and hormonal activity; inflammatory mediators; and a dysmotility effect from anesthesia. Patients have difficulty tolerating oral intake and may complain of bloating, abdominal pain, and lack of flatus and stool passage; abdominal distention, normal or decreased bowel sounds, and diffuse mild tenderness may be observed on physical examination. An abdominal plain film generally shows air throughout the intestine but may be confused with mechanical small-bowel obstruction because air-fluid levels and minimal colonic gas may also be seen; therefore, CT and barium studies may help distinguish these entities. Management of an ileus is generally supportive, including the discontinuation of any medications such as narcotics and calcium channel blockers that may further slow gut motility and correction of electrolyte and other metabolic abnormalities. Although nasogastric tubes may be appropriate for a small-bowel obstruction, they are generally not efficacious in managing an ileus except to provide symptomatic relief from emesis. Rectal tubes are used to decompress the colon but would not be helpful in treating an ileus. Studies of metoclopramide therapy in this setting have not shown accelerated postoperative gastrointestinal recovery.

Gastro 40 A 58-year-old man is evaluated for a 4-week history of progressive difficulty swallowing liquids. The symptom begins immediately with the initiation of a swallow and has recently been associated with coughing. The patient does not have fever or weight loss. His only significant medical history is atherosclerotic cardiovascular disease; his only medications are a β-blocker, a statin, and aspirin. Vital signs are normal, and physical examination shows only crackles at the posterior base of the right lung field. Which of the following is the most appropriate next diagnostic test in the evaluation of this patient? A Ambulatory esophageal pH monitoring B CT scan of the chest and abdomen C Esophageal manometry D Videofluoroscopy

D In patients with dysphagia, immediate onset of symptoms with initiation of a swallow, difficulty swallowing liquids, and cough associated with the swallowing difficulty are characteristic of oropharyngeal rather than esophagealdysphagia. The diagnosis and management of swallowing difficulties depend initially on determining whether the patient's symptoms are characteristic of either oropharyngeal dysphagia or esophageal dysphagia. The cornerstone of this determination is the medical history. In this patient, the immediate onset of symptoms with the initiation of a swallow, the difficulty swallowing liquids, and the cough associated with the swallowing difficulty make oropharyngeal, or transfer, dysphagia more likely than esophageal dysphagia. Videofluoroscopy, which allows real-time radiographic analysis of swallowing function, is the most sensitive test for oropharyngeal dysphagia. The procedure can show abnormal movement of a bolus, such as aspiration, pooling in pharyngeal recesses, movement of anatomic structures, muscle activities throughout the area, and oral and pharyngeal transit times. Common causes of oropharyngeal dysphagia are stroke, Parkinson disease, amyotrophic lateral sclerosis, myasthenia gravis, and muscular dystrophy. Ambulatory esophageal pH monitoring is the most accurate technique to diagnose gastroesophageal reflux disease but is not the study of choice for evaluation of oropharyngeal dysphagia. CT scan of chest and abdomen can be an important adjunct to the evaluation of patients with esophageal dysphagia, because extraluminal causes of obstruction can be detected by cross-sectional imaging. Esophageal manometry is used in the diagnostic evaluation of patients with motility disorders of the esophagus. Although patients with such disorders may present with dysphagia of both solids and liquids, the immediate onset of symptoms with swallowing makes oropharyngeal dysphagia more likely.

Gastro 58 A 42-year-old man is evaluated for a 1-month history of progressive jaundice, pruritus, and dark urine. The patient has a 15-year history of ulcerative colitis; he is poorly compliant with mesalamine therapy and occasionally requires corticosteroid therapy. He drinks two cans of beer a day and has never used injection drugs or received a blood transfusion. He takes no other medications. On physical examination, vital signs are normal. There is mild jaundice and hepatomegaly but no splenomegaly, ascites, or abdominal tenderness. There is no asterixis. Laboratory studies: Platelet count 350,000/µL (350 × 109/L) INR 1.1 Aspartate aminotransferase 150 U/L Alanine aminotransferase 180 U/L Bilirubin (total) 4.2 mg/dL (71.8 µmol/L) Alkaline phosphatase 450 U/L Total protein 6.0 g/dL (60 g/L) Albumin 3.8 g/dL (38 g/L) Which of the following is the most likely diagnosis? A Autoimmune hepatitis B Hepatitis A C Hepatitis C D Primary sclerosing cholangitis

D Patients with acute hepatitis have a marked elevation of aminotransferases, whereas patients with primary sclerosing cholangitis have a cholestatic pattern. This patient likely has primary sclerosing cholangitis, a chronic cholestatic liver disease associated with inflammatory bowel disease and characterized by fibrosis, inflammation, and stricturing of the biliary tree. Up to 85% of affected patients have underlying inflammatory bowel disease, but <5% of patients with inflammatory bowel disease have primary sclerosing cholangitis. The disorder is more common in patients with ulcerative colitis than with Crohn disease. Most patients are diagnosed while asymptomatic with abnormal results on liver biochemistry tests, but jaundice and pruritus can occur in patients with advanced disease. The diagnosis is usually made by endoscopic retrograde cholangiopancreatography, which is especially useful in advanced disease where histologic samples can be taken to rule out cholangiocarcinoma and stents can be placed if there is a dominant stricture. Magnetic resonance cholangiopancreatography can also be used. Autoimmune hepatitis, which is more common in women than in men, may have a cholestatic picture as in this patient, but it is often associated with other autoimmune disorders, such as hemolytic anemia, idiopathic thrombocytopenic purpura, type 1 diabetes mellitus, thyroiditis, and celiac disease. Laboratory findings in patients with autoimmune hepatitis include elevated serum aminotransferase values, hypergammaglobulinemia, mild hyperbilirubinemia, elevated serum alkaline phosphatase values, and the presence of autoantibodies. Serum gamma globulin values ≥1.5 times the upper limit of normal are common. This patient's gamma globulin level is not elevated (estimated by subtracting the albumin concentration from the total protein), making autoimmune hepatitis unlikely. Patients with acute hepatitis C are usually asymptomatic and therefore rarely present clinically, but 60% to 85% of persons who acquire acute hepatitis C develop chronic infection. The cholestatic picture in the absence of other signs of advanced liver disease is inconsistent with hepatitis C cirrhosis. Hepatitis A can present acutely with fulminant liver failure, but the concentrations of the aminotransferases would be much higher, usually greater than 500 U/L.

Gastro 62 A 35-year-old woman in her second trimester of pregnancy is evaluated for liver cysts detected on ultrasonography of the abdomen. She has had a normal pregnancy and has no abdominal pain, jaundice, fever, or weight loss. Her only medications are a multivitamin and folic acid. On physical examination, the temperature is 36.1 °C (97.0 °F), the blood pressure is 105/70 mm Hg, the pulse rate is 90/min, and the respiration rate is 14/min. The right lower lobe of the liver is palpable two finger breadths below the costal margin. Laboratory tests, including liver chemistry tests, are normal. Ultrasonography shows four 2- to 4-cm fluid-filled cysts with no internal masses or debris. Which of the following is the most appropriate diagnostic evaluation? A Aspiration of the cysts after delivery B Measurement of serum α-fetoprotein and carcinoembryonic antigen C MRI of the liver D No further evaluation

D Simple liver cysts are cystic formations that containing clear fluid; few simple cysts enlarge or cause symptoms. No further diagnostic tests are required in this asymptomatic patient with cystic liver lesions; the ultrasonographic findings show the cysts to have the characteristics of simple cysts (hypoechoic; no solid component) and no features of more concerning lesions such as cystadenoma or cystadenocarcinoma (for example, thickened irregular walls) or metastasis (necrotic center). Simple cysts are the most common benign mass of the liver and are characterized as cystic formations containing clear fluid that do not communicate with the intrahepatic biliary tree; few cysts enlarge or cause symptoms. Simple cysts are not estrogen sensitive and have no relation to the pregnancy. Subsequent use of oral contraceptives would be acceptable. Clinical features combined with the ultrasonographic findings are usually sufficient to distinguish simple cysts from liver abscess, necrotic malignant tumor, hemangioma, and hamartoma. The lack of symptoms attributable to the cysts and the ultrasonographic features are sufficient to feel confident about the diagnosis. Therefore, further imaging, aspiration, or measurement of tumor markers is not needed.

Gastro 57 A 65-year-old woman is evaluated in routine follow-up. Three months ago she was evaluated for fatigue and pruritus and was found to have an elevated serum alkaline phosphatase concentration and antimitochondrial antibody positive at a titer of 1:640. Liver biopsy specimen was consistent with stage 1 primary biliary cirrhosis. The pruritus is controlled with cholestyramine therapy. The patient is taking calcium, 1500 mg/d, and vitamin D, 800 IU/d. She has no other medical problems or symptoms. On physical examination, vital signs are normal; BMI is 26. The rest of the examination is normal. Laboratory studies: Bilirubin (total) 1.0 mg/dL (17.1 µmol/L) Aspartate aminotransferase 65 U/L Alanine aminotransferase 70 U/L Alkaline phosphatase 200 U/L Dual energy x-ray absorptiometry scan shows normal bone mineral density. Which of the following is the most appropriate therapy for this patient? A Alendronate B Estrogen replacement C Evaluation for liver transplantation D Ursodeoxycholic acid

D Ursodeoxycholic acid therapy is indicated for early-stage primary biliary cirrhosis and may reduce the rate of liver transplantation and prolong survival in affected patients. This patient has early-stage primary biliary cirrhosis, and the most appropriate therapy is ursodeoxycholic acid. Primary biliary cirrhosis is a chronic progressive cholestatic liver disease of unknown cause. This disorder is overwhelmingly a disease of women (>90%) older than 30 years. The most common symptom is persistent fatigue, which occurs in as many as 80% of patients. Either localized or general pruritus frequently develops. The pruritus often begins in the perineal area or on the palmar and plantar surfaces and is typically worse at night or in a warm environment. Jaundice and abdominal pain may also develop. However, many patients may be asymptomatic at presentation. An antimitochondrial antibody titer of ≥1:40 is the serologic hallmark for the presence of primary biliary cirrhosis and occurs in 90% to 95% of patients. However, the titer does not appear to correlate with the severity or progression of the clinical disease. Treatment with ursodeoxycholic acid improves the biochemical profile, reduces pruritus, decreases progression to cirrhosis, and delays the need for liver transplantation if begun at an early stage of disease. The data are more conflicting for later stages of disease. Therapy is usually continued indefinitely. Alendronate may be appropriate for patients with primary biliary cirrhosis and osteoporosis, but this patient has normal bone mineral density and is taking adequate calcium and vitamin D replacement. Hormone replacement therapy is no longer regarded as the mainstay of therapy for osteoporosis. In the Women's Health Initiative, the use of conjugated estrogens and medroxyprogesterone in postmenopausal women increased the risk of cardiovascular disease, invasive breast cancer, stroke, deep venous thrombosis, and pulmonary embolism. The patient reports no menopausal symptoms that might be an indication for short term use of estrogen replacement therapy. The patient has no indications of liver failure, and therefore liver transplantation is not indicated.

Gastro 19 A 53-year-old man is evaluated after a recent episode of substernal chest pain. He was evaluated in the emergency department for chest pain, and serial electrocardiograms and measurement of cardiac enzymes showed no evidence of myocardial ischemia. An outpatient stress test also showed no evidence of myocardial disease. The patient has a history of medically controlled hypertension, and his only medication is amlodipine. On physical examination, the patient appears healthy and vital signs are normal. Barium esophagography shows a segmented or "corkscrew" (Korkenzieher) esophagus. Esophageal manometry shows simultaneous contractions in the distal esophagus with 50% of swallows. Which of the following is the most appropriate therapy for this patient? A Esophageal bougienage B Laparoscopic myotomy C Oral anticholinergic therapy D Oral proton pump inhibitor therapy E Pneumatic dilatation

D Diffuse esophageal spasm is frequently related to gastroesophageal reflux; empiric therapy with acid suppression is the recommended first therapeutic intervention. This patient has chest pain resulting from diffuse esophageal spasm. Although the diagnosis can be suggested by clinical presentation and the distinctive appearance of the esophagus on barium study, the disorder can only be diagnosed on manometry characterized by the finding of more than 20% of swallows having simultaneous contractions in the distal esophagus. Although the treatment for diffuse esophageal spasm is not clearly defined or the subject of general expert agreement, it is clear that many cases result from uncontrolled gastroesophageal reflux. Consequently, an empiric trial of acid suppression is usually recommended as the first therapeutic maneuver. Oral anticholinergic agents have had limited success in this disorder, and their use is associated with frequent side effects. The more aggressive options of esophageal dilatation and surgical intervention have been proposed in case reports but have not been shown to be generally useful. Such approaches are reserved for patients with severe symptoms refractory to medical therapy.

Gastro 55 A 58-year-old woman is evaluated in the emergency department for substernal chest pain of 18 hours' duration. She describes the pain as a tightening that is not associated with eating or exertion and that radiates to the neck. The pain is not accompanied by dyspnea, nausea, or diaphoresis. She had a similar episode 1 month ago, and an exercise stress test showed no areas of ischemia. The patient's medical history includes hypertension and type 2 diabetes mellitus; her medications include ramipril, metformin, and aspirin. On physical examination, the patient appears uncomfortable but not acutely ill. The temperature is 37.2 °C (99.0 °F), the blood pressure is 130/74 mm Hg, the pulse rate is 88/min, and the respiration rate is 16/min; the BMI is 31.5. The lungs are clear; the heart rate is regular and there are no murmurs. Electrocardiography shows nonspecific ST and T wave abnormalities, which are unchanged from a previous examination. Which of the following is the most appropriate management for this patient? A Ambulatory esophageal pH monitoring B Esophageal manometry C Esophagogastroduodenoscopy D Oral proton pump inhibitor therapy E Repeat exercise stress test

D Empiric proton pump inhibitor therapy is the first step in management of esophageal noncardiac chest pain. Esophageal disease is the most common cause of noncardiac chest pain. This patient's lack of signs and symptoms referring to the heart and her previous normal stress test make cardiac disease very unlikely. Because noncardiac chest pain is often associated with gastroesophageal reflux disease, it is reasonable in this patient to attempt empiric acid suppression therapy with a proton pump inhibitor. Ambulatory esophageal pH monitoring can be used if the patient's condition does not respond to empiric therapy or if the patient has atypical symptoms. A history consistent with diffuse esophageal spasm or achalasia might require esophageal manometry as a first test, but it should also be reserved for patients who do not respond to empiric therapy. Esophagogastroduodenoscopy is indicated in patients with long-standing reflux disease to screen for Barrett esophagus or in patients with such alarm symptoms as gastrointestinal bleeding, weight loss, or fever. Repeating the stress test would not be indicated before a trial of a proton pump inhibitor in this patient with likely reflux disease.

Gastro 28 A 65-year-old man is evaluated in the emergency department for fever and abdominal pain. The patient has a history of diverticulitis, and his latest flare 2 months ago was treated as an outpatient with antibiotics. While improved, he still has residual pain in the left lower quadrant. He has no other significant medical history and has not traveled recently. On physical examination, the temperature is 38.8 °C (101.8 °F), the blood pressure is 120/85 mm Hg, the pulse rate is 100/min, and the respiration rate is 18/min; the BMI is 25. There is right upper quadrant abdominal pain with normal bowel sounds and no peritoneal signs, scleral icterus, or ascites. Laboratory studies: Hemoglobin 14.2 g/dL (142 g/L) Leukocyte count 17,000/µL (17 × 109/L) with 15% band forms Platelet count 320,000/µL (320 × 109/L) INR 1.0 Bilirubin (total) 1.6 mg/dL (27.4 µmol/L) Bilirubin (direct) 0.6 mg/dL (10.3 µmol/L) Aspartate aminotransferase 97 U/L Alanine aminotransferase 86 U/L Alkaline phosphatase 220 U/L CT scan of the abdomen and pelvis shows a mass with a fluid level consistent with a hepatic abscess. Antibiotic therapy is begun. Which of the following is the most appropriate next step in the management of this patient? A Endoscopic retrograde cholangiopancreatography B Lobectomy C MRI with gadolinium D Percutaneous drainage of the hepatic lesion

D Hepatic abscess is usually managed with percutaneous drainage of the lesion and intravenous antibiotics. This patient requires percutaneous drainage of the hepatic lesion, which is likely an abscess. He has a history of recurrent diverticulitis, which does not appear to have been adequately treated in the past. He now has fever, right upper quadrant pain, and a large fluid-filled lesion in his liver. The clinical manifestations of pyogenic liver abscess usually include fever and right upper quadrant abdominal pain. Most pyogenic liver abscesses are due to biliary tract infection. Intra-abdominal infection may result in liver abscesses from seeding via the portal vein, whereas systemic bacteremia may cause abscesses from seeding via the hepatic artery. Abscesses resulting from an intra-abdominal infection are frequently due to anaerobic organisms, and many liver abscesses are polymicrobial. Percutaneous aspiration is recommended for diagnosis and subsequent treatment of a suspected pyogenic liver abscess. Antibiotics may be started even before aspiration and should not interfere with culture results. Parenteral antibiotic therapy to cover enteric gram-negative bacilli, anaerobes, and enterococci is recommended until culture results are available. Antibiotics specific for the causative organisms are then started and are continued for 4 to 6 weeks. Oral agents can be instituted after the abscess is drained and the patient is afebrile. Lobectomy is not required in this patient because in most cases, a hepatic abscess can be managed with the combination of antibiotics and percutaneous drainage. Endoscopic retrograde cholangiopancreatography (ERCP) would not be appropriate because there is no evidence that this patient has cholangitis or obstructive jaundice. Although he has abnormal liver chemistry tests, these can be explained by the presence of the large abscess rather than biliary disease, especially because the CT scan does not show any evidence of ductal dilatation to suggest obstructive jaundice or cholangitis, which might require ERCP. MRI with gadolinium is not appropriate because the diagnosis can be made on the information provided, and no further confirmatory imaging is needed. The patient needs to be treated, and antibiotics and percutaneous drainage are required at this time.

Gastro 78 A 55-year-old man is evaluated for a 3-month history of five or six bowel movements a day. The patient was diagnosed with pancolitis 25 years ago. He has had severe attacks in the past necessitating corticosteroid therapy. His disease had been maintained on therapy with azathioprine and mesalamine, and he is contemplating (in Betracht ziehen) a change in his regimen to an anti-tumor necrosis factor-α agent. His most recent surveillance colonoscopy was 18 months ago, and before instituting the change, he undergoes another colonoscopy. He is otherwise healthy, and his only medications are the azathioprine, 150 mg/d, and mesalamine, 4.8 g/d. On physical examination, vital signs are normal; there is mild lower abdominal tenderness, but the rest of the examination and the results of laboratory studies are normal. Colonoscopy shows numerous pseudopolyps and moderate to severe active inflammation in the left colon, with a transition to normal appearing mucosa at the splenic flexure. Biopsy specimen from the sigmoid colon shows high-grade dysplasia; a specimen from the descending colon shows low-grade dysplasia. Which of the following is the most appropriate management for this patient? A Annual surveillance colonoscopy with multiple biopsies B Hemicolectomy with colorectal anastomosis C Therapy with infliximab D Total proctocolectomy

D High-grade dysplasia detected at a surveillance colonoscopy for ulcerative colitis necessitates total proctocolectomy. This patient has had intensive medical therapy including azathioprine, an immunomodulator, and occasional corticosteroid therapy. His disease is now active despite his compliance with an intensive medical regimen. His medical options are limited, and infliximab would be the next best choice of nonsurgical therapy. However, the presence of multifocal dysplasia (with one being high-grade) precludes any option other than colectomy. A total proctocolectomy is the only acceptable option for this patient. Between 25% and 40% of patients with ulcerative colitis eventually require surgery for dysplasia, cancer, or medically refractory disease. In order to achieve a 90% sensitivity for the detection of dysplasia on surveillance colonoscopy, 33 biopsy specimens must be taken; the number increases to 56 specimens for a 95% sensitivity. However, in this patient high-grade dysplasia has already been established, and therefore further surveillance is unnecessary. Although the active disease and areas of dysplasia do not appear to affect the more proximal colon, a segmental colectomy, although appropriate for sporadic colorectal carcinoma, is not appropriate in ulcerative colitis. The inflammatory activity will recur in the remaining colon, and the cancer risk persists throughout the colon, particularly in this patient who has had severe pancolitis in the past.

Gastro 12 A 42-year-old woman is evaluated for a 20-year history of constipation. She has approximately one or two bowel movements a week consisting of lumpy or hard stool. She strains at defecation and has a sense of incomplete evacuation after a bowel movement. She does not have bloody stool, abdominal pain or discomfort, weight loss, or diarrhea. She is otherwise healthy, and her only medication is an occasional over-the-counter laxative or stool softener. On physical examination, vital signs are normal. The anorectal tone is normal, and on rectal examination, the patient is able to expel the examiner's finger when asked to mimic a bowel movement. Laboratory studies are normal. Radiopaque marker study shows delayed transit time through the right colon. Which of the following is the most likely diagnosis? A Chronic intestinal pseudo-obstruction B Constipation-predominant irritable bowel syndrome C Pelvic floor dysfunction (dyssynergic constipation) D Colonic inertia (slow-transit constipation)

D In a radiopaque marker study, patients with slow-transit functional constipation have prolonged transit of markers through the proximal colon, whereas patients with dyssynergic functional constipation have normal transit time but impaired rectal expulsion of the markers. Functional constipation is a common complaint. To make the diagnosis, there should not be any secondary causes such as an underlying metabolic or neurologic disease, mechanical obstruction, or drug effect. The disorder is characterized by the presence of two of the following Rome III criteria for functional constipation (for at least 3 months): straining during ≥25% of defecations; lumpy or hard stools ≥25% of defecations; sensation of incomplete evacuation for ≥25% of defecations; sensation of anorectal obstruction/blockage for ≥25% of defecations; manual maneuvers to facilitate ≥25% of defecations; and less than three defecations per week. Patients should not meet the Rome criteria for irritable bowel syndrome, which consist of recurrent abdominal pain or discomfort at least 3 days a month in past 3 months associated with two or more of the following: improvement with defecation; onset associated with change in frequency of stool; and onset associated with change in form (appearance) of stool. Patients with functional constipation should also not have alternating loose stools and constipation. There are two types of functional constipation: slow-transit (colonic inertia) and dyssynergic. This patient has slow-transit functional constipation. Affected patients have prolonged transit of radiopaque markers through the proximal colon, which is likely the result of an abnormality in the enteric nerve plexus. In dyssynergic functional constipation or pelvic floor dysfunction (also known as outlet delay), transit time through the colon is normal but rectal expulsion of radiopaque markers is impaired, often resulting from the loss of the coordinated relaxation (or paradoxical contraction) of the puborectalis muscle and external anal sphincter. A radiopaque marker study therefore helps to differentiate these two types of functional constipation. Anorectal manometry can also help distinguish the two types, but a simpler diagnostic maneuver is having the patient mimic a bowel movement and attempt to expel the examiner's inserted finger; if the patient cannot do so, dyssynergia rather than slow transit is present. This patient does not meet criteria for irritable bowel syndrome. There is no underlying disease or radiographic evidence in this patient to support the diagnosis of chronic pseudo-obstruction.

Gastro 100 A 65-year-old woman is evaluated 1 week after having had an esophagogastroduodenoscopy for persistent abdominal pain. The procedure showed a 1-cm, clean-based ulcer in the duodenal bulb and scattered antral erosions. Biopsy specimens from the stomach showed nonspecific gastritis but no evidence of Helicobacter pylori infection. Serum antibody testing for H. pylori was also negative. Proton pump inhibitor therapy was started, and the patient's symptoms were alleviated. The patient has a history of mild osteoarthritis and osteoporosis, and her medications include a nonprescription analgesic for arthritis and a calcium supplement, vitamin D, and alendronate. On physical examination, vital signs are normal. The abdominal examination reveals no tenderness, hepatomegaly, or palpable masses. Complete blood count is normal. Which of the following is the most appropriate next step in the management of this patient? A Measure serum gastrin B Perform fecal antigen test for Helicobacter pylori C Repeat esophagogastroduodenoscopy with biopsy of the ulcer D Review the nonprescription arthritis analgesic E Stop alendronate therapy

D In patients with peptic ulcers, nonprescription medications should be reviewed to determine whether NSAIDs are being inadvertently used. The two most common causes of peptic ulcer disease are NSAIDs and Helicobacter pylori infection, which account for more than 90% of cases. This patient has a history of arthritis for which she takes an over-the-counter analgesic, and therefore, inadvertent use of NSAIDs, which are widely available without a prescription and are often used in arthritis as analgesics, needs to be considered; many patients who take such nonprescription medications are unaware that they are taking NSAIDs. H. pylori infection has been ruled out in this patient by the negative histology for the organism as well as negative serum antibody testing; therefore, no further testing for H. pylori is needed. Measuring serum gastrin should be considered in a patient in whom there is a suspicion of an acid hypersecretion state, such as a gastrinoma (Zollinger-Ellison syndrome), clinical features of which include multiple peptic ulcers, ulcers in unusual locations, severe esophagitis, or fat malabsorption, none of which this patient has. Malignancy always needs to be considered in a patient with a gastric ulcer; therefore, biopsies of the ulcer and follow-up endoscopy to ensure ulcer healing would be recommended. However, this patient has a duodenal ulcer, which is much less likely to represent a malignancy, and biopsy of the ulcer or follow-up endoscopy to assess for healing is not needed. Alendronate therapy for osteoporosis has been associated with esophagitis and rare cases of gastric or duodenal ulcers; however stopping alendronate without considering the more common causes of peptic ulcer disease would not be appropriate at this time.

Gastro 8 A 41-year-old woman is evaluated for a 4-month history of intermittent mid-upper-abdomen pain, which does not radiate and is not affected by eating. She had gastroesophageal reflux when she was pregnant, but she says that the current symptoms are not like those of reflux or heartburn. She occasionally feels nauseated and mildly bloated, but she has not vomited, felt early satiety, or lost weight. She does not have difficulty swallowing or painful swallowing. Her bowel movements are normal. She has been pregnant twice and had two healthy children, both delivered by cesarean section. Her medical history also includes a cholecystectomy 5 years ago. Her only current medication is a multivitamin. On physical examination, she is afebrile; the pulse rate is 65/min and the blood pressure is 110/65 mm Hg. There is no jaundice or scleral icterus; mild epigastric tenderness is present. Bowel sounds are normal; there are no abdominal bruits, palpable masses, or lymphadenopathy. Complete blood count and liver chemistry tests are normal. Which of the following is the most appropriate next diagnostic test in the evaluation of this patient? A Abdominal ultrasonography B Esophagogastroduodenoscopy C Gastric scintigraphy D Helicobacter pylori stool antigen E Small-bowel radiograph

D In patients younger than 55 years who have new-onset dyspepsia without alarm symptoms, a "test and treat" approach for Helicobacter pylori is recommended. This patient has new-onset uninvestigated dyspepsia. In a patient younger than 55 years without alarm features, the recommendation is to "test and treat" for Helicobacter pylori infection. The H. pylori stool antigen test detects active infection and therefore can be used in the initial diagnosis as well as for eradication testing. An abdominal ultrasonography can be used in the evaluation of presumed biliary colic or chronic mesenteric ischemia (the latter if Doppler evaluation is additionally performed). The likelihood of biliary disease in this patient who is post-cholecystectomy and who has normal serum chemistry tests is low. Chronic mesenteric ischemia is also very unlikely because her pain does not increase with eating, and she has not lost weight, has no known risk factors for atherosclerosis, and has no abdominal bruits. An esophagogastroduodenoscopy is not needed given her age and lack of alarm features, but the procedure could be considered if her symptoms persist after H. pylori treatment. Gastric scintigraphy is used in the evaluation of gastroparesis and should only be performed after mechanical obstruction has been ruled out with upper endoscopy or barium radiography. The patient's predominant symptom is pain and she has not vomited or had early satiety, and therefore, dyspepsia is more likely than gastroparesis. Although this patient has had several abdominal surgeries putting her at risk for a small-bowel obstruction from adhesions, the fact that her pain does not worsen with eating, she has not vomited, has maintained normal bowel movements, and has normal bowel sounds make obstruction unlikely; therefore, a small-bowel radiograph would be unnecessary at this point.

Gastro 36 A 37-year-old man is evaluated to determine his risk for colon cancer because his 62-year-old mother was recently diagnosed with colon cancer. The patient is healthy and takes no medications. He has a bowel movement daily, has not noticed any change in bowel habits, and has never seen blood in his stool. He does not have any abdominal pain, weight loss, or jaundice. His mother's cancer was in the ascending colon, and she is recovering from surgery. His maternal uncle was diagnosed with colon cancer at age 48 years, and he died of the disease a year later. His mother's other two siblings, aged 44 and 46 years, are healthy. His maternal grandfather was diagnosed with colon cancer in his 60s but died of complications of heart disease. His maternal grandmother survived endometrial cancer diagnosed in her 50s and died in her 70s of a stroke. There is no history of cancer on his father's side. Which of the following is the most appropriate management for this patient? A Blood test of the mother to evaluate for microsatellite instability B Colonoscopy when he reaches age 50 years C Flexible sigmoidoscopy now D Referral for genetic counseling

D Initial genetic testing for colon cancer syndromes is most effective in a patient with cancer rather than in an unaffected family member. This patient has a family history that meets the Amsterdam criteria for hereditary nonpolyposis colorectal cancer (HNPCC). He should be referred to a genetic counselor to review the family history and be able to address the many issues that surround genetic testing and the implications for both the patient and his family members. The best approach to test for HNPCC in this case and confirm the diagnosis would be to obtain tumor tissue from the patient's mother and test it for microsatellite instability. If the test is positive, a blood test can be done in the patient's mother in an attempt to find a germline mutation in the mismatch repair genes, and if identified, the patient may be tested to determine whether he is a carrier of that mutation. A negative blood test in the mother does not exclude the diagnosis of HNPCC. Screening for colon cancer in HNPCC should be initiated between age 20 and 25 years and requires a complete colonoscopy because cancers frequently occur in the ascending colon. Genetic testing for colon cancer syndromes should be performed under the guidance of a genetic counselor. The counselor is skilled at appropriate data collection, pedigree analysis, and pre- and posttest counseling. Flexible sigmoidoscopy may identify polyposis for patients with familial adenomatous polyposis; however, it is inadequate screening for other hereditary colorectal cancer syndromes, such as attenuated familial adenomatous polyposis and HNPCC.

Gastro 38 A 25-year-old woman is evaluated for a 2-year history of almost daily bloating and lower abdominal cramping; the symptoms are associated with constipation, relieved with bowel movements, and seem worse when she is under stress. She has one or two small bowel movements a week and often has a feeling of incomplete evacuation. She never has diarrhea and has not had blood in the stool, nocturnal awakening with pain or for bowel movements, or weight loss. She has taken a fiber supplement without relief. The patient is otherwise healthy, and her only medication is an oral contraceptive pill that she has been taking for 1 year. Her mother had a similar condition when she was younger, but both her parents are alive and well. On physical examination, vital signs are normal; there is mild lower abdominal tenderness with no rebound, guarding, or palpable abdominal masses. Laboratory studies reveal hemoglobin 13.1 g/dL (131 g/L); serum biochemistry tests, including thyroid-stimulating hormone, are normal. Which of the following is the most appropriate next step in the management of this patient? A Colonoscopy B CT scan of the abdomen and pelvis C Discontinue the oral contraceptive D Reassurance and polyethylene glycol

D Irritable bowel syndrome is a clinical diagnosis that can be made confidently when patients meet the Rome III criteria and do not have alarm indicators. This patient has irritable bowel syndrome. As a young woman, she fits the demographic profile, and she also meets the Rome III criteria, with abdominal pain relieved by defecation and a change in bowel habits. The most recent formal criteria are the Rome II criteria, which require the presence of at least two of three symptoms occurring for 3 months (not necessarily consecutive) during a 12-month period. These symptoms include pain relieved with defecation, onset associated with change in stool frequency, or onset associated with change in the consistency of the stool. In clinical practice, these criteria have a positive predictive value of 98%. Importantly, she has no alarm indicators, including older age, male sex, nocturnal awakening, rectal bleeding, weight loss, or family history of colon cancer. In the absence of alarm symptoms, additional tests have a diagnostic yield of 2% or less. Furthermore, laboratory studies indicate no anemia or thyroid deficiency. Irritable bowel syndrome is a clinical diagnosis, and there are no laboratory, radiographic, or endoscopic findings that aid in diagnosis. Additional evaluation is not only unnecessary and expensive but also potentially harmful, especially when invasive procedures are ordered; additionally, confidence in the diagnosis is undermined when serial testing is ordered. The patient should be reassured that although this problem is annoying and inconvenient, it is not life-threatening. The patient has constipation-predominant irritable bowel syndrome, and her symptoms will likely be alleviated if she has more frequent and satisfying bowel movements. Because fiber supplementation has not been helpful, a nonabsorbed osmotic laxative such as polyethylene glycol will likely provide her significant relief. There is no indication for the patient to undergo a CT scan or colonoscopy. Oral contraceptives are not typically associated with the syndrome, and she began taking the medication after the onset of her symptoms.

Gastro 32 A 30-year-old woman is evaluated for a 9-month history of cramping midepigastric discomfort that is relieved by defecation; the discomfort is sometimes accompanied by bloating. The stool is often watery. She has not had fever, chills, or weight loss. The patient is otherwise healthy and takes no medications; there is no family history of gastrointestinal disease. On physical examination, the patient is afebrile; the blood pressure is 105/70 mm Hg, the pulse rate is 72/min, the respiration rate is 14/min, and the BMI is 23. The abdomen is soft and not tender or distended; the stool is brown and negative for occult blood. Complete blood count and serum biochemistry studies, including liver studies, vitamin B12, vitamin D, and thyroid-stimulating hormone, are normal. Which of the following is the most appropriate management for this patient? A Colonoscopy B CT enteroscopy C Gluten-free diet D Symptomatic management

D Irritable bowel syndrome is a diagnosis of exclusion but in the absence of alarm symptoms, invasive work up is not necessary This patient presents with symptoms that meet the Rome III criteria for irritable bowel syndrome (IBS). The Rome criteria were developed to establish consensus guidelines for diagnosis of functional bowel disorders. Criteria for IBS are symptoms of recurrent abdominal pain or discomfort and a marked change in bowel habit for at least 6 months, with symptoms experienced on at least 3 days a month for at least 3 months. Two or more of the following must also apply: (1) pain is relieved by a bowel movement; (2) onset of pain is related to a change in frequency of stool; and/or (3) onset of pain is related to a change in the appearance of stool. IBS is the most common gastrointestinal condition diagnosed in the United States. It is characterized by chronic abdominal pain and altered bowel habits in the absence of any organic cause. In this otherwise healthy young woman, reassurance that she has a chronic but not a life-threatening disease with recommendation of a high-fiber diet should be the initial therapy. CT enteroscopy or colonoscopy would be premature at this point given the absence of alarm symptoms: fever, weight loss, blood in stool, abnormal physical examination, family history of inflammatory bowel disease or colon cancer, or pain or diarrhea that awakens/interferes with sleep. This patient does not have evidence of malabsorption, anemia, or weight loss to suggest a diagnosis of celiac disease; therefore, an empiric gluten-free diet would be inappropriate.

Gastro 2 A 19-year-old woman is evaluated for a 2-week history of nausea and new-onset jaundice. Six weeks ago she had an uncomplicated cystitis, which resolved after a 3-day course of therapy with trimethoprim-sulfamethoxazole. On physical examination, the temperature is 37.3 °C (99.2 °F), the blood pressure is 120/85 mm Hg, the pulse rate is 88/min, and the respiration rate is 14/min; the BMI is 31. There is conjunctival icterus, jaundice, and right upper quadrant tenderness on deep palpation. Murphy sign is not elicited, and there is no asterixis or stigmata of chronic liver disease. Stool is negative for occult blood. Laboratory studies: Leukocyte count 7800/µL (7.8 × 109/L) with normal differential Bilirubin (total) 12.0 mg/dL (205.2 µmol/L) Bilirubin (direct) 5.6 mg/dL (95.6 µmol/L) Aspartate aminotransferase 23 U/L Alanine aminotransferase 35 U/L Alkaline phosphatase 464 U/L Antinuclear antibody titer Negative Anti-smooth muscle antibody Negative Antimitochondrial antibody Negative Ultrasonography of the right upper quadrant shows normal caliber of the hepatic ducts, a normal gallbladder without wall thickening, and no cholelithiasis. Which of the following is the most appropriate management for this patient? A Cholecystectomy B Endoscopic retrograde cholangiopancreatography C Liver biopsy D Observation

D Liver test elevations in the setting of a recently started medication should raise the suspicion of a possible drug induced liver injury. This patient has drug-induced liver injury secondary to trimethoprim-sulfamethoxazole therapy. Antibiotics are common causes of drug-induced liver injury, which can cause either an elevation in the aminotransferases or, as in this patient, a cholestatic form of liver injury. Some drugs have their own particular fingerprint of injury. For example, acetaminophen causes a predominant hepatocellular injury, whereas trimethoprim-sulfamethoxazole often produces a cholestatic form characterized by an alkaline phosphatase level more than twice normal. If a mixed pattern injury occurs in combination with an elevated alkaline phosphatase level, the patient is at increased risk of progressive liver injury. Phenytoin is an example of a drug that characteristically produces a mixed pattern. Drug-induced liver injury is often difficult to diagnose because there is no gold standard for diagnosis. Some drugs causing hepatic injury may be associated with the more familiar hypersensitivity syndrome characterized by fever, rash, and peripheral eosinophilia, but many drug reactions are characterized only by hepatic injury. Therefore, it is often a diagnosis of exclusion of other causes of liver injury in the presence of a potential offending agent taken within a recent period of time, usually weeks. Most patients can be monitored conservatively because discontinuation of the offending drug usually leads to eventual recovery, which can, however, take months. It is important to monitor for signs of progressive liver injury by monitoring prothrombin time and clinical status of the patient. Cholecystectomy is incorrect because the patient has no radiographic evidence to suggest cholecystitis, her gallbladder is normal on imaging, she has no Murphy sign, gallstones, leukocytosis, or anything else to suggest cholecystitis. Endoscopic retrograde cholangiopancreatography (ERCP) is not correct because the patient has no clinical or radiographic suggestion of choledocholithiasis. Proceeding to ERCP in the absence of clinical suspicion is unlikely to be effective and places the patient at up to a 5% risk of post-ERCP pancreatitis. Liver biopsy is incorrect because there is a good clinical suspicion that the patient has drug-induced liver injury. If, however, her condition does not improve or if the diagnosis were less clear, then biopsy may be correct. Another time when liver biopsy might be appropriate is in the patient with hepatocellular injury who may have autoimmune hepatitis, which could respond to corticosteroid therapy.

Gastro 3 A 35-year-old woman is evaluated for symptomatic ulcerative colitis. One year ago, she was diagnosed with pan-ulcerative colitis and responded well to initial and maintenance therapy with balsalazide. However, 2 months ago she developed urgent bloody diarrhea several times a day and lower abdominal cramping; prednisone, 40 mg/d, alleviated her acute symptoms, but her symptoms have returned with prednisone tapering. The patient is otherwise healthy, and her medications are balsalazide, 750 mg three times a day, prednisone, 15 mg/d, and calcium with vitamin D. On physical examination, vital signs and other findings are normal. Laboratory studies reveal hemoglobin 11.4 g/dL (114 g/L) and plasma glucose 140 mg/dL (7.77 mmol/L). Stool analysis for Clostridium difficile toxin A and B is negative. Which of the following is the most appropriate next step in the treatment of this patient? A Add olsalazine B Add budesonide C Add metronidazole D Increase prednisone dosage to 40 mg/d and add 6-mercaptopurine

D Patients with ulcerative colitis who become corticosteroid-dependent should be started on therapy with an immunomodulator, such as azathioprine or 6-mercaptopurine with a steroid taper. 5-Aminosalicylates (5-ASA) are the first-line therapy for ulcerative colitis, and remission can often be induced and maintained with a 5-ASA only. When 5-ASA therapy is not effective initially or patients develop a flare while in remission on 5-ASAs, often a short course of corticosteroids is required to induce or re-induce remission. However, corticosteroids are not effective as maintenance therapy and have many potential side effects, including hyperglycemia, osteoporosis, hypertension, mood instability, acne, infection, and osteonecrosis. Although some patients may maintain remission with continued 5-ASA therapy after corticosteroid taper, other patients become corticosteroid-dependent or -resistant, as did this patient, and therapy with an immunomodulator such as 6-mercaptopurine or azathioprine should be started. These agents are nucleotide analogues that interfere with DNA synthesis and induce apoptosis. Therapy with these agents may be required for up to 3 months before providing clinical benefit, and therefore, they are generally started with corticosteroids, which are then tapered. Because corticosteroids are not effective maintenance therapy, simply increasing the dose of prednisone without adding an immunomodulator would not be appropriate in this patient. The addition of another 5-ASA, such as olsalazine, will not provide any greater benefit. Antibiotic therapy has not been shown to be effective in the treatment of ulcerative colitis, and the patient's stool was negative for Clostridium difficile. Budesonide is a nonsystemic corticosteroid that is useful in the induction of remission in patients with ulcerative colitis disease involving the terminal ileum and right colon, but would not be of added benefit in this patient with pan-ulcerative colitis.

Gastro 66 A 25-year-old woman is brought to the emergency department by her husband for yellowing of the eyes and increasing confusion and somnolence. The patient is 30 weeks' pregnant and just returned from visiting her parents in Africa. She has been previously healthy and takes only prenatal vitamins. Previously a social drinker, she stopped consuming alcohol entirely at the onset of her pregnancy. On physical examination, the temperature is 37.2 °C (99.0 °F), the blood pressure is 90/40 mm Hg, the pulse rate is 100/min, and the respiration rate is 12/min; the BMI is 20. Examination reveals a gravid uterus and asterixis. Laboratory studies: Hemoglobin 14 g/dL (140 g/L) Leukocyte count 15,000/µL (15 × 109/L) Platelet count 450,000/µL (450 × 109/L) INR 4.7 Bilirubin (total) 12.0 mg/dL (205.2 µmol/L) Bilirubin (direct) 9.0 mg/dL (153.9 µmol/L) Aspartate aminotransferase 3000 U/L Alanine aminotransferase 2870 U/L Alkaline phosphatase 400 U/L Albumin 2.3 g/dL (23 g/L) Ammonia 120 µg/dL (70.4 µmol/L) Hepatitis A virus (IgM) Negative Hepatitis B surface antigen (IgG and IgM) Negative Hepatitis B virus DNA (PCR) Negative Hepatitis C virus antibody Negative Antinuclear antibody Negative Anti-smooth muscle antibody Negative Antimitochondrial antibody Negative Alcohol screen Negative Herpes simplex virus (PCR) Negative The peripheral blood smear is normal. Ultrasonography of the abdomen shows a small liver, no ductal dilatation, and no varices. Which of the following is the most likely cause of this patient's fulminant hepatic failure? A Acute autoimmune hepatitis B Acute alcoholic hepatitis C HELLP syndrome D Hepatitis E virus infection

D Pregnant patients and those with chronic advanced liver disease who become infected with hepatitis E virus are at risk for fulminant hepatic failure. This young pregnant woman has clinical and biochemical evidence of fulminant hepatic failure, which is the rapid development of severe acute liver injury with impaired synthetic function and encephalopathy in a previously normal person or a patient with well-compensated liver disease. She is encephalopathic and has profound coagulopathy with an INR greater than 4.0. Her elevated ammonia level is also a poor prognostic indicator. Her recent travel to Africa and the fact that she is pregnant should raise suspicion for acquisition of acute hepatitis E virus infection. Hepatitis E virus causes large outbreaks of acute hepatitis in underdeveloped countries. Clinically, hepatitis E is similar to hepatitis A. Hepatitis E is usually self-limited, and chronic infection does not occur. However, patients exposed to this virus especially in the pregnant state or in the presence of chronic advanced liver disease are at risk for developing fulminant hepatic failure. Unfortunately, there is no direct therapy and treatment is supportive care; in this patient management would include delivery of fetus and evaluation for liver transplantation if possible. Although autoimmune hepatitis and the HELLP syndrome (hemolysis, elevated liver enzymes, low platelets) can both lead to acute liver failure, the patient's platelet count is elevated and she does not have evidence of hemolytic anemia and has a normal peripheral blood smear and therefore, does not meet the criteria for HELLP syndrome. In addition, she does not have associated clinical evidence of pre-eclampsia (hypertension and proteinuria after 20 weeks of gestation) or eclampsia (preeclampsia and seizures (Krämpfe)) often seen with the HELLP syndrome. Although autoimmune hepatitis is a possibility, she has no history of previous liver disease or abnormal liver chemistry tests, and it would be unlikely to see a flare of autoimmune hepatitis in the pregnant state when the immune system is often suppressed by pregnancy. The clinical presentation of patients with alcoholic hepatitis ranges from being asymptomatic to developing jaundice, fever, and findings consistent with portal hypertension. Serum aspartate aminotransferase (AST) and alanine aminotransferase (ALT) values are less than 300 U/L, and the AST:ALT ratio is usually greater than 2. Mild to moderate serum alkaline phosphatase elevations are common. The lack of alcohol intake and the marked elevations of the aminotransferase levels with an AST:ALT ratio that is less than 2 excludes alcoholic hepatitis as the cause of her fulminant hepatic failure.

Gastro 52 A 57-year-old woman is evaluated in the intensive care unit for rapidly progressive renal failure requiring dialysis. The patient had been hospitalized for advanced liver disease including mental status changes secondary to encephalopathy. She has ascites. The liver disease is the result of chronic hepatitis C virus infection. The patient has no history of renal insufficiency and has not received antibiotics, intravenous contrast agents, or other nephrotoxic agents. Her medications are lactulose, nadolol, midodrine, octreotide, and albumin. She does not drink alcohol. On physical examination, the temperature is 36.6 °C (97.8 °F), the blood pressure is 110/70 mm Hg, the pulse rate is 97/min, and the respiration rate is 12/min; the BMI is 22. Laboratory studies: Creatinine 5.4 mg/dL (412.0 µmol/L) Urea nitrogen 120 mg/dL (42.8 mmol/L) Urine sodium less than 5 meq/L (5 mmol/L) Urinalysis Negative Ultrasonography shows normal-size kidneys and no obstruction. Which of the following is the most appropriate management for this patient? A Add lisinopril B Kidney and liver transplantation C Kidney transplantation D Liver transplantation E Peritoneovenous shunt

D The hepatorenal syndrome resolves with liver transplantation. This patient has the hepatorenal syndrome. The diagnosis is made in the absence of other causes of renal disease in the setting of advanced liver disease. Other causes of renal failure should be excluded by performing a careful history and physical examination, obtaining basic laboratory tests, and ruling out an infectious process (especially spontaneous bacterial peritonitis). Failure to improve following withdrawal of diuretics and administration of 1-1.5 L of normal saline is indicative of the hepatorenal syndrome. Renal replacement therapy is indicated for patients with clinically significant volume overload or severe electrolyte abnormalities. There are two basic forms of the syndrome: type 1, which is what this patient has, is rapidly progressive over a course of 1 to 2 weeks; type 2 more commonly occurs in the outpatient setting in patients who also have diuretic-resistant ascites. Type 1, however, leads to rapidly progressive renal dysfunction and oliguria ultimately necessitating the initiation of supportive dialysis. The hepatorenal syndrome is purely a reflection of advanced liver disease caused by severe renal vascular vasoconstriction in the setting of profound splanchnic vasodilation. The syndrome is not due to intrinsic renal disease and reverses with liver transplantation. Therefore, this patient should proceed with liver transplant alone, and the kidney function will return after liver function is restored. A peritoneovenous shunt drains peritoneal fluid from the peritoneum into the internal jugular vein and has been used both for the treatment of refractory ascites and the hepatorenal syndrome. Insertion of a peritoneovenous shunt has been shown to modestly improve the creatinine concentration but not survival. Furthermore, shunting is associated with the risk of complications and cannot be recommended for this patient. Angiotensin-converting enzyme (ACE) inhibitors such as lisinopril have been used in the treatment of hepatorenal syndrome without success. In addition, this class of drugs can induce hypotension and reduce the glomerular filtration rate. Angiotensin-converting enzyme inhibitors cannot be recommended for the treatment of hepatorenal syndrome.

Gastro 16 A 45-year-old woman is evaluated for a 2-week history of right leg and flank pain and a slight limp (Hinken). The patient was diagnosed with Crohn disease at age 20 years when she was evaluated for abdominal pain, nausea, and vomiting, and small-bowel radiographic series revealed a stenotic area in her terminal ileum with proximal dilatation and an enteroenteric fistula. She had an elective ileocolic resection with a primary anastomosis when her disease proved to be refractory to corticosteroid therapy. She has since been pain-free but has required intermittent courses of antibiotics and mesalamine for diarrheal flares of disease. She has had routine colonoscopic examinations with colonic biopsies showing scattered aphthous ulcerations and biopsy specimens revealing non-necrotizing granulomas in the colon and neo-terminal ileum. Her most recent colonoscopy was 5 years ago. She is otherwise healthy and takes no medication. On physical examination, the temperature is 38.0 °C (101.0 °F), the blood pressure is 120/80 mm Hg, and the pulse rate 90/min; there is right-sided abdominal tenderness and restricted painful extension of the right hip; otherwise, the range of motion of the hip is normal and without pain. She has an abnormal gait, favoring her right leg. Which of the following is the most likely diagnosis? A Avascular necrosis of the hip B Crohn disease-related arthritis C Osteoporosis-related fracture of the femoral neck D Psoas muscle abscess

D The inflammation in Crohn disease is transmural and can extend into any adjacent structure, including muscle. The prevalence of extraintestinal manifestations in patients with inflammatory bowel disease ranges from 21% to 40%. In most large studies, the prevalence is higher in Crohn disease than in ulcerative colitis. The right colon and transverse colon can abut the right psoas muscle, and the transmural inflammation of Crohn disease can lead to extension into the surrounding tissues and abscess formation. In this patient, who has a history of stricturing disease with fistula formation, it is likely that the fever, pain, and limp are complications of penetrating disease. Peripheral and axial musculoskeletal syndromes occur in approximately 30% of patients with inflammatory bowel disease and are considered part of the seronegative spondyloarthropathies. The course of peripheral arthritis tends to parallel the activity of the bowel disease. Isolated hip involvement would be an unusual extra-colonic manifestation of Crohn disease and it would not explain this patient's physical examination findings. Patients with inflammatory bowel disease are at increased risk for developing osteoporosis, secondary to the disease process itself and the medications commonly used for treatment (such as corticosteroids). The fracture risk in such patients is 40% greater than that of the general population. All patients with inflammatory bowel disease older than 65 years with a history of corticosteroid use for more than 3 months should undergo bone mineral density scan. Furthermore, both short- and long-term corticosteroid therapy is associated with avascular necrosis of the femoral head. However, this patient has not taken corticosteroids for 25 years, and neither steroid-related avascular necrosis nor osteoporosis is likely and cannot account for her fever and flank pain.

Gastro 84 A 42-year-old woman is evaluated in the emergency department for the acute onset of epigastric pain that radiates to the back and is associated with nausea and vomiting. The patient had previously been healthy and has no history of alcohol or tobacco use. Her only medication is an oral contraceptive pill. On physical examination, the temperature is 37.2 °C (99 °F), the blood pressure is 158/90 mm Hg, the pulse rate is 101/min, and the respiration rate is 20/min. There is no scleral icterus or jaundice. The abdomen is distended with mid-epigastric tenderness without rebound or guarding and with hypoactive bowel sounds. Laboratory studies: Leukocyte count 13,500/µL (13.5 × 109/L) Aspartate aminotransferase 131 U/L Alanine aminotransferase 567 U/L Bilirubin (total) 1.1 mg/dL (18.8 µmol/L) Amylase 824 U/L Lipase 1432 U/L Radiography of the abdomen shows mild ileus. Which of the following is the most appropriate next step in the evaluation of this patient? A CT scan of the abdomen and pelvis B Endoscopic retrograde cholangiopancreatography C Esophagogastroduodenoscopy D Ultrasonography of the abdomen

D The most common cause of acute pancreatitis in the United States is gallstones, which are diagnosed with abdominal ultrasonography. The diagnosis of pancreatitis relies heavily on the serum amylase and lipase, which are elevated in 75% to 90% of patients. Serum lipase is more specific and stays elevated longer than amylase. The two most common causes of acute pancreatitis in the United States are alcohol and gallstones. In this patient who does not consume alcohol, gallstones are the most likely cause of acute pancreatitis as shown by the pattern of liver enzymes. Abdominal ultrasonography is the most sensitive test for detecting the presence of gallstones and ductal dilation, which can provide indirect evidence for the presence of a retained common duct stone. Ultrasonography has no risk, is widely available, and is relatively inexpensive. CT scan is less sensitive than ultrasonography for the detection of cholelithiasis. CT with contrast is indicated in patients with moderate or severe pancreatitis or those who do not improve clinically within 48 to 72 hours, to confirm the diagnosis, to exclude other intra-abdominal processes, to grade the severity of pancreatitis, and to diagnose local complications such as pancreatic necrosis, pseudocyst, or abscess. Magnetic resonance cholangiopancreatography is used if there is a contraindication to intravenous radiocontrast. Endoscopic retrograde cholangiopancreatography (ERCP) is the most sensitive test for choledocholithiasis and can provide direct treatment by removing common duct stones. ERCP is indicated in patients with persisting pancreatitis, persistent elevation of aminotransferase levels, or dilated bile ducts suggesting the presence of retained bile duct stones. Stone extraction with biliary sphincterotomy improves the outcome, prevents further attacks of acute biliary pancreatitis, and reduces pancreatitis-related complications. However, in this patient there is not enough evidence yet that a common duct stone is still present to perform this more invasive test before ultrasonography. Upper endoscopy will not aid in determining the cause of acute pancreatitis.

Gastro 20 A 38-year-old man is evaluated for a 3-month history of bloating (blähen) and increased frequency of defecation. He has four to six bowel movements a day, including nocturnal bowel movements several times a week. The stool tends to be loose but there is no blood or melena. He has significant bloating but no abdominal pain. The patient underwent a Roux-en-Y gastric bypass procedure 2 years ago for morbid obesity; he lost 45.5 kg (100 lb) after the procedure. His weight stabilized 1 year ago, but he has lost 2.2 to 4.5 kg (5 to 10 lb) in the past few months. His medical history also includes hypertension, which resolved after the bariatric surgery, and his medications include a daily multivitamin with iron and daily oral calcium, vitamin D, and vitamin B12 supplements. The patient's sister has Crohn disease, and he has twin 15-month-old daughters who attend a daycare center. On physical examination, the blood pressure is 110/62 mm Hg, and the BMI is 29. He has redundant abdominal skin and healed abdominal incisions. The abdomen is not distended or tender; bowel sounds are normal. There is normal rectal tone and no palpable rectal abnormalities or perianal disease. Laboratory studies: Hemoglobin 11.7 g/dL (117 g/L) Mean corpuscular volume 103 fL Vitamin B12 185 pg/mL (136.5 pmol/L) (n 150-670pmol/l) Folate 30 ng/mL (67.9 nmol/L) (n 4-20 nmoll) C-reactive protein 0.6 mg/dL (6.0 mg/L) Tissue transglutaminase antibodies (IgA and IgG) Negative Stool culture and examination for fecal leukocytes and ova and parasites on three stool specimens are negative. Colonoscopy with terminal ileal examination is grossly normal. Esophagogastroduodenoscopy shows altered anatomy consistent with the bypass procedure; small-bowel biopsy specimens are negative. CT enterography is normal. Which of the following is the most likely diagnosis? A Celiac disease B Crohn disease C Giardiasis D Irritable bowel syndrome E Small intestinal bacterial overgrowth

E Small intestinal bacterial overgrowth is a late complication of bariatric surgery and should be considered in patients presenting with diarrhea, bloating, and features of malabsorption after such surgery. Patients who undergo bariatric surgery are at risk for specific early and late complications of the procedure; small intestinal bacterial overgrowth is a late complication that can manifest with bloating, diarrhea, and features of malabsorption. Having segments of small bowel excluded from the usual stream of gastric acid, bile, and proteolytic enzymes, which all act to decrease excess bacterial growth in the small bowel, is a risk factor for bacterial overgrowth. The diarrhea in bacterial overgrowth can be from deconjugation of bile salts from the intestinal bacteria leading to fat malabsorption as well as from decreased disaccharidase levels leading to carbohydrate malabsorption. Other clues in this patient include the macrocytic anemia, with a low serum vitamin B12 level (bacteria bind and consume vitamin B12 and cleave it from intrinsic factor) and an elevated serum folate level (intestinal bacteria synthesize folate). Even though the patient is taking oral vitamin B12 replacement, his level is still low, suggesting inadequate replacement versus excessive loss. Culture of the small intestine or breath testing can be done to substantiate the diagnosis; alternatively, an empiric trial of antibiotic therapy could be considered. Although celiac disease should be considered in any patient with diarrhea and features of malabsorption, the negative serologic studies and negative small-bowel biopsy rule out the diagnosis. Irritable bowel syndrome should not cause nocturnal stools, weight loss, and vitamin derangements. Crohn disease is possible given the patient's symptoms and family history of inflammatory bowel disease, but the negative endoscopic, histologic, and radiographic findings make this disorder unlikely. The patient has young children in diapers who attend a daycare center, and, therefore, giardiasis is in the differential diagnosis; however, stool ova and parasite testing on three occasions has been negative, with the yield for repeated testing close to 90%.

Gastro 61 A 40-year-old man undergoes a pre-employment physical examination and mentions that he has a 10-year history of heartburn and regurgitation that are not associated with dysphagia or weight loss and that are well-controlled with once-daily omeprazole therapy. He has stopped the omeprazole before, but his symptoms return. He has never had an endoscopic examination. His father was diagnosed with esophageal cancer at age 62 years, and the patient agrees to undergo screening for Barrett esophagus. He is otherwise healthy, and physical examination and laboratory evaluation are normal. Esophagogastroduodenoscopy reveals a small hiatal hernia but no features of Barrett esophagus. In addition, eight 2- to 4-mm polyps are detected in the body and fundus of the stomach; the polyps are uniform in appearance, and biopsy specimen of one of the polyps reveals a fundic gland polyp. Which of the following is the most appropriate next step in the evaluation of this patient? A Colonoscopy B Endoscopic ultrasonography C Helicobacter pylori serum antibody testing D Repeat endoscopy with removal of all polyps E No further testing

E Fundic gland polyps convey no defined cancer risk and require no follow-up. Fundic gland polyps, the most common form of gastric polyps, are small (1 to 5 mm in diameter), multiple (often <10), located in the body and fundus of the stomach, and convey no known cancer risk. The endoscopic description of this patient's polyps is classic for fundic gland polyps, with histologic confirmation of the diagnosis. In this setting, no further testing is recommended. Fundic gland polyps may occur more frequently in patients with familial adenomatous polyposis syndrome, in which case fundic gland polyps are often found in greater numbers, with higher risk of dysplasia, yet low cancer risk. Unless this patient had a known family history of familial adenomatous polyposis or had an unusually high number of polyps with or without dysplasia, colonoscopy would not be recommended simply because of the presence of the polyps. Gastric hyperplastic polyps often occur in the setting of chronic active gastritis and may regress with eradication of Helicobacter pylori; however, fundic gland polyps are not related to H. pylori infection and testing for the organism is not recommended in this setting. Testing for H. pylori is also not indicated in this patient because his symptoms are controlled with omeprazole. Although only one of the polyps was sampled, repeating the endoscopy to remove other polyps is not necessary because of the uniformity of the polyps on visual inspection and the fact that they convey no defined cancer risk. Endoscopic ultrasonography is similarly not required because these lesions have no defined risk for invasiveness.

Gastro 89 A 42-year-old woman is evaluated in the emergency department for lower chest pain and dyspnea that awoke her suddenly from sleep. The pain is moderately severe and does not radiate. The patient had laparoscopic Roux-en-Y gastric bypass for morbid obesity 10 days ago and was discharged from the hospital 5 days ago. She has had mild nausea and an average of five bowel movements a day since surgery. She also has a history of reflux esophagitis and hypertension; her medications include a multivitamin with iron, calcium, vitamin D, thiamine, vitamin B12, pantoprazole, metoprolol, and prochlorperazine. On physical examination, the patient appears to be in mild distress; she is afebrile, the blood pressure is 105/70 mm Hg, the pulse rate is 110/min, and the respiration rate is 22/min. The oxygen saturation is 88% with the patient breathing ambient air, and the BMI is 48. Cardiac examination shows regular tachycardia. The lungs are clear bilaterally. Abdominal examination shows several small, mildly tender surgical incisions with no surrounding erythema; there is no guarding or rebound, and bowel sounds are normal. Laboratory studies reveal a normal complete blood count, normal cardiac enzymes, and normal serum lactate. Electrocardiography shows sinus tachycardia without ST changes. Chest radiograph shows linear atelectasis at the lung bases. Abdominal radiograph shows a small amount of intraperitoneal air, which is unchanged from postoperative radiographs, and a normal bowel gas pattern. Which of the following is the most likely diagnosis? A Bowel obstruction B Gastrojejunostomy leak C Gastroesophageal reflux D Myocardial infarction E Pulmonary embolism

E Pulmonary embolism is an infrequent early complication after bariatric surgery, but it accounts for 50% of deaths in these patients. Pulmonary embolism is a potential early complication (within the first 30 days) of bariatric surgery, and although infrequent, it accounts for 50% of all deaths in such patients. In this patient who recently underwent bariatric surgery and who presents emergently with chest pain, dyspnea, tachycardia, tachypnea, and hypoxemia, pulmonary embolism is the most likely cause, and immediate evaluation and management are important. A postoperative bowel obstruction may occur secondary to internal hernias or adhesions, and patients with obstruction often present with nausea, pain, and abdominal distention. This patient has had stable nausea since surgery but has normal bowel sounds, is having bowel movements, and has normal abdominal imaging, making obstruction less likely. A patient with an anastomotic leak may also present with tachycardia and pain, but it is often associated with fever and leukocytosis, neither of which this patient has. Some intraperitoneal air would not be uncommon after a recent surgery, but comparison with past films, as in this case, should always be considered. Gastroesophageal reflux is a common cause of noncardiac chest pain, but would not explain the patient's tachycardia, tachypnea, and hypoxemia. Although myocardial infarction needs to be considered in any patient presenting with chest pain, the normal cardiac enzymes and lack of ischemic changes on electrocardiography make myocardial infarction unlikely.

Gastro 60 A 74-year-old man is evaluated for a 2-month history of recurrent melena. He has had recurrent 2 - or 3-day periods of black, tarry stool followed by normal stool for the next week. His most recent black stool was yesterday. During this time, he has developed anemia and required transfusions of a total of eight units of blood. He has not had chest pain, abdominal pain, nausea or vomiting, or bleeding. The patient's medical history includes hypertension and mild chronic obstructive pulmonary disease; his medications are enalapril, albuterol by metered-dose inhaler, and aspirin. On physical examination, the temperature is 37.0 °C (98.6 °F), the blood pressure is 132/74 mm Hg, the pulse rate is 84/min, and the respiration rate is 12/min; the BMI is 26.5. There is no scleral icterus and no abdominal distention or tenderness; bowel sounds are normal and there is no organomegaly. Rectal examination reveals black, tarry stool that is positive for occult blood. Laboratory studies reveal hemoglobin of 8.6 g/dL (86 g/L) with a mean corpuscular volume of 80 fL; leukocyte count, platelet count, prothrombin time, activated partial thromboplastin time, and INR are normal. Esophagogastroduodenoscopy on two occasions and a colonoscopy fail to identify a bleeding source. Which of the following is the most appropriate next step in the evaluation of this patient? A CT angiography B Double-balloon enteroscopy C Intraoperative endoscopy D Small-bowel barium examination E Wireless capsule endoscopy

E Wireless capsule endoscopy is the preferred test to identify a bleeding source following a negative evaluation with upper endoscopy and colonoscopy. This patient has obscure, overt gastrointestinal blood loss accompanied by anemia requiring blood transfusion. The most likely source of bleeding in this older patient is angiodysplasia from the small bowel, as endoscopy on two occasions as well as colonoscopy were nondiagnostic, and the melenic stool suggests a proximal gastrointestinal bleeding source. The next test would be a wireless capsule endoscopy. Double-balloon enteroscopy is an option in this patient but only after the capsule endoscopy has identified a potential bleeding source. The capsule study may direct a peroral or transanal route for the double-balloon enteroscopy, which can then be used for diagnosis and to treat a bleeding source. CT angiography and small-bowel barium radiography are insensitive for detecting sources of bleeding and therefore not likely to assist in the management of the patient. Intraoperative endoscopy would allow a detailed assessment of the bowel but should be reserved until after less invasive options have been carried out to avoid the morbidity and mortality resulting from surgery.


संबंधित स्टडी सेट्स

Consumer Behavior - WK 10 Quiz 9

View Set

Alterations in Cardiac Function Practice Questions

View Set

Ch.12 Neuromuscular Junction (Bio)

View Set

HazMat - Chemical and Physical Properties

View Set